You are on page 1of 1226

Absite review

Question 1
Which portion of the gastrointestinal tract is
most responsible for the absorption of iron?

A.
B.
C.
D.
E.

Stomach
Duodenum
Jejunum
Ileum
Colon

Answer 1
B. Maximal absorption of iron occurs in the
duodenum

Question 2
Which portion of the gastrointestinal tract is
most responsible for the absorption of bile
acids and folate?

A.
B.
C.
D.
E.

Stomach
Duodenum
Jejunum
Ileum
Colon

Answer 2
D. Ileum

Question 3
Which of the following parts of the duodenum
are not considered to be retroperitoneal?

A.
B.
C.
D.

1st proximal portion


2nd (descending)
3rd (transverse)
All of the above are retroperitoneal

Answer 3
A. The proximal portion of the first part of the
duodenum is intraperitoneal

Question 4
All of the following are true regarding small
bowel anatomy and physiology except:
A. Brunners glands produce an alkaline solution
B. Enterochromaffin cells are involved in 5hydroxytryptamine release, APUD , serotonin
production
C. Gastrin is produced by the parietal cells in the
stomach and are innervated by the vagus
D. The ileum is around 150 cm in length while the
jejunum is about 100 cm

Answer 4
C is false.
Gastrin is produced by antral G cells in the stomach and
occasionally by pancreatic cells
G cells are innvervated by the vagus.
Gastrin releasing peptide is released by the post-ganglionic
fibers of the vagus during parasympathetic stimulation
Bombesin also stimulates gastrin release
Gastrin stimulates enterochromaffin-like cells to release
histamine
Gastrin also targets parietal cells by increasing the amount of
histamine-> the parietal cells therefore increase HCl secretion

Answer 4
The other statements are true.
The jejunum is the maximum site of absorption, though
the terminal ileum absorbs B12, bile acids, and folate.
Brunners glands produce an alkaline solution
Enterochromaffin cells are involved in 5hydroxytryptamine release, APUD , serotonin
production
The ileum is around 150 cm in length while the jejunum
is about 100 cm

Question 5
A 52 yo who has had a complicated history of
multiple gastric and pancreatic procedures
complains of loose stools that float. Which of the
following is true of this condition?
A. It can be caused by hyposecretion of gastric acid
B. You should recommend higher intake of fats to
compensate for the loss
C. Is a known sequlae of extensive terminal ileum
resection
D. Pancrease is not effective

Answer 5
C. Steatorrhea is a known sequlae of terminal
ileum resection. Interruption of bile salt
resorption interferes with micelle formation.
Gastric hyper (not hypo) secretion of acid
increases intestinal motility and interferes with
fat absorption
Treatment involves controlling diarrhea (Lomotil),
decrease PO intake (esp fats), Pancrease, and H2
blockers

Question 6
Which of the following amino acids is a major
metabolic fuel for enterocytes?

A.
B.
C.
D.

Alanine
Glycine
Leucine
Glutamine

Answer 6
D. Glutamine is a major metabolic fuel for
enterocytes and may play an important role in
modulating the cytokine release from
intestinal lymphocytes

Question 7
You just performed a gastrectomy on a patient that had
previously been in-house receiving pre-operative TPN @ 90
cc/hr. 4 hours post-op, the patient seems unusually
somnolent, somewhat confused, and diaphoretic. His blood
glucose returns as 35 mg/dL. Which of the following is likely
to be true of this condition?
A. It is unusual to see symptoms of hypoglycemia when serum glucose
levels are above 25 mg/dL
B. This was probably caused by stopping the TPN altogether during
the operation
C. TPN should not be administered intraoperatively as operative stress
puts the patient on TPN at risk for hyperosmolar, nonketotic coma
D. This is likely due to post-operative dumping syndrome

Answer 7
B. Most likely, this patient is hypoglycemic because
the TPN was not ran at all during the operation.
TPN associated hypoglycemia is caused by the sudden
slowing of TPN administration, and is the most
common cause of hypoglycemia other than excessive
insulin administration
Blood glucose levels less than 50 mg/dL can produce
profound symptoms (dizziness, coma, arrhythmia,
hypotension, diaphoresis)

Answer 7
It is usually safe to run TPN @ ~ 50 cc/ hr
intraoperatively- it is unlikely that this will lead to
hyperglycemia that will cause nonketotic coma
10% dextrose can be given when TPN is to be
stopped
Weaning from tPN can usually be stopped when
the infusions rates are 50 cc/hr or less

Question 8
Which of the following is NOT true about pancreatic
exocrine secretions?
A. The principal cations are sodium and potassium
B. The concentration of the principal anions (bicarb and
chloride) varies depending on pancreatic stimulation
C. When the stimulus to secrete fluid is minimal, the
chloride concentration is relatively high
D. When the stimulus to secrete is maximal, the chloride
concentration is relatively high

Answer 8
D is incorrect. When the stimulus for the pancreas to
secrete is maximal, the chloride concentration is
relatively low, not high

Pancreatic Exocrine
The principal cations of pancreatic juice are sodium
and potassium
The concentration of the principal anions (bicarb
and chloride) varies depending on pancreatic
stimulation
When the stimulus to secrete fluid is minimal, the
chloride concentration is relatively high and bicarb
is low
When the secretory stimulus is maximal, the bicarb
concentration rises and chloride falls

Pancreatic Exocrine
The above is a result of the passive exchange of intraductal
bicarb for interstitial chloride
This exchange occurs as juice flows through the larger pancreatic ducts on its
way to the duodenum.
At slower rates, there is more opportunity for exchange to take place and bicarb
to subsequently be lost

The alkaline juice helps to neutralize gastric acid in the duodenum, thus
providing optimal pH for the activity of pancreatic digestive enzymes.

Question 9
Which of the following about bacterial sepsis is
NOT true?
A. Gram positive septicemia usually has a poorer
prognosis than gram negative infection
B. Lipid A of the LPS endotoxin is an importent
component for virulence
C. TNF can cause self-stimulation of monocytes and
macrophages to their full state of activation
D. IL-1 may stimulate T lymphocytes to produce TNF

Answer 9
A is not true. Gram NEGATIVE septicemia
usually has a poorer prognosis than gram +
infection

Gram Negatives/ TNF


Gram negative septicemia usually has a poorer
prognosis than gram + infection
The endotoxin, which represents the LPS outer
membrane of these bacteria usually contains the
lipid A moiety, which is important in virulence
TNF is produced primarily by T lymphocytes and
macrophages in response to many stimuli, including
IL-1, interferon, LPS, viruses, and BCG

Gram Negative Sepsis/ TNF release


In gram infections, a stimulated macrophage
produces TNF.
TNF can cause self-stimulation of monocytes and
macrophages to their full state of activation
Large array of cytokine and inflammatory signals
develop
The paracrine function of TNF is to stimulate
neutrophils to full activation

Question 10
A 45 yo male underwent extensive adhesiolysis with multiple
serosal tears. The operation took around 4 hours and at the
end of the case, his abdomen was slightly difficult to close. He
developed an ileus post-op, a wound dehiscence requiring
takeback, and then on day 9 started draining succus from his
midline excision. Which of the following is not true?
A. Colonic fistulas are more likely to close compared to SB.
B. High output fistula are more likely to be from the proximal bowel
and less likely to close with conservative management.
C. 80% close spontaneously
D. Iatrogenic enterocutaneous fistulas are more common than
spontaneous fistulas from inflammatory bowel disease

Answer 10
C. Is not true. 40% of fistulas close spontaneously
Colonic fistulas are more likely to close compared to SB.
High output fistula are more likely to be from the proximal
bowel and less likely to close with conservative
management.
Most fistulas are iatrogenic
Tx initially with NPO, bowel rest, TPN, octreotide, output
management
Best surgical option is takedown and resection

Answer 10

FRIENDS
Foreign body
Radiation
IBD
Epithelization
Neoplasm
Distal obstruction
Sepsis

Question 11
Which of the following about bowel obstructions is
not correct?
A. The most common cause of colonic obstruction without
previous surgery is cancer
B. The most common cause of SBO in a patient without
surgery is cancer
C. The most common cause of SBO with a history of
previous surgery are adhesions
D. The most common cause of a SBO in a patient with no
history of abdominal surgery is an inguinal hernia

Answer 11
B is false. The most common cause SBO with
no history of abdominal surgery is a hernia
(not malignancy)
Most common cause of SBO with a surgical hx
is adhesions
Most common cause of colon obstruction with
or with no PSuH is cancer

Question 12
Which of the following is associated with vinyl
chloride exposure?

A.
B.
C.
D.

Inflammatory bowel disease


Ovarian cancer
Angiosarcoma of the liver
Leukocytosis

Answer 12
C. Angiosarcoma of the liver is associated with
vinyl chloride exposure.
Vinyl chloride is a known carcinogen of the
lung, liver, and brain.
Is a chemical used in the plastics industry

Question 13
Which of the following types of cautery
generates less heat on a slower frequency
causing tissue dehydration and vessel
thrombosis?

A.
B.
C.
D.

Cutting mode
Coagulation mode
Bipolar mode
None of the above

Answer 13
B. Coagulation mode generates less heat on a
slower frequency causing tissue dehydration
and vessel thrombosis

Question 14
Which of the following types of cautery leads
to heat being generated over the target area
quickly with minimum lateral spread?

A.
B.
C.
D.

Cutting mode
Coagulation mode
Bipolar mode
None of the above

Answer 14
A. This best describes the cutting mode

Question 15
Which of the following types of cautery is the
safest because it disperses energy only
between its tips?

A.
B.
C.
D.

Cutting mode
Coagulation mode
Bipolar mode
None of the above

Answer 15
C. This best describes bipolar mode- this mode
is often safest as it only affects tissues within
the circuit or grasp of the instrument

Question 16
Which of the following factors is NOT significant in
the amount of heat generated by a cautery device?
A.
B.
C.
D.
E.

Size of the contact area


Power setting (frequency)
Length of activation time
Patient BMI
Whether the waveform is continuous or intermittent

Answer 16
D. Patient BMI has the least to do with the amount of
heat a cautery device can generate

Safest Electrosurgery Settings


High frequency alternating current is delivered as either unipolar or
bipolar.
Unipolar device has a generator and electrode
When the system is activated the patient becomes a part of the circuit
The energy is converted to heat inversely compared to the area of contact
Thus, the application electrode is small compared to the returning
electrode to disperse the energy to prevent burning the patient

In the cutting mode, heat is generated over the target area quickly
with minimum lateral spread (heating without coagulation.)
In the coagulation mode, the electrocautery generates less heat on
a slower frequency, thus causing tissue dehydration and vessel
thrombosis

Safest Electrosurgery Settings


Bipolar devices establish a short circuit between the tips of the
instrument without the need for a grounding pad.
These are the most efficient and often safest as it only affects tissues within
the circuit or grasp of the instrument.

Overall, the heat generated is dependent on 4 factors

Size of the contact area


Power setting (frequency)
Length of activation time
Whether the waveform is continuous or intermittent

Other types of electro surgery include lasers, photodynamic therapy,


argon beam coagulator, high frequency sound wave techniques,
harmonic scalpel, ultrasonic tissue ablation, RFA, cryoablation,
microwave ablation, and ultrasonic cavitation devices

Question 17
A 35 yo female with a known history of HSV and HPV in
the past presents with pelvic pain, fever, and
tachycardia. After you review her CT A/P you are
concerned for a small, contained tuboovarian abscess.
Which of the following management plans is most
appropriate at this time?
A. Send UA, B-HCG, G & C cultures, and give her 14 days of PO augmentin
B. Post her for a diagnostic laparoscopy
C. Send UA, B-HCG, G & C cultures, admit her and initiate cefoxitin and
doxycycline
D. Send UA, B-HCG, G & C cultures and consult interventional radiology for
aspiration

Answer 17
C. In this patient with a contained TOA, send
UA, B-HCG, G & C cultures, admit her and
initiate cefoxitn and doxycycline

Question 18
A 35 yo female that has been treated for
multiple STDs presents with pelvic pain, fever,
and tachycardia. Which of the following are
indications to operate?
A.
B.
C.
D.

Intraperitoneal rupture of a TOA on scan


Persistence of pelvic abscess despite abx therapy
Chronic pelvic pain
All of the above are indications to operate

Answer 18
D. All of the following are indications to
operate on a patient with PID

Question 19
You work the above patient up and find a clearly
ruptured TOA. She states that she has one child
and would like to try to have another within the
next year. Which of the following is the most
reasonable treatment option?

A.
B.
C.
D.

Observation and IV abx


Unilateral salpingo-oophorectomy
Hysterectomy and salpingooophorectomy
Transvaginal drainage

Answer 19
B. Unilateral salpingoophorectomy should be
attempted if possible

Treatment of Pelvic Inflammatory Disease (PID)


PID is largely limited to sexually active females and is
classified as acute or chronic.
The most common organisms that produce this
infection are N. Gonorrhoeae and Chlamydia
Treatment PID can be treated as an in- or outpatient depending on the severity
Patients with peritonitis, high fever, or suspected tub-ovarian abscess (TOA)
should be admitted for IV abx Cefoxitin 2 g IM with oral probenecid or
equivalent cephalosporin for Gonorrhea.
Doxycycline is used to cover chlamydia

Treatment of Pelvic Inflammatory Disease (PID)


Surgical therapy becomes necessary when there is:
Intraperitoneal rupture of a TOA
Persistence of pelvic abscess despite abx therapy
Chronic pelvic pain

Unilateral salpingo-oophorectomy is the preferred


surgical tx for young women with reproductive goals
With a ruptured TOA,, hysterectomy and
salpingooophorectomy are commonly indicated.

Question 20
A 42 yo male is s/p MVA and just underwent a panCT. The only significant finding is a moderate-sized
duodenal diverticula. The gallbladder otherwise
appears normal. He does not report any history of
abdominal pain or GI problems. Which of the
following should you recommend?

A.
B.
C.
D.

Resection with gastrojejunostomy


Resection with primary closure
Observation
ERCP with biopsy of the diverticulum base

Answer 20
C. In this asymptomatic patient, observation is
appropriate

Treatment Duodenal Diverticulum


Duodenal diverticula
Need to r/o gallbladder disease
Observe unless perforated, bleeding, obstructing, or
symptomatic
Frequency of SB diverticula- duodenum > jejunum >
ileum
Needs segmental resection though possibly
gastojejunostomy if perforated

Question 21
You operate on a 35 yo male with a SBO and find a
lesion approximately 2 ft proximal to the ileocecal valve
that appears to be the point of obstruction. There is an
outpouching of the bowel is markedly inflamed
circumferentially at this point. Which of the following is
not true?

A.
B.
C.
D.

Segmental resection is appropriate


This is caused by failure of the omphalomesenteric duct to close
Is most likely to present as SBO in both children and adults
There is a high chance you will find pancreatic tissue in this lesion

Answer 21
C is not true- though it is more likely to present
as a SBO in adults, in children it often presents
as a painless lower GI bleed.

Meckels Diverticulum
Segmental resection is appropriate when there is
complicated diverticulitis, inflammation involving
the base, or the neck is 3 the diameter of the
small bowel
It is caused by failure of the omphalomesenteric
duct to close
Is most likely to present as SBO in adults but is
may present as a painless LGI bleed in children
Accounts for 50% LGIB in children < 2 yrs

Meckels Diverticulum
There is a high chance you will find pancreatic
tissue in this lesion as it is the most common tissue
found
Gastric tissue is more likely to be symptomatic
Do not have to remove incidental diverticulum
Is a true diverticulum
2% IC valve, 2% population, usually presents in first
2 years of life with bleeding

Question 22
A 42 yo male is on cyclosporine as a part of his
immunosuppressive regimen s/p renal transplant.
Which of the following is true about this drug?

A.
B.
C.
D.

It is mostly eliminated through the kidneys


It inhibits both resting and active T lymphocytes
It is an amine that is a derivative of methotrexate
It inhibits the activation of resting T lymphocytes

Answer 22
D. It inhibits the activation of resting T lymphocytes

Question 23
Which of the following would you least likely
expect as a side-effect?
A.
B.
C.
D.

Hyperkalemia
Nephrotoxicity
Tremors
Hypokalemia

Answer 23
D. Hypokalemia

Cyclosporine
Discovered in 1972; relatively new class of
immunosuppressive agents
Cyclic peptide produced by a fungus
Many of its effects are T-cell specific
Inhibits activation of resting T-lymphocytes
resulting in an inhibition of IL-2 production
However, once T-lymphocytes are activated,
cyclosporine is not effective in suppressing the
immune response

Cyclosporine

Absorbed slowly and incompletely from GI tract


Excretion is primarily through bile
Well-documented enterohepatic cycle
Adverse effects:

Hirsuitism
Neurotoxicities
Hyperkalemia
Nephrotoxicity
Hepatotoxicity
Nephrotoxicity, hypertension, and tremors most common

Question 24
Which of the following factors are not a part of
the Harris-Benedict equation for energy
expenditure?
A.
B.
C.
D.
E.

Weight
Height
Body temperature
Sex
Age

Answer 24
C. Body temperature is not a variable in the
equation of basal energy expenditure
Basal energy expenditure (BEE)
Male= 66.5 + 13.7 x weight (kg) + 5.0 x height (cm)
6.8 x age (yr)
Female- 655.1 + 9.56 x weight + 1.85 x height (cm)
1.68 x age (yr)

Question 25
Fill in the blanks:
Protein generates ( A ) calories per gram and it
takes ( B ) grams of protein to produce 1 gram of
nitrogen.
The resting energy expenditure for most patients is
(c) calories per kg per day. Most non-protein
calories are provided by (D- choose from lipids,
glucose, or alcohol).
Fat provides (E) calories per gram where proteins
and carbohydrates each produce around (F) calories
per gram

Answer 25

A- 4
B- 6.25
C- 25
D- glucose
E- 9
F- 4

Energy Requirements
Harris-Benedict Equation- determines basal energy
expenditure
Formulas based on body weight, height, sex, age
Metabolic cart measurement- determines energy
expenditure from oxygen utilization and CO2
generation
Basal energy expenditure (BEE)
Male= 66.5 + 13.7 x weight (kg) + 5.0 x height (cm) 6.8 x
age (yr)
Female- 655.1 + 9.56 x weight + 1.85 x height (cm) 1.68 x
age (yr)

Energy Requirements
Resting energy expenditure of most pts- 25 kcal/
kg/day
In stressed pts this may increase to 35-40 kcal/kg
Adequate caloric replacement can be determined
by frequent metabolic measurements such as
nitrogen excretion
This represents gluconeogenesis of protein to meet energy
requirements if adequate non-protein calories are
unavailable

Question 26
Fill in the blanks:
A normal, healthy man needs about (A) grams
of protein per kg of body weight per day.
The ratio of grams nitrogen to non-protein
calories should be around (B:C)

Answer 26
A- A normal, healthy man needs ~ 1 g protein
per kg per day
B: 6.25 g protein will have 1 g nitrogen
C. 1 gram nitrogen : 100 non-protein calories is
appropriate nitrogen balance

Question 12
To maintain optimal nitrogen balance
(assuming you are going to give glucose to
provide all of the non-protein calories), how
many grams of glucose will a 72kg man need
daily?
A.
B.
C.
D.

350 g
400 g
450 g
500 g

Answer 12
C. 450 grams of glucose
Since 1 gram of nitrogen should be given for 100
non-protein calories, it would take 6.25 grams of
protein and 25 grams of glucose to provide this
Assuming his energy requirements are ~ 25
kcal/kg/day ->
This amounts to 1800 nonprotein calories per day (72 kg x
25 kcal/kg/day)
Since glucose produces 4 calories per gram-> 1800 cal/ 4
cal/g = 450 gm glucose

Question 27
On the same 72 yo male, to maintain nitrogen
balance, how many grams of protein should he
be provided daily?
A. 115 g protein
B. 122.5
C. 125
D. 130

Answer 27
B. 122.5 grams of protein.
Remember the ratio of 6.25 grams protein to
25 grams glucose?
Therefore, the 450 grams glucose we calculated
should be divided by 25- gives you a factor of
18
For protein, 6.25 grams is now multiplied by 18
to give 122.5 grams of protein

Protein Requirements in TPN


Determination of the patients protein requirement is the
first step in formulating a nutritional support plan
Normal, active man needs 0.9- 1.5 grams protein per kg of
body weight per day
6.25 protein make 1 g nitrogen
Requirements dependent on clinical state
1 gm/kg/day needed early in refeeding after starvation
2-3 g/ kg/day in burned or severely septic pts
Total intake may be limited to 40-50 g/day in liver failure

Protein Requirements in TPN


Nitrogen / non-nitrogen protein calories 1:100 1:200 to obtain optimal
nitrogen balance
Example:
1:100 = 1 gram N to 100 calories
OR
6.25 g protein to 25 g glucose
A 70 kg male will require:
Non-protein calories- 70 kg x 25 = 1750 cal/day

Question 28
Which of the following about Crohns disease is not
true?
A. The most commonly involved bowel segment is the
terminal ileum
B. Anal disease can be manifested as large skin tags and
should be treated with Flagyl, not excision
C. Less than of these patients eventually need an
operation
D. Perirectal disease may respond to resection of diseased
small bowel

Answer 28
C is false- most patients with Crohns ultimately
need an operation

Crohn Disease
The most commonly involved bowel segment is the
terminal ileum
Anal disease can be manifested as large skin tags and
should be treated with Flagyl, not excision
90% of these patients eventually need an operation
Perirectal disease may respond to resection of diseased
small bowel
Intermittent abdominal pain, n/v/d/f, weight loss
Even after resection, recurrence rate is around 50%

Question 29
Which of the following characteristics are not
consistent with Crohns Disease?
A.
B.
C.
D.
E.

Mucosal only involvement


Skip lesions
Cobblestoning
Creeping fat
fistulas

Answer 29
A. Crohn disease tends to have transmural
bowel involvement

Crohn Disease
Extraintestinal manifestations

Arthritis, pyoderma gangrenosum,


Erythema nodosum, ocular disease
Growth failure
Megaloblastic anemia from B12 and folate malabsorption

Pathology

Transmural involvement
skip lesions
cobblestoning
narrow deep ulcerations
creeping fat
fistulas

Crohn Disease
Medical agents 5-ASA
Infliximab (Remicade)- a TNF-alpha inhibitor (usually
for abscess & fistula)
Steroids
Azathioprine
Loperamide

Question 30
A 40 yo woman with a long standing history of
Crohn disease is found to have a perianal fistula
above the dentate line involving the sphincter
muscles. The fistula is well-drained and there is no
sign of pelvic sepsis. What is the best option for
treatment?

A.
B.
C.
D.
E.

Seton placement
Anti-TNF antibody
Treatment with cipro and flagyl
Fistulotomy
Diverting colostomy

Answer 30
B. Anti-TNF antibody would be most
appropriate

Perianal Fistula in Crohns


Perianal disease affects ~ 1/3 of patients with
Crohns
Management- optimize medical treatment, then
surgery
Fistulas above dentate line involving the
sphincter are complex
Medical mgmt w/ anti-TNF ab preferable 1st choice
Fisulotomy increases risk of incontinence

Perianal Fistula in Crohns


Treatment with flagyl and cipro is best for noncomplex fistulae near the anus
Diverting colostomy and advancement flap- for
more advanced disease that fails mgmt
Seton- important to consider in cases of pelvic
sepsis

Question 31
A 32 yo male has had multiple ileal resections
secondary to distal small bowel obstruction from
Crohns. Which of the following conditions is he at
risk for developing?
A.
B.
C.
D.

Calcium oxalate kidney stones


Macrocytic anemia
Calcium bilirubinate gallstones
All of the above

Answer 31
D. All of the above are sequlae of extensive
ileal resection

Crohn Disease
Surgical indications

Obstruction- partial can be tx conservatively


Abscess- sometimes tx with perc drainage
Megacolon- 15% perforation, usually contained
Blind loop obstruction
Fissures- Setons and conservative measures preferable; do not do lateral
internal sphincerotomy
Enterocutaneous fistula
Anorectal/ vaginal fistulas- may need advancement flap or diversion
Do not need clear margins, just get 2 cm away from gross disease

Crohn Disease
Complications from terminal ileum removal:
Megaloblastic anemia from decreased B12 uptake
Decrease bile salt uptake- increased bile salt delivery
to colon-> steatorrhea
Decrease oxalate binding- increased intraluminal fat
binds calcium & allows for oxalate absorption
Gallstones (calcium bilirubinate) from malabsorption
of bile acids

Question 32
A 45 yo male presents with right lower
quadrant pain; you end up exploring him &
find a 2cm carcinoid that is close to the
appendiceal base. You next perform:
A. Close surveillance
B. Right hemicolectomy
C. Appendectomy
D. Right hemicolectomy with removal of at least 10
cm terminal ileum

Answer 32
B. Right hemicolectomy is appropriate.
For appendiceal carcinoids:
Appendectomy adequate if less than 2 cm
Need right hemicolectomy if near the base or
greater than 2 cm

Question 33
He does well for a year at which point he returns
because of intermittent flushing and diarrhea. Though
your first CT doesnt show anything, an octreotride scan
suggests there is a small mass in the left lateral
segment of the liver. Which of the following is not
true?
A. If this anatomically appears resectable, you should attempt to do so.
B. Cholecystectomy should be performed in case he needs embolization in the
future
C. Octreotide can be helpful in managing symptoms
D. You should not try to resect any metastatic carcinoids

Answer 33
D. It is appropriate to attempt resection of
metastatic carcinoids.

Carcinoids
Treatment Should be approached oncologic ally like other GI tract cancers- segmental
resection and lymphadenectomy
Cholecystectomy should be performed in patients with resectable liver mets in
case they need embolization in the future
Octreotide can be helpful in managing symptoms
Pentagastrin can exacerbate symptoms
Bronchospams- tx w/ aprotinin
Flushing- tx w/ alpha-blockers
Chemotherapy (streptozocin and 5FU) for pts with unresectable mets and
carcinoid syndrome

Question 34
A 75 yo man is sent to your clinic with complaints of food
sticking in in throat, embarrassing bad breath, and a nagging
cough. Of note, he recently was hospitalized with aspiration
pneumonia. Which of the follow are not true about his
condition?
A. It is caused by a pulsion diverticulum that arises adjacent to the
inferior pharyngeal constrictor
B. Gastrograffin swallow (instead of barium) should be performed
because gastrograffin is inert if aspirated
C. Surgical intervention is indicated in symptomatic patients with this
condition regardless of size
D. Open repair is performed through the left neck and always involves
a myotomy through the cricopharyngeus and thyropharyngeus
muscles

Answer 34
B. is incorrect. Barium is the usual agent used
because it is inert if aspirated.

Question 35
Which of the following about Zenkers
diverticulum is true?
A. Surgical repair is usually successful
B. Large (> 2 cm) diverticula do not need to be
resected
C. They are full-thickness diverticula
D. Most successful treatment involves esophageal
resection

Answer 35
A. Most patients do well after myotomy +/resection of the diverticulum

Zenkers Diverticulum
Usually presents in older patients
Thought to be due to a loss of tissue elasticity and
muscle tone with age
Mucosal and submucosal layers herniate between
thyropharyngeus fibers and horizontal cricopharyngeus
fibers
Usually dissects down the left side of neck
Symptoms- nagging cough, excess salivation,
regurgitation of foul-smelling undigested material,
halitosis, voice changes
More severe- respiratory infections lung abscess,
aspiration pneumonia

Zenkers Diverticulum
Dx by barium swallow
EGD not necessary to diagnose but is performed to
evaluate anatomy and r/o cancer
Surgical therapy is indicated in symptomatic patients
regardless of diverticulum size
Myotomy should always be performed
Diverticulum larger than 2 cm should be excised
Alternative treatments- diverticulopexy, endoscopic
obliteration of the common wall, botox injection

Zenkers Diverticulum
Complications- RLN trauma, fistulas, hematoma,
infection, aspiration, recurrence
Fistulas usually controlled once prevertebral space
drained
Aspiration most serious
Division of cricopharyngeus may lead to massive
tracheobronchial aspiration in someone with incompetent
GE jxn
Severe GER may be relative contraindication to this
procedure until corrected

Zenkers Diverticulum
90% do well with surgical treatment
Those without diverticula or with poor
pharyngeal contractility may not do as well
Open repair vs. endoscopic For diverticula less than 3 cm, surgical repair
superior
For diverticula greater than 3 cm, results are similar

Question 36
An 8 yo male is brought to you for evaluation of a
midline neck mass that moves with swallowing. He
otherwise reports no complaints or symptoms. You
suspect a thyroglossal duct cyst. Which of the following
is NOT true about this condition?
A. May occasionally find an external fistula on the skin
B. When papillary cancer is found in a TG cyst, there is a 25% chance that
papillary cancer exists elsewhere in the thyroid
C. Squamous cell cancer is the most common that will arise from these cysts
D. Almost always require excision

Answer 36
C is false.

Thyroglossal Duct Cysts


Remnants of the tract along which the thyroid gland
descended into the neck from the foramen cecum
Account for ~ 70% of all congenital abnormalities of the neck
May be found in any age; most common in the first decade of
life
Can be a lone cyst, a cyst with a sinus tract, or a solid core of
thyroid tissue
Variable in size
Almost always found in the midline, at or below the level of
the hyoid bone
Can occasionally be slightly lateral with external skin fistula

Thyroglossal Duct Cysts


Differential dx Dermoid cysts, lymphadenomegaly in the anterior jugular chain, and
cutaneous lesions

Operative treatment indicated both due to cosmetic


considerations & high incidence of recurrent infection,
including abscess formation.
1% of thyroglossal duct cysts contain cancer
Papillary cancer most common, then squamous cell

25% of patients with papillary thyroid cancer in thyroglossal


duct cysts have papillary thyroid cancer in other parts of the
thyroid gland
About 10% have nodal metastases, occasionally bilateral

Question 37
A 2 year old boy presents with a compressible
neck mass that transilluminates. You suspect this
is a cystic hygroma. Which of the following is true
about this condition?
A. Usually present with pain and compressive symptoms
B. 1/3 harbor focus of squamous cell carcinoma so
should be excised
C. Most often present in 2nd decade of life
D. Often recur after excision

Answer 37
D is correct

Cystic Hygroma
Lymphangioma that arises from vestigial lymph channels
in the neck
Almost always noted by the second year of life
Range from simple thin-walled cyst in the floor of the
mouth to involvement of all tissue of mouth to
mediastinum
Painless cyst in the posterior cervical triangle or
supraclavicular area in 80%

Cystic Hygroma
Diffuse, soft, doughy, irregular mass
Transilluminates
May feel like lipomas but have less well-defined
margins
Aspiration- straw-colored fluid
Can be confused with angiomas (which are
compressible), pneumatoceles from the apex of the
lung, or aneurysms
Differentiate from vascular lesions by arteriography.

Cystic Hygromas
Occasionally grows suddenly as a result of an upper
respiratory tract infection, infection of the hygroma
itself, or hemorrhage into the tissues
Can compress the trachea and cause dysphagia.
May be treated expectantly if asymptomatic
Tend to regress spontaneously
Reserve excision for those that are symptomatic or do not regress

Satellite lesions and extension into surrounding


structures can make complete excision difficult
Recurrences common
Staged resections often necessary

Question 38
A 67 yo male presents with a right cervical enlarged
lymph node. FNA reveals squamous cell carcinoma.
You suspect that it is somewhere in the head or neck
given his long smoking history but despite extensive
workup, you cant find the primary. No distant disease
is found. Which of the following is the most
appropriate initial treatment?
A. Chemotherapy and radiation
B. Tracheostomy and PEG

C. Right sided neck dissection with radiation


D. Bilateral neck dissection

Answer 38
C is most appropriate

Mass With Unknown Primary


When cervical lymph nodes are found to contain
metastatic squamous cell carcinoma, the primary
tumor is in the head and neck about 90% of the time
If no primary tumor is identified, the patient should
undergo endoscopic evaluation of the nasopharynx,
the hypopharynx, the esophagus, the larynx, and the
tracheobronchial tree under general anesthesia
Biopsies of the nasopharynx, the tonsils, and the
hypopharynx often identify the site of origin

Mass With Unknown Primary


If the biopsies do not reveal a primary source of
cancer, treatment should be unilateral neck
dissection, followed by radiation therapy directed
toward the neck, the entire pharynx, and the
nasopharynx
15 to 20% of cases, the primary cancer is
ultimately detected
Overall 5-year survival in such cases ranges from
25 to 50%.

Mass With Unknown Primary


Malignant melanoma found in a cervical lymph
node but no primary tumor is evident
Ask about previous skin lesions & repeat head and neck
examination (pay attention to the scalp, the nose, the
oral cavities, and the sinuses)
Ophthalmologic examination
If physical examination and radiographic studies find no
evidence of metastases, modified neck dissection should
be performed on the involved side.

Mass With Unknown Primary


Metastatic adenocarcinoma in a cervical lymph
node with no known primary tumor:
Most common primary sites in the head and neck are the
salivary glands and the thyroid gland
Possibility of an isolated metastasis from the breast, the GI
tract, or the genitourinary tract need to be considered
If no primary site is identified, the patient should be
considered for protocol-based chemotherapy and
radiation therapy, directed according to what the primary
site is most likely to be in that patient

Question 1
What phase of the cell cycle determines the
length of the cell cycle?
A.
B.
C.
D.

M
G1
G0
S

Answer 1
B. The G1 phase determines how long the cell
cycle will take

Question 2
Which of the following correctly describes the
Cori cycle?
A.
B.
C.
D.

The hepatic process of glycogenolysis


The metabolism of glucose by the heart
The utilization of ketones by the brain
The hepatic conversion of lactate into glucose

Answer 2
D. The Cori cycle describes the hepatic
conversion of lactate into glucose

Question 3
Which is NOT an initial response to vascular
injury?
A.
B.
C.
D.

Vasoconstriction
Platelet adhesion
Thrombin generation
Vascular dilation

Answer 3
D. Vascular dilation

Question 4
Which of the following factors is degraded by
plasmin?
A.
B.
C.
D.
E.

Factor V
Factor VII
Fibrinogen
Von Willenbrand factor
Fibrin

Answer 4
D. Plasmin does not degrade VWF.

Question 5
Which 2 factors do DDDAVP and conjugated
estrogens cause the epithelium to release?
A.
B.
C.
D.

Factor II and VWF


Plasmin and Factor VIII
Plasmin and VWF
Factor VIII and vWF

Answer 5
D. Factor VIII and vWF are released from epithelium
in response to DDAVP and estrogens

Question 6
A 38 year old kidney transplant patient is admitted with
fever and chills after he received 6 weeks of IV antibiotic
coverage for aspiration pneumonia. A thick walled cavity in
the chest apex on CT scan with underlying parenchymal
lung destruction is found. There is a crescent shaped
radiolucency within this cavity with a central mass that
shifts when the patient is placed in the prone position on
CT scanning. What is the most likely diagnosis?

A.
B.
C.
D.

Adenocarcinoma of the lung


Metastatic lesion
Tuberculosis
Aspergilloma

Answer 6
D. Aspergilloma
Also known as a "fungus ball
Aspergillosis is an opportunistic infection related to an
underlying lung disorder (bronchiectasis, chronic lung abscess,
tuberculosis or cavitary lung carcinoma)
It infects a chronic lung abscess cavity and invades underlying
lung tissue, hence it is a 'complex' aspergilloma.
Invasive aspergilloma is a life threatening aggressive disease
that affects immunocompromised patients.
CT abnormalities- 'halo sign', air crescent lesions, a cavitary
lesion or consolidation of the lung associated with
pneumothorax.

Question 7
You are performing an axillary dissection and
take only level 1 nodes. Where does your
medial border extend?
A.
B.
C.
D.

The lateral edge of the pectoralis minor muscle


The medial edge of the pectoralis minor muscle
The thoracic inlet
None of the above

Answer 7
A. A level one dissection would extend to the
lateral border of pec minor

Levels of Axilla
Remember the pectoralis minor separates
levels of axilla
I= lateral to pec minor
II=posterior
III= medial to pec minor
This extends to Halstedts ligament/thoracic inlet.

Question 8
A 56 yo female presents with a sore, inflamed appearing right
breast that has puckering concerning for inflammatory breast
cancer. You confirm this on biopsy and her other films are
otherwise negative for metastatic disease. Which of the
following should you next recommend?
A. Neoadjuvant radiation followed by mastectomy
B. MRM with postoperative chemoradiation
C. Neoadjuvant chemo, MRM, then radiation
D. Neoadjuvant chemoradiation only

Answer 8
C. Neoadjuvant chemo, MRM, then radiation

Rx inflammatory breast cancer

Neoadjuvant chemo first


Mastectomy (modified radical)
Postoperatively they should receive XRT

Question 9
A 67 yo male that you performed a Hartmanns procedure 6 weeks ago who
had feculent peritonitis and a prolonged postoperative course is in clinic for
follow up. He notes some malaise and RUQ pain. You order a CT which
shows a hepatic abscess. Which of the following is not true about this
condition?
A. It usually responds to drainage and antibiotics
B. It is most common in patients with previous GI infection
C. E. coli and Klebsiella are common causative organisms
D. Primary therapy should be wedge resection

Answer 9
D. Primary therapy should not be wedge resection

Pyogenic liver abscess

Primary causes are biliary infection


(cholecystitis/cholangitis) or seeding from
portal vein drainage (appendicitis/diverticulitis)
E. coli, klebsiella and strep are most common
organisms
Rx with abx and/or percutaneous drainage and
search for primary source if not obvious

Question 11
A 41 yo male that returned from a 6 month business stay in
Mexico presents with RUQ pain and fever. He otherwise has
no significant PMH. Scan reveals an abscess and cultures show
Entameoba histolytica, Which of the following should you
recommended for treatment?
A. Laparoscopic drainage and ablation of cyst wall
B. Wedge resection
C. IV flagyl and drainage
D. Mebendazole and drainage

Answer 11
C. IV flagyl and drainage

Rx Amebic Liver Abscess


Causative organism is Entamoeba histolytica which
enters liver via portal system from primary GI infection
Often present with fever, RUQ pain and tenderness
Indirect hemagglutination test may be helpful in
diagnosis
Rx with Metronidazole
Surgery or perc drainage reserved for abx failures.

Question 12
A 65 yo female has a new diagnosis of a right colon
adenocarcinoma and 2 hepatic tumors involving segments 1
and 2. She has no other obvious disease and is in good health.
Which of the following should you recommend?
A. Right hemicolectomy and right hepatic lobectomy
B. Right hemicolectomy and RFA of the 2 lesions
C. Neoadjuvant chemoradiation
D. Right hemicolectomy and left hepatic resection

Answer 12
D. Right hemicolectomy and left hepatic resection

Colon Cancer with Hepatic Mets


Remember the anatomy:

Seg 1 = caudate
2-4 = left lobe (2/3 compose left lat segment)
5-8 =right lobe.
For board purposes, unilobar disease is usually
resected.
Segment 1 and 2 would be resected with left hepatic
lobectomy

Question 13
You stop the coumadin and begin workup for Protein C
deficiency. Another anticoagulant is initiated. Which of
the following are correct statements?
A. Protein C is a hepatically made product and it has a relatively
short half life
B. Protein C has a relatively long half life and is produced mostly
in the peripheral tissues
C. Protein C is vitamin K dependent
D. Protein C deficiency is an autosomal-dominant gene disorder
E. A, C, and D
F. B, C, and D

Answer 13
E. Protein C is vitamin K dependent product
that is made in the liver. The deficiency is due
to an autosomal dominant gene disorder.

Question 14
Which of the following is true about Coumadin?
A. Coumadin is a vitamin K antagonist that can produce a
transient hypercoagulable state cue to the coagulation
inhibition normally provided by the short-half life Protein
C
B. You can expect that a patient on anabolic steroids may
be particularly sensitive to coumadin
C. A person taking corticosteroids may be less sensitive to
the effects of coumadin
D. All of the above
E. Only A and B

Answer 14
D. All of the above are correct statements
about coumadin

Protein C
Is a vitamin K dependent inhibitor of the procoagulant
system
Made in liver
Inactivates factor V and VII:C
Inhibitory action is facilitated by protein S
4-5% of people under 45 have protein C deficiency
Protein C deficiency most commonly presents as
unexplained venous thrombosis.
Autosomal dominant genetic disorder due to either a CRM gene deletion or
dysfunction gene

Protein C
Protein C and S have short half-lives
Vitamin K antagonists such as coumadin can
produce a transient hypercoagulable state
This is due to loss of coagulation inhibition normally
provided by protein C

Protein C deficiency causes Warfarin Induced


Skin Necrosis

Warfarin

Anticoagulant effect of coumadin is consistently REDUCED in pts on:

Anticoagulant effect of coumadin is consistently ENHANCED in pts on:

Barbituates
Contraceptives
Corticosteroids
ACTH
Phenylbutazone
Clofibrate
Anabolic Steroids
D-thyroxine
Glucagon
Quinidine
Many antibiotics

Factors other than coumadin that can be evaluated by prothombin time and prolong it:
II, V, VII, X
Fibrinogen
Heparin, warfarin, liver disease

Question 15
Which of the following may have or cause vitamin K
deficiency and lead to impaired coagulation?
A. A newborn baby
B. Someone who has absolutely no vegetables in their diet
C. A person with Crohns disease that has multiple bowel
resection and clinically has short bowel syndrome
D. A cachectic patient with AIDS that has been taking
multiple prophylactic antibiotics for the last 8 months
E. All of the above
F. A, B, and C

Answer 15
E. All of the above may be deficient in vitamin
K

Question 16
Which of the following correctly states the mechanism of
coumadin?
A. It causes anticoagulation by breaking down fully formed factors 2,
7, 9, and 10
B. It affects the gamma carboxylation of the precursor proteins of the
clotting cascade
C. The process is a competitive inhibition and can be overcome by
administering vitamin K
D. The process is a noncompetitive inhibition so replacement of
factors with FFP is the only option to treat a patient with too much
coumadin on board
E. A, B, and C
F. B and C only
G. None of the above

Answer 16
F is correct

Vitamin K
Present in most edible vegetables, particularly green leaves
Produced also in vivo by intestinal bacteria
Serves as component of specialized microsomal enzyme that
affects posttranslational gamma carboxylation of precursor
proteins of the clotting cascade
Factors 2, 7, 9, 10 are vitamin K dependent

80% of ingested vitamin K is absorbed from small bowel into


intestinal lymph
Is a fat-soluble substance
Deficiencies can occur in fat malabsorption
Long-term use of abx may eliminate intestinal bacteria as a source and
promote deficiency when dietary intake is suboptimal

Vitamin K
Warfarin induces hypoprothrombinema and
anticoagulation by affecting the gamma
carboxylation of the precursor protein
This is a competitive inhibition so it can be
overcome by Vit k administration

Newborns tend to lack vitamin K


Inherently low stores at birth
Immature GI flora
Limited dietary intake

Vitamin K
Routine determination of PT should precede
surgical procedures when pts have any significant
risk factors for deficiency
People with less than 70% normal activity should receive
corrective therapy with vitamin K
Vit k deficient states can be differentiated from liver
disease related hypoprothombinemic states by
measuring noncarboxylated prothombin precursors that
accumulate in vitamin deficiency

Question 17
You are performing a Whipple when you encounter a vessel in an
unexpected position that you think may be an aberrent right hepatic artery.
Which of the following is true about this condition?
A. It is a right hepatic artery that arises from the celiac and travels inferiorly
towards the porta near the head of the pancreas.
B. It arises from the SMA and passes posterior to the portal vein
C. Ligation of this vessel will cause hepatic ischemia that will require a right
hepatic resection
D. It arises directly off of the aorta

Answer 17
B. It arises from the SMA and passes posterior to the portal
vein

Anatomy Aberrent RHA


Replaced right hepatic arises from the SMA and
passes posterior to the portal vein
Replaced left hepatic arises directly from the
celiac or from the left gastric and passes directly
into the liver
Either can be ligated if needed without
significant issue relative to hepatic ischemia

Question 18
A 28 yo female who has been on OCPs for 5
years is found to have an asymptomatic hepatic
adenoma. Which of the following is the most
likely complication from this lesion
A.
B.
C.
D.

Malignant transformation
Rupture and hemorrhage
Infection
Portal vein thrombosis

Answer 18
B. Rupture and hemorrhage

Hepatic Adenoma

Benign neoplasm.
Most common in young women.
Assoc with OCP use.
Primary complication is rupture and hemorrhage.
If diagnosed and small, manage by stopping OCP use
If not a/w with OCP use, is large, or symptomatic,
resection is indicated

Question 19
A 96 yo man presents with rectal bleeding and is
found to have a 2 cm villous rectal polyp. Which of
the following would be a contraindication to transanal
excision?

A. 25% circumference of the rectum


B. 6 cm away from anal verge
C. 2.1 cm in size
D. Lymphovascular invasion

Answer 19
D. Lymphovascular invasion

Villous Adenoma in Elderly Patient


This is best treated with transanal excision
Criteria for transanal excision:

Must be polyp or no greater than T1 tumor


Must be <40% circumference of rectum
Must be within 8-10cm of anal verge
If invasive, must not have lymphovascular invasion
(LVI) and must not be poorly differentiated (these
are risk factors for LN mets)

Question 20
A 67 yo female is found to have a 2 cm mass in
the anal canal. Biopsy shows SCC pathology.
Which of the following should you
recommend?
A.
B.
C.
D.

WLE
APR
Mitomycin , 5-FU, and XRT
Fulgaration and podoxyflin

Answer 21
C. Mitomycin , 5-FU, and XRT

SCC of Anal Canal


Chemo (5-FU/Mitomycin) + Radiation is the
standard treatment
Surgery reserved for failures of chemo/XRT

Question 22
A 62 yo female undergoes FNA for a thyroid
mass. This reveals a follicular cell neoplasm so
you perform a lobectomy. Final pathology shows
Hurthle Cell neoplasm. Which of the following is
the next best management plan?
A.
B.
C.
D.

Thyroid irridation
Completion thyroidectomy
Observation
Completion thyroidectomy and LND

Answer 22
C. Observation

Hurthle Cell Neoplasm


Tx of Hurthle cell tumors of the thyroid is the same
as for follicular neoplasms
They are difficult to diagnose by FNA- may appear
as follicular neoplasm on FNA
Like FNA, frozen section also difficult to distinguish
carcinoma
Surgery is always a lobectomy as only ~20%, of
these will be carcinoma, just like follicular
If carcinoma identified on permanent section, then
completion lobectomy is indicated

Question 23
2 weeks s/p Whipple your pt has early satiety
with po intake. You start metoclopramide and
erythromycin. Which receptor does
erythromycin bind to increase gastrointestinal
motility?
A.
B.
C.
D.

Somatostatin receptor
Acetylcholine and dopaminergic receptors
GABA receptor
Motilin receptor

Answer 23
D. Erythromycin binds to the motilin receptor.
The motilin receptor is found primarily in the
stomach, duodenum, and colon.
Metoclopramide improves motility by
stimulating acetylcholine release and blocking
dopaminergic receptors.

Question 24
A 32 yo female with MEN-1 has both parathyroid hyperplasia
with hyperparathyroidism as well as a 1.5 cm gastrinoma. She
has no complaints of reflux or history of PUD. Which of the
following should you recommend?
A. Gastrectomy then parthyroidectomy
B. Resection of the gastrinoma then parathyroidectomy
C. Parathyroidectomy with autotransplantation
D. Medial management only

Answer 24
C. Parathyroidectomy with autotransplantation

Sequence of Therapy in MEN 1


Resect the parathyroids first then do
gastrinoma.
School of thought is that once you correct the
hypercalcemia, it makes control of acid
hypersecretion easier and makes rx gastrinoma
purely elective.

Question 25
A 50 yo undergoes total thyroidectomy for a large
goiter. Postoperatively, her voice somewhat hoarse
and it is slightly worse when she sees you in clinic 1
week later. You suspect a RLN injury. Which of the
following should you recommend?
A.
B.
C.
D.

Re-exploration and repair of the nerve


Radiotherapy
ENT consult with injection of the vocal cords
High dose steroids

Answer 25
C. ENT consult with injection of the vocal cords

RLN injury
If recognized immediately, then repair nerve
primarily.
Recognized postop, then goal should be
medialization of the cords, usually by
injection directly into cords
This can allow improved phonation and optimized
glottic closure to prevent aspiration.

Question 26
A 75 yo man presents with a painless, small mass overlying his
left parotid. You obtain a biopsy which reveals cells suspicious
for pleomorphic adenoma. Which of the following is true
about this condition?
A. It is the second most common benign tumor of the parotid
B. There is a relatively high transformation rate into malignant disease
so total parotidectomy with facial nerve reconstruction is
recommended
C. Though the tumor itself is benign, there is a slight chance it may
undergo malignant transformation so superficial parotidectomy is
recommended
D. The histologic appearance tends to be very homogeneous

Answer 26
C. Superficial parotidectomy is the
recommended treatment

Parotid Gland Tumors- Benign


70% are benign; 70% of all salivary tumors originate in parotid
Pleomorphic adenoma- most common (70%); mixed tumor with
epithelial and myoepithelial elements
8.5% chance of ultimately becoming malignant; fast growth suggestive of
malignancy
Superficial parotidectomy with facial nerve preservation usually suffice

Warthin tumor- 2nd most common; papillary cystadenoma


lymphomatosum
Older pts, smokers, traditionally thought to have male predilection (now
questionable)
Slow growing, cystic, well-demarcated, very low malignant transformation
Low recurrence after resection

Parotid Tumors- Benign


Less common- adenomas and oncocytomas
Treatment- superficial parotidectomy with
sparing of the facial nerve
Pain is due to nerve infiltration/ fixation->
suspect malignancy in pts who present with
pain

Question 27
A 51 yo man presents with a firm mass in the left
parotid. Biopsy is obtained which returns as suspicious
for acinic cell carcinoma. Which of the following is
true?
A. This is a highly malignant tumor with 5-year disease free survival less than
10%
B. This is considered a low grade tumor and when treated with complete
resection has a good prognosis
C. Neoadjuvant chemoradiation should be administered first with surgery being
reserved for nonresponders

D. This tends to present at a slightly older age than other salivary tumors so 51
is unusual

Answer 27
B. Acinic cell carcinoma is low-grade and has
~85% 5 year survival with resection.

Question 28
A 75 year old man who underwent a total
parotidectomy with wide resection for a malignant
tumor 1 year ago presents with a new mass in his lung
that is suspicious for metastatic disease. You review the
pathology and operative report from his record and it
states that there was perineural infiltration and skip
lesions along the facial nerve. Which of the following
tumors is this most likely to be?

A.
B.
C.
D.

Acinic cell carcinoma


Warthins tumor
Adenoid cystic carcinoma
Mucoepidermoid carcinoma

Answer 28
C. Perineural invasion is most consistent with
adenoid cystic carcinoma

Parotid Gland Tumors- Malignant


Includes primary SCC, mucoepidermoid
carcinoma, acinic cell carcinoma,
adenocarcinoma, adenoid cystic carcinoma,
carcinoma arising from pleomorphic adenoma,
and malignant mixed tumor
Low grade- Acinic cell, low-grade
mucoepidermoid, adenoid-cystic
High-grade- adenocarcinoma, squamous cell,
high grade mucoepidermoid

Malignant Parotid Tumors


Treatment- Usually involves excision of all involved
tissues
Complete parotidectomy
Can also include facial nerve, mandible, skin, external
auditory canal, and neck dissection of lesion is high grade
or nodes are involved

Acinic Cell

6% of salivary gland tumors; 83% are in parotid


Mean age- 44
Usually do not invade nerves and metastasize late
5-yr survival ~ 85%

Parotid Tumors- Malignant


Adenoid cystic carcinoma Aggressive
Exhibits high incidence of perineural spread with skip
metastasis along the facial nerve
Incidence of local recurrence high
Postoperative radiation therapy should be provided to
patients who are at high risk for relapse (close or positive
margins, perineural invasion
Cervical LN mets low though distant mets found in 38%
Long term f/u necessary

Parotid Tumors
Mucoepidermoid carcinoma Most common malignant tumor of the parotid- 1/3 of parotid cancers
Subclassified into low-, intermediate-, and high-grade tumors
High-grade tumors-

Selective cervical node dissection & postoperative radiation therapy


often required
5-year survival rate less than 50% with multimodality tx
Low-grade tumors-

More circumscribed; more mucinous cells


Surgical therapy without radiation yields a 5-year survival rate of 75%.

Question 29
You perform a total parotidectomy on this patient from which he
seems to progress from well initially. At a 3 month follow up, he
states that he experiences redness and sweating over his cheek and
near his ear when he eats or even thinks about food. Which of the
following is most likely to be true?
A. You should obtain a CT of his head and neck to evaluate for tumor
recurrence
B. It is caused by transection of the facial nerve
C. If the symptoms easily replicate by having him eat a lemon slice, you can
consider Botox injection to see if that relieves the symptoms
D. It is most likely secondary to injury and inappropriate regeneration of
the auriculotemporal branch of the trigeminal nerve
E. C and D only
F. All of the above

Answer 29
E. C and D are correct

Question 30
A 50 yo male that underwent a Billroth II gastric resection for
gastric outlet obstruction 1 month ago presents for follow-up.
He states that he experiences nausea, occasional nonbilious
emesis, and dizziness about 2 hours after he eats. Which of
the following is most likely?
A. If you were to check his glucose when the symptoms occur, it
would most likely be high
B. This usually does not improve with conservative treatment so you
should anticipate converting him to a Roux-en-Y
C. Octreotide may be helpful
D. This is most likely due to retained antrum so you will need to
consider repeat resection

Answer 30
C. This describes late dumping syndrome and
may respond to octreotide

Dumping Syndrome
Gastrointestinal and vasomotor symptoms
Thought to be due to unmetered entry of ingested food
into SB
May be seen after vagotomy & resection or division of
pyloric sphincter
Early sx- immediately after meal from hyperosmotic
load into the small bowel
Nausea, epigastric discomfort, borborygmi, palpitations, dizziness

Late- 1-3 hrs after meal


Reactive hypoglycemia in addition to sx of early dumping

Dumping Syndrome
Usually responsive to dietary modifications (small, lowfat diet, limit liquids with meals)
Octreotide- can improve symptoms
50-100 mg before meal
Inhibit release of vasoactive peptides from gut
Decrease peak insulin levels and slow intestinal transit
Prevents changes in pulse, systolic BP, and RBC volume during early dumping and
blood glc levels during late dumping

Question 31
A 50 yo male that underwent a subtotal gastrectomy with BII
reconstruction for a T2 adenocarcinoma 1 month ago presents for
follow-up. He experiences frequent epigastric pain after meals with
subsequent bilious vomiting that does not relieve the pain. He has
lost about 15 pounds though otherwise has been stable. Which of
the following is false?
A. He will likely respond well with dietary modification and bile chelators
B. This usually does not improve with conservative treatment so you should
anticipate converting him to a Roux-en-Y
C. This is most likely due to retained antrum so you will need to consider
repeat resection
D. All of the above
E. A and C only

Answer 31
B. This is consistent with alkaline reflux which
is less likely to respond to conservative
treatment and will ultimately need conversion
to a Roux-en-Y

Alkaline Reflux Gastritis


Postprandial epigastric pain, nausea, vomiting of bile,
evidence of bile reflux into stomach with gastritis
Persist in 1-2% of patients who have had vagotomy/
drainage or resection
Differential dx- recurrent ulceration, biliary or pancreatic
disease, afferent loop obstruction, esophagitis
Gastric acid analysis- basal hypochlorhydria with little
increase w/ pentagastrin administration
Obtain serum gastrum measurements to r/o ZE syndrome
and retained antrum

Alkaline Reflux Gastritis


Endoscopy- red, friable mucosa, edema, glandular
atrophy, intestinal metaplasia
Antacids, H2 lockers, bile acid chelators, dietary
manipulation not definitively beneficial
Reoperation to divert contents from gastric mucosa
Convert to Roux-en-Y GJ with 50-60 cm intestinal limb
Very effective in eliminating bilious vomiting
20-30% still have pain or persistent gastric emptying
dysfunction

Question 32
You perform a subtotal gastrectomy with B2 reconstruction on
a 62 yo man with an antral GIST tumor. On postoperative day
1 he is experiencing severe abdominal pain and nonbilious
emesis. You obtain imaging which is concerning for a
markedly dilated bowel limb proximal to the anastomosis.
Which of the following is true?
A. This is most likely secondary to anastomtic edema so you should
place an NGT and start octreotride
B. You would be less likely to have this complication if you did an
antecolic gastrojejunostomy
C. This requires urgent operative intervention
D. In the chronic form (from partial obstruction) patients may develop
microcytic anemia
E. B, C, and D

Answer 33
C. This picture is concerning for afferent limb
obstruction- the patient needs intervention as
he is high risk for duodenal stump blowout.

Afferent Limb Syndrome


Partial or complete obstruction of the afferent limb
proximal to the GJ anastomosis
Afferent limb- duodenal remnant and segment of
jejunum proximal to gastrojejunostomy
Jejunal limb is subject to adhesions, herniaton,
obstruction, volvulus
Rare (~1%) after Billroth II
Can occur immediately or remotely
Acute- operative emergency; secretions remained trapped in limb so vomiting is
nonbilious
Chronic- from partial obstruction- relief of postprandial discomfort with
projectile bilious vomiting

Afferent Limb Syndrome


One of main causes of duodenal stump blowout
Prolonged stasis in chronic disease can lead to bacterial
deconjugation of bile salts-> steatorrhea, malnutrition, Vit
B12 deficiency-> megaloblastic anemia
Pts with increased risk of developing syndrome
Jejunal portion of afferent limb longer than 30 cm
Antecolic GJ
Improper closure of mesocolic defects

Surgical intervention usually necessary


Some attempting endoscopic therapy, though not routine

Question 34
During the workup for epigastric discomfort that began about 6 months
ago, you find a 4 cm mass in the mid-body of the stomach. Biopsy is
consistent with low-grade GIST. He otherwise is healthy with no prior
abdominal operations. Which of the following is the next best step in
management?
A. Preoperative multiagent chemotherapy and radiation followed by resection 6
weeks after treatment

B. Wedge resection of the gastric lesion


C. Subtotal gastrectomy with Billroth 2 reconstruction
D. Treatment with Gleevec

Answer 34
B. Wedge resection is most appropriate

Characteristics/ Management of GIST


GISTs are malignant soft tissue tumors in sarcoma
family
Characterized by mutation in c-KIT (95%) and in
platelet-derived growth factor (PDGF)
Gleevec- targeted inhibitor aimed at c-KIT
They dont go to nodes since they are in sarcoma
family
Usual spread is to liver, lungs and peritoneum

Characteristics/ Management of GIST


Treatment- primarily surgical primarily
Do not need huge margins (wedge resections acceptable)
Mesenteric resection can be limited since they dont usually spread to nodes

Risk factors for recurrence are size >5cm and grade (>5 mitoses/high power
field).
Risk categories categorized into:
Low (<5cm and low grade)
Intermediate (<5cm/high grade or 5- 10cm/low grade)
High (>5cm/high grade or >10cm)

Characteristics/ Management of GIST


Intermediate and high risk tumors receive
adjuvant Gleevec for 1 year after resection
Low risk get surgery only.
Trend is toward organ preservation- in cases
involving multiple organs, Gleevec is given to
shrink tumor (>90% will respond) and later
operate with intent to preserve as much as
possible

Question 1 (of 3)
The primary nutrition source for colonocytes
is:
A.
B.
C.
D.

Glutamine
Short chain fatty acids
Glucose
Alpha-ketoglutarate

Question 2 (of 3)
The primary fuel source for small bowel is:
A.
B.
C.
D.

Glutamine
Short chain fatty acids
Glucose
Alpha-ketoglutarate

Question 3
The primary fuel source for cancer cells is:
A.
B.
C.
D.

Glutamine
Short chain fatty acids
Glucose
Alpha-ketoglutarate

Answer 1-3
1. B; 2. A; 3. A
The primary fuel source for colon cells are short
chain fatty acids. Occasionally patients with a
Hartmanns pouch can get a disuse proctitis that can
be treated with SCFAs.
The primary source of nutrition for small bowel cells
is glutamine. It is thought that tube feeds with
glutamine may help decrease gut translocation in
patients with sepsis, trauma, etc.
Glutamine also is the primary fuel source for cancer
cells.

Question 4
During prolonged starvation the brain switches
from using glucose to using:
A.
B.
C.
D.

Glutamine
Short chain fatty acids
Arginine
Ketones

Answer 4
D. The brain switched from glucose to ketones
after prolonged starvation

Question 5
The p53 tumor suppresor gene is primarily
involved in
A.
B.
C.
D.

Cell cycle regulation


Cell adhesion
Angiogenesis
Chemotaxis

Answer 5
A. P53 is primarily involved in cell cycle
regulation and apoptosis

Question 6
Hyperacute rejection following organ
transplantation is most often due to:
A.
B.
C.
D.

ABO incompatibility
Rh incompatibility
Previously sensitized T cells
Macrophages

Answer 6
A. Hyperacute rejection is most often due to
ABO incompatibility and involves pre-formed
antibodies of the recipient to donor antigens

Question 7
Hyperacute rejection is an example of
hypersensitivity reaction:
A.
B.
C.
D.

Type 1
Type 2
Type 3
Type 4

Answer 7
B. Hyperacute rejection is an example of a
Type II hypersensitivity reaction

Question 8
New proteinuria in a pt following kidney
transplant is most consistent with:
A.
B.
C.
D.

Acute rejection
UTI
Chronic rejection
Renal vein thrombosis

Answer 8
D. New proteinuria is most consistent with
renal vein thrombosis

Question 9
The maximum amount of collagen in a wound
occurs at:
A.
B.
C.
D.

1 week
3 weeks
8 weeks
3 months

Answer 9
B. Maximum collagen accumulation occurs at
2-3 weeks. After that the amount of collagen
stays the same but continued cross-linking
improves strength

Question 10
Diaphragm injuries are:
A.
B.
C.
D.

Most common on the right


Most common on the left
Occur most commonly with penetrating injury
Are easy to find with CT

Answer 10
B. Diaphragm injuries are more common with
blunt trauma and on the left as the liver
protects the right side.
Diaphragm injuries can be hard to find on CT
unless there is gross herniation

Question 11
A 22 yo man suffers a GSW to the left flank. He
is hypotensive but has a pulse. You start to
bolus fluid. The next step should be:
A.
B.
C.
D.

Angiography
CT scan
Left thoracotomy
Laparotomy

Answer 11
D. Laparotomy. This patient is hypotensive
with a penetrating wound to the abdomen. He
needs to go straight to the OR.

Question 12
You perform a laparotomy in the previous pt
and find a large left sided retroperitoneal
hematoma that extends above the celiac
artery. The patient is hypotensive after 6 units
of blood. The next step should be:
A.
B.
C.
D.

Mattox manuever
Cattell manuever
Pelvic embolization
Infra-diaphragmatic control of the aorta

Answer 12
D. Because the hematoma extends above the celiac artery
a Mattox maneuver is not going to give you proper
exposure.
Either perform a left thoracotomy and clamp the aorta or
place an infradiaphragmatic clamp across the aorta until
you get control of the situation.
* Mattox maneuver- retroperitoneal mobilization and
medial rotation of all the left-sided viscera to expose
retroperitoneal vessels
* Cattell- take down cecum though hepatic flexure @
white line then mobilize bowel medially to expose
retroperitoneal vasculature

Question 13
You clamp the infradiaphragmatic aorta , perform a
Mattox maneuver, and find the left renal vein is
completely avulsed from the IVC. The next step
should be:
A. Replace the infradiaphragmatic clamp with a
suprarenal clamp, oversew the renal vein, and ligate IVC
connection
B. Replace the infradiaphragmatic clamp with a
suprarenal clamp, perform primary repair
C. Left nephrectomy
D. Pack and go to ICU

Answer 13
A. The left renal vein has the adrenal vein and
gonadal vein collaterals making ligation of the
left renal vein safe.
The right renal vein does not have these
collaterals so in this case you would have to
perform a nephrectomy if the pt was unstable
or a re-anastomosis.

Question 14
All of the following are true about aldosterone
except:
A.
B.
C.
D.

Causes resorption of Na
Causes excretion of H and K ions
Acts at the distal convoluted tubule
Primarily acts on protein kinase C

Answer 14
D. Aldosterone causes resorption of Na and
excretion of H and K at the DCT by stimulating
the transcription of Na/K ATPase and NA/H
ATPase. This effectively causes more water
reabsorption

Question 15
All of the following are true about nitric oxide
except:
A. Increases cAMP
. The precursor is arginine
C. Is primarily released from vascular endothelium
D. The receptor is guanylate cyclase

Answer 15
A. Arginine is the precursor to nitric oxide. NO
acts on guanylate cyclase to increase cGMP
and cause vasodilation

Question 16
A 10 yo boy has a cyst in his lateral neck medial
to the anterior border of the SCM muscle.
This cyst most likely connects to the:
A.
B.
C.
D.

External auditory canal


The tonsillar pillar
The nasal septum
Thoracic duct

Answer 16
B. Type II branchial cleft cysts extend from the
anterior border of the SCM through the carotid
bifurcation, to the tonsillar pillar

Question 17
The most common branchial cleft cyst is:
A.
B.
C.
D.

Type 1
Type 2
Type 3
Type 4

Answer 17
B. Type II is the most common type of
branchial cleft cyst

Question 18
Treatment of branchial cleft cysts involves
A.
B.
C.
D.

Antibiotics
Resection
XRT
Chemotherapy

Answer 18
B. Treatment of branchial cleft cysts involves
resection

Question 19
The blood supply to the parathyroid glands is:
A. Superior thyroid artery for both glands
B. Inferior thyroid artery for both glands
C. Superior a. for the superior glands; inferior a. for
the inferior glands
D. Transverse cervical artery for both glands

Answer 19
B. The blood supply to the superior and
inferior parathyroid glands is the inferior
thyroid arteries.

Question 20
A 55 yo man undergoes a right pneumonectomy for
lung cancer. 6 days postop he develops fever,
serosanguinous sputum, and the air-fluid level in
the post-pneumonectomy space is lower on the
CXR. There is also new infiltrate in the left lower
lobe. The most appropriate step is:

A.
B.
C.
D.

Bronchoscopy
Chest CT
V/Q scan
Antibiotics

Answer 20
A. The combination of serosanguinous sputum
production, change in air-fluid level, and a new
infiltrate is highly suggestive of a bronchopleural fistula.
The new infiltrate is formed from aspiration
into the remaining lung of the residual fluid in
the post-pneumonectomy air space

Answer 20
Bronchopulmonary fistula are more common
after pneumonectomy compared to lobectomy
The best way to diagnose the problem is
bronchoscopy

Question 21
An 18 yo female is referred by her PCP secondary to RLQ
abdominal pain that started 5 days ago. Her temp is 39C, WBC
18, urine BHCG is negative, and GYN exam is negative. A CT A/P
shows a large phlegmon in the RLQ. Which of the following is
the most appropriate treatment?
A. Ex lap via a midline incision with drainage of the abscess and
appendectomy
B. RLQ incision with drainage of the abscess and appendectomy
C. Percutaneous drainage of the abscess with antibiotics; defer
appendectomy for 6-8 weeks after resolution
D. IV antibiotics only
E. Diagnostic laparoscopy with abscess drainage, appendectomy, and
drain placement

Answer 21
C. Small abscesses can be treated with
antibiotics, hydration, and NPO status.
Large phlegmons should be treated with
percutaneous drainage, antibiotics, and
interval appendectomy.
Complex or non-contained abscesses should be
treated surgically.

Question 22
Following a workup for hematuria, a 63 yo man is
found to have a renal cell carcinoma of the right
kidney. The finding least likely to affect this
patients survival adversely is:
A. Tumor thrombus that extends to the junction of the
renal vein and IVC
B. Single metastatic focus in the lung
C. Tumor invasion into the renal vein
D. Metastases to regional lymph nodes
E. Direct extension of the tumor into the duodenum

Answer 22
A. Renal cell carcinoma is poorly responsive to
chemotherapy or radiation
Surgical excision is best hope for cure
In 4-10% of pts, a tumor thrombus extends into the
lumen of the renal vein, and occasionally the IVC in right
sided tumors
5-year survival rates reported between 47-84% is the
tumor thrombus is removed without leaving residual
disease and the vessel wall is uninvolved.

Characteristics of RCC
Tends to have hematogenous mets
Metastatic disease worsens prognosis (very poor)
Some evidence supports resection of single mets (lung
most common) though overall cure rate not affected
Direct extension into Glissons capsule into surrounding
structures has a poorer prognosis- 5 year survival less
than 5%
LN involvement also has a poorer prognosis (5 yr or 530%)

Question 23
A 34 yo man is brought in with a GSW to the back.
He has 5/5 strength un both upper extremities and
in the LLE. There is decreased sensation in the left
leg. The right leg has normal sensation but 2/5
strength. Which of the following conditions
describes this injury?

A.
B.
C.
D.
E.

Central cord syndrome


Anterior cord syndrome
Brown-Sequard syndrome
Cauda equina syndrome
Complete cord syndrome

Answer 23
C. Brown-Sequard
Incomplete spinal cord syndromes have variable
prognosis for recovery
Greater recovery expected in patients in who there is
greater initial sparing of function below the level of
injury
Brown-Sequard is incomplete/ hemitransection of the
cord usually from penetrating trauma
Results in ipsilateral motor and propriorecpetion loss below the level of injury
and contralateral loss of pain and temperature beginning 1-2 levels below the
injury
Has the best prognosis of all incomplete syndromes

Question 24
A 75 yo female with longstanding osteoarthritis and
cervical scoliosis is brought in after a motor vehicle
accident. You note that she has 3/5 weakness in
both arms though her LE strength is 5/5 on both
sides. Her rectal tone is normal. Which of the
following injuries is this most consistent with?

A.
B.
C.
D.

Anterior spinal syndrome


Central cord syndrome
Brown Sequard syndrome
Cauda equina syndrome

Answer 24
B. This is most consistent with central cord
syndrome

Incomplete Spinal Cord Injury Syndromes


Central cord- occurs essentially in the cervical spine
Injury to central portion of cord

Results in sacral sparing (preservation of perianal sensation and tone) and


greater weakness in the UEs with sparing of the motor function in the LEs
Reflects the topographic organization of motor tracts within the spinal cord in
which the upper extremity tracts are located in a more central position within
the cord
Carries the second best prognosis for recovery with the lower extremities often
recovering better function than the upper

Spinal Cord Injury Syndromes


Anterior cord syndrome Results from damage to the anterior spinal artery
Involves damage to the anterior two-third of the cord and sparing of
the posterior columns
Patients have minimal (if any) motor function distally
Pain and temperature sensation is lost as well
Proprioception, deep pressure, and vibratory sensation are preserved
Prognosis for motor recovery is poor

Cauda equina syndrome Involves injury to the lumbosacral nerve roots within the spinal canal
Results in an areflexic bladder, bowel, and lower limbs

Question 25
The resting energy requirements of a 70 kg
man are approximately (kcal/day)
A.
B.
C.
D.
E.

1400-1500
1600-1700
1800-1900
2000-2100
2200-2300

Answer 25
A. The average resting post-absorptive 7- kg man
consumes oxygen at a rate of about 200 ml/mn or
288 L/day. This equals about 1450 kcal/day.
In general, energy needs increase as illness severity
increases
Uncomplicated postop requirements- 1500-1700
Sepsis- 2000-2400
Multitrauma and mechanical ventilation- 2200-2600
kcal/day
Major burn- 2500-3000

Question 26
A 66 yo F with compensated cirrhosis due to
hepatitis C undergoes laparoscopic sigmoid
resection for a diverticular stricture.
Postoperatively her sodium falls from 128 to 110
meQ though she is asymptomatic. Which of the
following is the best initial treatment plan for this
patient?

A.
B.
C.
D.

1 liter of 3% NS given over 12 hours


60 mmol NaPhos IV over 6 hours
Fluid restriction
Spironolactone

Answer 26
C. Fluid restriction is most appropriate in this
scenario

Question 27
Which structure is most often injured when
performing a 4 compartment fasciotomy?
A.
B.
C.
D.

Dorsalis pedis artery


Peroneal nerve
Sural nerve
Achilles tendon

Answer 27
B. The peroneal nerve is the most frequently
injured structure

Question 28
You are placing a central line in the ICU via the right
subclavian vein. After you puncture the vein and reach
for the wire, the patient quickly takes a large breath and
becomes hypotensive. You are concerned he has an air
embolus. Which of the following management plans is
most appropriate?

A.
B.
C.
D.

Continue line placement and get a STAT CXR


Continue line placement and attempt to aspirate the bubble
Place the patient in trendelenberg with his left side down
Place him on 4 L O2 per nasal cannula for 24 hours

Answer 28
C. When air embolism is suspected, place the
patient in Trendelenberg with left side down

Question 29
The defect that causes familial hypercalcemic
hypocalcuria is:
A. Unresponsive receptor in kidney to PTH
B. Production of PTH molecule that doesnt work
C. Defect in distal convoluted tubule that causes
calcium resorption
D. Defect in renal conversion of vitamin D

Answer 29
C. Familial hypercalcemic hypocalcuria is due
to a defect in the distal convoluted tube that
causes calcium resorption

Tertiary Hypoparathyroidism
Renal disease corrected with transplant
PTH still overproduced
3 gland parathyroidectomy or 4 gland
removal with autotransplantation usually cures

Familial Hypercalcemic Hypocalcuria


Increased serum calcium and low urinary calcium
(should be high if hyperparathyroidism)
Defect in receptor of distal convoluted tubule
that causes increased resorption of calcium
Calcium levels usually 9-11; normal PTH, low
urinary calcium
No treatment, no surgical indication

Pseudohyperparathyroidism
Defect in kidney PTH receptor
No response to PTH

Parathyroid Cancer

Rare cause of hypercalcemia


50% 5 year survival
Hypercalcemia causes mortality
Increased calcium, PTH, alk phos
Lung most common site for mets
Wide en bloc resection
50% recur

Multiple Endocrine Neoplasia


APUD cells
Synchronously or metachronously occurring
lesions
Autosomal dominant, 100% penetrance,
variable expression

Question 30
Which of the following is appropriately matched?
A. MEN1- prolactinoma, gastrinoma,
pheochromocytoma
B. MEN II a- pheochromocytoma, parathyroid
hyperplasia, medullary ca thyroid
C. MEN IIb- pheochromocytoma, gastrinoma,
medullary ca thyroid
D. MEN II a- neuroma, medullary thyroid ca,
pheochromocytoma

Answer 30
B correctly describes MEN IIa

MEN-1
Parathyroid hyperplasia- Usually first to
become symptomatic, urinary sx, 4 gland rsxn
with autotransplantation
Pancreatic Islet Cell tumors; gastrinoma #1; 50%
multiple, malignant major morbidity
Pituitary adenoma; most common prolactinoma
Need to correct hyperparathyroidism first

MEN 2a
Parathyroid hyperplasia
Pheochromocytoma; often bilateral, benign
Medullary cancer of thyroid
Nearly all patients, bilateral
Diarrhea common; usually 1st part to be
symptomatic
#1 cause of death

Need to correct pheochromocytoma first

MEN 2b
Pheochromocytoma (bilateral, benign)
Medullary cancer of thyroid
Nearly all patients, diarrhea most common symptom
Bilateral
#1 cause of death

Mucosal neuromas
Marfans habitus, musculoskeletal abnormalities
Need to correct pheochromocytoma first

Question 31
Gene associated with MEN 1?
MEN 2?

Answer 31
MEN 1- MENIN gene
MEN 2- RET protooncogene

Hypercalcemia

Hyperthyroidism, Hyperparathyroidism
Immobilization, thiazide diuretics
Excess vitamin D, milk-alkali syndrome
Granulomatous disease
Malignancy
Hematologic 25%- Lytic bone lesions
Nonhematologic- 75%- Cancers that release PTHrp
(small cell, breast)

Hypercalcemia
Mithramycin- inhibits osteoclasts; significant side
effects
Hypercalcemic crisis
Breast cancer mets to bone release PTHrp->
hypercalcemia is not due to bony destruction
High urinary cAMP (PTH action on kidney)

Hematologic malignancies can cause bony


destruction
High serum Ca and low urine cAMP

Question 32
Which of the following lies anterior to the
scalene anticus muscle?
A.
B.
C.
D.

Subclavian artery
Brachial plexus
Vagus nerve
Subclavian vein

Answer 32
D. Subclavian vein

Question 33
A 40 yo woman undergoes a difficult
choledochojejunostomy secondary to a biliary stricture
from CBD injury after laparoscopic cholecystectomy 6
months ago. You examine her postoperatively and find
her abdomen markedly distended. She is in pain but
does not have peritonitis. Her BP is 85/48 mmHG.
Which of the following is the most likely cause of these
symptoms?

A.
B.
C.
D.

Biliary leak
Acute gastric distention
Missed small bowel injury
Obstructed foley catheter

Answer 33
B. This is most consistent with acute gastric
distention.
Remember, in the post-op period (most often
around 12 hours) abdominal distention with
pain and hypotension is likely due to acute
gastric dilatation and should be treated with
decompression

Question 34
A 25 yo female with RLQ pain undergoes a
diagnostic laparoscopy for suspected appendicitis.
Her appendix appears normal but you note a 4.5
cm cyst on her right ovary that has clear fluid.
Which of the following is the most appropriate
management plan?
A. Remove her appendix, follow the cyst with an
ultrasound in 3 months
B. Aspirate the cyst
C. Right salpingo-oophorectomy
D. Right partial oophorectomy

Answer 34
B. It is important the cyst be aspirated

Question 35
Antibiotic prophylaxis in patients undergoing
exploratory laparotomy for gunshot wounds to
the abdomen has been shown to:
A. Lower the risk of anastomotic leaks
B. Lower midline wound infections
C. Decrease the occurrence of intraabdominal
abscesses
D. Improve survival

Answer 35
B. Prophylactic abx decrease the risk of
midline wound infections

Question 36
Which of the following in the Glasgow Coma
Scale best predicts immediate recovery after
injury?
A.
B.
C.
D.

Ability to spontaneously open eyes


Good verbal communication
Full motor response
None of the above

Answer 36
C. Best motor response is the most predictive
factor for immediate recovery

Question 1
The Line of Sappey is:
A.
B.
C.
D.

2 cm below the umbilicus


In the axilla
Supraclavicular
2 cm above the umbilicus

Answer 1
D. 2 cm above the umbilicus

Line of Sappey
Is the line 2 cm above the umbilicus to L 2-3
Generally, melanoma drains to axillary nodes
above this line or to ilioinguinal nodes below
this line

Question 2
2 weeks s/p Whipple, a 62 yo female develops
fever, chills, and hypotension. Her blood
cultures grow E. coli. What is the key stimulant
involved in this condition?
A. TGF- beta
B. Lipid A
C. IL-2
D. Protein C

Answer 2
B. Lipid A is a part of the LPS complex found in
E. coli sepsis
It is responsible for the release of TNF- alpha
involved in inflammation and sepsis

Question 3
Pyriodostigmine is:
A.
B.
C.
D.

Cholinesterase inhibitor
Succinylcholine inhibitor
Steroid inhibitor
Beta blocker

Answer 3
A. Pyridostigmine is a cholinesterase inhibitor
It increases the amount of ACH and is used to
treat myasthenia gravis

Question 4
Trimethoprim inhibits:
A.
B.
C.
D.

Purine synthesis
Small subunit of ribosome
Large unit of ribosome
DNA gyrase

Answer 4
A. Trimethoprim inhibits purine synthesis by
inhibiting dihydrofolate reductase

Question 5
Cyclosporine binds to:
A.
B.
C.
D.

IL-2 receptor
Cyclophilin
INF-gamma
FK binding protein

Answer 5
B. Cyclophilin
Cyclosporine binds to the cyclophilin protein
and forms a complex that binds to calcineurin.
It inhibits IL-2 transcription and prevents T-cell
activation
Zenepax is an antibody against IL-2
Tacrolimus binds to FK binding protein

Question 6
Oxygen is bound more tightly to hemoglobin
when there is a:
A.
B.
C.
D.

Increase in CO2
Increase in temperature
Decrease in H+ ion concentration
Increase in levels of 2,3 DPG

Answer 6
C. Decrease in H+ ion concentration, or more
alkalotic condition, will cause oxygen to be more
tightly bound to Hgb.

Question 7
The organism found specifically in human bites
is:
A.
B.
C.
D.

Pasteurella multocida
Staph aureus
Eikinella
Strep pyogenes

Answer 7
C. Eikinella is an organism found only in
human bites.
Pasteurella multocida is the most common
agent in cat and dog bites

Question 8
Which of the following is the most common
organism responsible for spontaneous
bacterial peritonitis?
A.
B.
C.
D.

Staph aureus
E. coli
Strep pyogenes
Bacteroides fragilis

Answer 8
B. E. coli is the most common causative
organism of SBP.

Question 9
BRCA 1 is associated with a:
A.
B.
C.
D.

40% lifetime risk of ovarian cancer


30% lifetime risk of ovarian cancer
20% lifetime risk of ovarian cancer
10% lifetime risk of ovarian cancer

Answer 9
BRCA 1 gene together with family history has a
60% lifetime risk of breast cancer as well as a
40% risk of ovarian cancer and 1% risk of male
breast cancer
BRCA 2 is a/w a 10% risk of ovarian cancer and
10% of male breast cancer

Question 10
The brachial plexus is located:
A.
B.
C.
D.

Posterior to the middle scalene muscle


Anterior to the middle scalene muscle
Anterior to the anterior scalene muscle
Posterior to the posterior scalene muscle

Answer 10
A. B. Anterior to the middle scalene muscle

Scalene Anatomy

Question 11
A 72 yo man with a history of TIAs is found to
have 80% stenosis of the left carotid on
ultrasound. The most appropriate next step is:
A.
B.
C.
D.

Repeat ultrasound in 3 months


Initiate plavix and aspirin
Start aspirin only
Recommend carotid endarterectomy

Answer 11
D. Recommend carotid endarterectomy

Patients with symptomatic carotid disease and


stenosis greater than 60% should have CEA
Asymptomatic patients with more than 70%
stenosis should be considered for CEA

Question 12
The left gastric artery is a branch off of the:
A.
B.
C.
D.

Celiac artery
Splenic artery
Common hepatic artery
SMA

Answer 12
A. The celiac trunk

Blood supply of stomach

Question 14
A broncholith, or calcified granuloma, may be
found in which of the following conditions?
A.
B.
C.
D.

Actinomycosis
Coccidiomycosis
Blastomycosis
Histoplasmosis

Answer 14
D. Histoplasmosis is associated with calcified
granulomas on presentation

Question 15
A 67 yo male with a 4 cm mass in the left lower
lung is shown to be adenocarcinoma on biops6y.
Mediastinoscopy also shows paratracheal lymph
node involvement on the left side. The most
appropriate management plan at this time would
be:

A.
B.
C.
D.

Chemoradiation
Left LL resection and mediastinal LN dissection
Left pneumonectomy and mediastinal LND
Left LL resection only

Answer 15
A. Chemoradiation. Being that the patient has
+ paratracheal nodes, the disease is
unresectable.

Question 16
A 79 yo male from a nursing home undergoes a lap
chole and requires increased narcotics to control
his pain. 4 days later he is brought back because of
abdominal distention and worsening pain diffuse.
Films demonstrate a distended colon extending
through the cecum. Which of the following should
be done next?

A.
B.
C.
D.

Colonoscopy
Cecostomy
Serial abdominal exams
Total abdominal colectomy

Answer 16
A. Colonoscopy should be attempted first
given that the patient most likely has Ogilves
associated with recent opiate use. Treatment
should include decompressive cscope,
discontinuing drugs that inhibit intestinal
motility, and checking electrolytes.

Question 17
1 hour after CEA, a 66 yo male is found to be
substantially hoarse. Which of the following is
the most appropriate step in management?
A.
B.
C.
D.

Observation
MRI of head and neck
OR for re-exploration
Angiography

Answer 17
C. The patient should be taken back to the OR
as hoarseness immediately after surgery is
suggestive of a nerve injury

Question 18
A 66 yo male undergoes an Ivor-lewis
esophagectomy for a distal esophageal
adenocarcinoma. POD #1 he becomes febrile,
tachycardic, and has a WBC of 22. You obtain a
contrast study which shows an anastomotic leak.
Which of the following is the next best step?

A.
B.
C.
D.

Ask for an esophageal stent to be placed


Re-explore
Place a chest tube and keep NPO
Place a chest tube and esophageal stent

Answer 18
B. A leak on POD #1 would best be treated by
reexploration

Question 19
A 56 yo female has crampy RUQ and epigastric
abdominal pain that started about 3 days ago.
Films demonstrate multiple air-fluid levels in the
small bowel with distention. You also note
pneumobilia. Which of the following should you do
next?

A.
B.
C.
D.

Start broad spectrum antibiotics


Request percutaneous cholecystotomy tube placement
Order ERCP
Plan for exploratory laparotomy

Answer 19
D. This patient presents with what is most
likely gallstone ileus.
Pneumobilia, crampy abdominal pain, and
bowel obstruction are characteristic
It is most appropriate to explore the patient
and remove the stone

Question 20
A 52 yo male undergoes embolectomy for a clot to
the right lower extremity. 4 hours post-op his leg
becomes swollen and painful. It feels tight to
palpation. Which of the following is the next best
step?
A. Take back to the OR for angiography and possible graft
revision
B. Perform a femoral-tibial bypass
C. Initiate a heparin drip
D. Perform a 4 compartment fasciotomy

Answer 20
D. This patient is having a reperfusion injury
resulting in compartment syndrome so he
needs a fasciotomy

Question 21
A left to right cardiac shunt is seen in which of
the following conditions?
A.
B.
C.
D.

Atrial septal defect


Tetralogy of Fallot
Truncus arteriosus
Tricuspid atresia

Answer 21
A. Atrial septal defect is associated with left to
right shunt and in most cases closes
spontaneously by age 1.
Right to left shunt is associated with :
TOF
Truncus arteriosus
Tricuspid atresia

Question 22
During a laparoscopic cholecystectomy you
perform a cholangiogram and believe that you
have clipped across the distal hepatic duct.
The next most appropriate step is:
A.
B.
C.
D.

Open end-to-end anastomosis


Choledochojejunostomy
Hepaticojejunostomy
Whipple procedure

Answer 22
C. Hepaticojejunostomy

An end-to-end anastomosis in this setting can


lead to strictures

Question 23
The most common symptom after a pulmonary
embolism is:
A. Cough
B. Hemoptysis
C. Pleural pain
D. Dyspnea

Answer 23
D. Dyspnea is the most common symptom associated
with pulmonary embolism

Question 24
Epiphrenic diverticula are:
A.
B.
C.
D.

Located in the proximal esophagus


Are true diverticula
Commonly are asymptomatic
Are associated with achlasia

Answer 24
D. Epiphrenic diverticula are usually asymptomatic
and located in the distal 1/3 of the esophagus.
They are commonly associated with achlasia

Question 25
Maltose contains which of the following?
A.
B.
C.
D.

Galactose
Fructose
Glucose
Cellulose

Answer 25
C. Maltose contains glucose

Question 26
A 30 yo presents with a BP of 220/110 mmHg
and complaining of headaches. The most
appropriate test to begin his evaluation should
be:
A.
B.
C.
D.

Check a renin level


MIBG scan
24 hour urine VMA and metanephrine
TSH

Answer 26
C. 24 hour urine VMA and metanephrine
This patient may have a pheochromocytoma and the
best test is urine CMA and metanephrine
MIBG is the most sensitive test for localizing a pheo.

Question 27
Cortisol releasing hormone is released from
the :
A Pituitary
B. Thyroid
C. Hypothalamus
D. Adrenal

Answer 27
C. The hypothalamus releases GhRH, GnRH,
CRH, and TRH

Question 28
Which of the following genes is associated with
Von Hippel-lindau disease?
A.
B.
C.
D.

APC
VHL
BRCA 2
P53

Answer 28
B. VHL gene
VHL is a tumor suppressor gene involved in
Von Hippel-Lindau disease.
This disease is a/w pheochromocytoma, renal
angioma, renal cell carcinoma, and
hemangioblastoma

Question 28
The first line of therapy for ITP is:
A.
B.
C.
D.

Gamma globulin
Splenectomy
Platelet transfusion
Steroids

Answer 28
D. Steroids are considered as primary therapy
for ITP

Question 29
Which of the following is true of nitric oxide
(NO)?
A.
B.
C.
D.

It increases cGMP level


It is released from lymphocytes
It is released from platelets
Glutamine is its precursor

Answer 29
A. NO is produced in endothelial cells and its
release causes increase of cGMP levels.
This subsequently leads to vasodilation
Arginine is the precursor to NO

Question 30
Cantrells pentalogy does NOT involve which of
the following?
A.
B.
C.
D.

Pericardial defects
Diaphragmatic defects
Sternal defects
Liver defects

Answer 30
D. Liver defects.
Cantrells pentaology involves omphalocele,
diaphragmatic, sternal, pericardial, and cardiac
defects

Question 31
Regarding hormone receptors and breast
cancer, which of the following is true?
A. Both estrogen (ER) and progesterone (PR)
positive tumors have a worse prognosis
B. ER tumors have a worse prognosis
C. HER tumors have a poorer prognosis
D. Progesterone receptor positive tumors have a
poorer prognosis than PR negative tumors

Answer 31
C. HER receptor tumors have a poorer
prognosis
ER and PR positivity improves prognosis

Question 32
A 49 yo male vomits multiple times after drinking
heavily. Following this he develops severe chest
pain and presents to the ER. You obtain a chest XR
which shows a left sided pleural effusion and PTX.
Which of the following is the most likely location of
the perforation?

A.
B.
C.
D.

Right posterior middle esophagus


Left posterior middle esophagus
Left posterior upper esophagus
Left posterior lower esophagus

Answer 32
D. This patient likely has a Boerhaaves rupture
of the esophagus
The most common location is the posterior left
aspect of the lower esophagus

Question 33
A 65 yo man presents with an anal mass. You
biopsy the lesion and pathology shows that it is
a squamous cell carcinoma. Which of the
following is the next most appropriate step?
A.
B.
C.
D.

Abdominoperineal resection
Low anterior resection
Chemoradiation
Wilde excision

Answer 33
C. Chemoradiation is the best first treatment
for SCC of the anus

Question 34
A 55 yo man with epigastric pain that is not
responsive to PPIs undergoes an EGD. A mass
is noted in the stomach and biopsied. The
biopsy comes back as lymphoproliferative
tissue. The next most appropriate therapy is:
A.
B.
C.
D.

Chemoradiation
Radiation only
Amoxicillin, tetracycline, and PPI therapy
Total gastrectomy

Answer 34
C. The appropriate treatment of MALT
(mucosa associated lymphoproliferative tissue)
is eradication of H. Pylori

Question 35
A 63 yo man presents with abdominal pain in
the RUQ and nausea. The patient is 5 days
post-op from laparoscopic cholecystectomy.
The test you should first perform should be:
A.
B.
C.
D.

Abdominal CT
ERCP
Abdominal MRI
RUQ ultrasound

Answer 35
D. RUQ ultrasound should be done to evaluate
pain s/p lap chole

Question 36
A 50 year old male undergoes a left
pneumonectomy for a malignant mass. On post-op
day #5, he develops fever and serosanguinous
sputum. A chest xray shows air-fluid levels on the
left. The most appropriate step is:

A.
B.
C.
D.

Chest CT
Repeat the chest xray
ECHO
Perform bronchoscopy

Answer 36
D. This patient most likely has a
bronchopleural fistula.
Bronchoscopy is the most important diagnostic
test

Question 37
The MELD score for ESLD does NOT include:
A.
B.
C.
D.

INR
Serum creatnine
Total bilirubin
Albumin

Answer 37
D. The MELD score consists of:
Bilirubin
INR
Creatnine

The maximum score is 40

Question 38
Catecholamines are synthesized from which of
the following substances?
A.
B.
C.
D.

Arginine
Glutamine
Tyrosine
Tryptophan

Answer 38
C. Tyrosine is the substrate for catecholamines

Question 39
Hesselbachs triangle involves all of the
following structures except:
A. The inguinal ligament
B. The lacunar ligament
C. Rectus muscle
D. Inferior epigastric artery

Answer 39
B. Hesselbachs triangle involves the rectus
muscle, inferior epigastric artery, and the
inguinal ligament

Question 40
A 55 yo female received 7 days of ampicillin for a
UTI. Six days after she started taking the
medication she presents with diarrhea which tests
positive for C. dif. Which of the following should
you do next?

A.
B.
C.
D.

Observe the patient


Give 7 days of PO clindamycin
Give PO flagyl for 10-14 days
Admit the patient for hydration and IV gentamycin

Answer 40
C. Treatment of C. dif is flagyl 250-500 mg
orally 4 times a day for 10 days.
Vancomycin is also effective in treating C. dif.

Question 41
A 30 yo male presents with RLQ pain suspicious for
appendicitis. You perform laparoscopy and note
the he has terminal ileitis but the cecum and
appendix appear normal. Which of the following
should you do next?

A.
B.
C.
D.

Close the patient


Perform a right hemicolectomy
Perform a segmental ileal resection
Perform an appendectomy

Answer 41
D. This patient with terminal ileitis that does
not involve the cecum should still undergo
appendectomy.
Only leave the appendix when the cecum is
involved

Question 42
A 24 yo man feels a sudden and sharp pain in his
lower calf after playing basketball. Examination
reveals diffuse ecchymosis over his calf and weak
plantar and dorsiflexion of the ankle. Squeezing
the calf does not produce ankle plantar flexion.
The patient most likely injured the:

A.
B.
C.
D.

Posterior tibial tendon


Achilles tendon
Gastrocnemius
Plantaris tendon

Answer 42
Achilles tendon
The patient most likely has an achilles tendon
injury.
Squeezing the calf muscles and producing no
ankle flexion is characteristic for Achilles
tendon injury- this is called the Thompson test.

Question 43
A 32 yo female presents with an abdominal
bulge slightly to the right of the midline. You
obtain a CT scan which reveals a Spigelian
hernia. Which of the following is true about
this condition?
A.
B.
C.
D.

It very rarely incarcerates


It is through the linea semilunaris
Operative repair is not indicated
It is located inferior to the inguinal ligament

Answer 43
B. It is through the linea semilunaris
Spigelian hernias have a high rate of incarceration
When diagnosed, repair is indicated

Question 44
A preciously healthy 12 yo boy is brought to the ED
for severe testicular pain that started abruptly
about 5 hours ago. He is afebrile, stable, and his UA
is negative. His left testicle is swollen and TTP with
no cremasteric reflex noted. The next step should
be:

A.
B.
B.
D.

Take to the OR for exploration


Admit for pain control and observation
Send him home with anti-inflammatories and ice pack
Perform an ultrasound

Answer 44
A. The findings listed are consistent with
testicular torsion
He should immediately undergo exploration- 6
hours is the window after which the risk of
irreversible damage becomes high
An ultrasound can delay the procedure

Question 45
Indications for hiatal hernia repair include:
A. Regurgitation and aspiration in a patient with a
UGI that demonstrates reflux that is well-controlled
with medical therapy
B. A patient who has required medical therapy for
more than 5 years
C. A patient with low grade Barretts esophagus
D. A patient with high grade Barretts esophagus

Answer 45
Indications for hiatal hernia repair include high
grade Barretts, regurgitation and aspiration
not controlled with medical therapy, and a
patient that does not want to be treated
medically

Question 46
A 66 yo man presents with a 9 month history
of dysphagia to solids and food regurgitation.
He denies any abdominal pain or weight loss.
The best test to start his workup should be:
A.
B.
C.
D.

Upper endoscopy
Barium esophagogram
CT of the chest
Esophageal manometry

Answer 46
B. Barium esophaogram could delineate the
presence of the mass as well as a Zenkers
diverticulum

Question 47
A 65 yo man undergoes a low anterior resection for
rectal cancer. 7 hours post-op he develops a fever
of 104 and increased drainage is noted from the
inferior aspect of his incision. The most
appropriate step in management is:
A. Change the dressing every 4 hours
B. Change the dressing and start broad spectrum
antibiotics
B. Take to the OR for debridement
D. Transfer to the ICU for fluids, IV abx, and monitoring

Answer 47
C. This patient has an early invasive wound
infection and needs to be taken back for
emergent debridement
Clostridium perfringes or beta-hemolytic strep
group A are the usual pathogens
Penicillin is the treatment of choice for both
pathogens

Question 48
A 70 yo female with diabetes and cor pulmonale is
POD # 14 from a laparoscopic colon resection. She
does not have any complaints though you notice
she now has some blisters over her lower
extremities and 3+ pitting edema of both legs. Her
condition is most likely due to:

A.
B.
C.
D.

Beta hemolytic strep infection


Lymphedema
Superficial venous thrombosis
Peripheral vascular disease

Answer 48
B. Lymphedema
Her condition is most likely due to lymphedema from
recent immobilization
Beta hemolytic strep infection is usually unilateral
Superficial vein thrombosis usually develops in
varicose veins and is symptomatic (tender, painful,
erythematous)
Treatment of lymphedema involves diuresis and
compression of lower extremities with ace wraps

Question 49
An 80 yo woman develops dark and purple
lesions on her right arm. 10 years ago she
underwent a mastectomy with ALND. Her
condition is most likely due to :
A.
B.
C.
D.

Melanoma
Sarcoma
Sarcoidosis
Fibromatosis

Answer 49
B. Sarcoma.
This patient most likely has
lymphangiosarcoma from chronic lymphedema
after axillary dissection
Stewart-Treves syndrome

Question 50
Thromboxane is involved in:
A. Increased platelet aggregation and vasodilatation
B. Increased platelet aggregation and
vasoconstriction
C. Decreased platelet aggregation and vasodilatation
D. Decreased platelet aggregation and
vasoconstriction

Answer 50
B. Thromboxane causes platelet aggregation
and vasoconstriction by increasing calcium
within the platelet

Question 51
The first branch of the internal carotid artery
is:
A.
B.
B.
D.

The superior thyroid


Inferior thyroid
The ophthalamic
Vertebral artery

Answer 51
C. The first branch of the INTERNAL carotid is
the ophthalamic artery

Question 52
Hereditary spherocytosis involves defects in:
A.
B.
C.
D.

Fibrin
Spectrin
Collagen
Plasmin

Answer 52
B. Hereditary spherocytosis involves defect in
a spectrin protein

Question 53
A recurrent papillary or follicular thyroid
cancer can be detected with:
A.
B.
C.
D.

Rise in TSH level


Rise in TRH level
Transferrin
Rise in thyroglobulin

Answer 53
D. Serum thyroglobulin is the best test to
detect recurrent papillary or follicular thyroid
cancer.

Question 54
Which of the following is true about lobular
breast cancers?
A. They are often multicentric
B. They have calcifications on MMG
C. They make up about 30% of all breast cancers
D. There is no increased risk for bilateral
involvement

Answer 54
A. Lobular breast cancers are multicentric,
bilateral, and do not have calcifications.
They consist of about 10% of all breast cancers

Question 55
Somatostatinoma can present with all of the
following except:
A.
B.
C.
D.

Diabetes
Necrolytic migratory erythema
Gallstones
Steatorrhea

Answer 55
B. Somatostatinoma can present with
diabetes, gallstones, and steatorrhea.
Glucagonoma can present with necrolytic
migratory erythema

Question 56
Hereditary non-polyposis colon cancer
syndrome is associated with:
A.
B.
C.
D.

Ovarian and breast cancer


Thousands of polyps
Loss of p53 gene
Lung cancer

Answer 56
A. Pts with HNPCC have predisposition to
ovarian, breast, and colon cancer.
HNPCC does not present with thousands of
polyps
It is present:
In 3 primary relatives over 2 generations
In at least one person who was less than 50 yrs old
at the time of diagnosis

Question 57
A 69 yo patient with a history of pancreatitis
presents with severe upper GI bleeding from
gastric varices. On an EGD, you do not see any
esophageal varices. The best treatment would
be:
A.
B.
C.
D.

TIPS
Splenectomy
Splenorenal shunt
Portocaval shunt

Answer 57
B. Bleeding from gastric varices without
esophageal varices is most likely a/w a
thrombosed splenic vein.
Treatment is splenectomy

Question 58
Where would be the most common place to
find an accessory spleen?
A.
B.
C.
D.

The splenic hilum


Aortic bifurcation
Gallbladder fossa
GE junction

Answer 58
A. The splenic hilum would be the most likely
place to find an accessory spleen

Question 59
A 66 yo patient with chronic abdominal pain, a
history of chronic pancreatitis, and a dilated
pancreatic duct may benefit from:
A.
B.
C.
D.

A Whipple procedure
A lateral pancreaticojejunosotmy
Billroth 2 procedure
Percutaneous drainage

Answer 59
B. Patients with chronic pancreatitis and
chronic abdominal pain may benefit from the
Peustow procedure

Question 60
A 68 yo male is 5 days s/p LAR when he develops
fever, chills, and a WBC of 19,000 with diffuse
abdominal tenderness and feculent brown drainage
of ~ 1200 cc/day from his wound. Which of the
following is the next best step?

A.
B.
C.
D.

OR for takedown of the anastomosis


Percutaneous drainage
Re-exploration and repair of the anastomosis
APR

Answer 60
A. The next best step would be to re-explore,
being it is only POD #5. The best bet is to
perform a Hartmanns and colostomy

Question 61
A 77yo nursing home pt presents with
abdominal pain and films reveal a sigmoid
volvulus. Her vitals are stable and she does not
have peritonitis. The next best step is:
A.
B.
C.
D.

Perform serial exams


Perform colonoscopy
LAR
APR

Answer 61
B. Colonoscopy can usually decompress
patients with sigmoid volvulus. This is an
appropriate initial step in a stable patient; a
rectal tube should be left in place.

Question 62
A 33 yo man with a supracondylar humeral
fracture has his arm reduced and a sling is
placed. 2 hours later he develops severe
forearm pain and swelling. You cannot detect
distal pulses. The most appropriate step is to:
A.
B.
C.
D.

Open reduction and internal fixation


Fasciotomy
Angiogram with stent placement
Observation and ice packs

Answer 62
B. This patient has a compartment syndrome
so fasciotomies are most appropriate

Question 63
A 55 yo alcoholic presents to the ER after a large
episode of emesis and now has severe substernal
pain. The pain started about 8 hours ago.
Gastrograffin study shows perforation of the distal
esophagus with drainage into the mediastinum.
Which of the following is the most appropriate
treatment?

A.
B.
C.
D.

Esophagectomy
Emergent primary repair
Esophagectomy and placement of a FCJ
Nonoperative mgmt with NNGT and TPN

Answer 63
B. Primary repair of esophageal rupture is
acceptable within the first 24 hours- the
survival rate is around 90%
A left thoracotomy is performed for ruptures of
the distal esophagus
In highly unstable patients, diversion with an
esophagostomy may be safest.

Question 64
Charcots Triad includes all of the following
except:
A.
B.
C.
D.

RUQ pain
Tremor
Fever
Jaundice

Answer 64
B. Tremor is not a part of Charcots triad

Question 65
A 78 yo patient following a right
hemicolectomy for stage III adenocarcinoma
should have the following?
A.
B.
C.
D.

Chemotherapy and radiation


Radiation only
Q 3 labs, CEA, and imaging
Chemotherapy and follow up

Answer 65
D. Stage 3 colon cancer should have
chemotherapy

Question 66
After esophagectomy, the primary blood
supply to the stomach is:
A.
B.
C.
D.

Gastroduodenal artery
Right gastroepiploic artery
Left gastric artery
Left gastroepiploic artery

Answer 66
B. The right GE artery is what provides the
gastric conduit

Question 67
Coarctaction of the aorta is?
A.
B.
C.
D.

Proximal to the left subclavian artery


Distal to the left subclavian artery
Is a/w Downs syndrome
Is located in ascending aorta

Answer 67
B. Distal to the left subclavian artery
Coarctation of the aorta, or aortic coarctation,
is a congenital condition whereby the aorta
narrows in the area where the ducts arteriosus
(ligamentum arteriosum after regression)
inserts

Question 68
A newborn fails to pass meconium during the
first 24 hours and develops abdominal
distention. An x-ray of the abdomen shows
dilated colon. The most appropriate next step
is:
A.
B.
C.
D.

Perform a BE
Observe and administer stool softeners
Order an upper GI with SBFT
Perform a rectal biopsy

Answer 68
D. This patient may have Hirschsprungs,
which is characterized by an absence of
ganglion cells in the myenteric plexus
The best diagnostic test is a rectal biopsy

Question 69
The most common blood transfusion reaction
is:
A.
B.
C.
D.

Transfusion related acute lung injury


Febrile nonhemolytic transfusion reaction
Allergic reaction
Acute hemolytic reaction

Answer 69
B. Febrile nonhemolytic transfusion reaction
is the most common

Question 70
A 45 o develops dizziness and decreased
respiratory rate after he is given IV morphine.
Which drug is most effective in treating this
condition?
A.
B.
D.
D.

Edrophonium
Flumazenil
Neostigmine
Narcan

Answer 70
D. Overdose of narcan can present with slow
breathing, dizziness, and pinpoint pupils.
Narcan is the best drug to treat narcotic
overdose

Question 71
A 6 yo boy presents with a midline neck mass
that is moving when he swallows. You should
recommend:
A. Resection of the thyroglossal duct cyst
B. Resection on the cyst and LND
C. Removal of the mass and resection of the hyoid
bone
D. Removal of the mass and total thyroidectomy

Answer 71
C. Removal of the mass and resection of the
hyoid bone is the appropriate treatment for
thyroglossal duct cysts
This is called the Sistrunk procedure

Question 72
Gain of function of the following gene is
implicated in the development of colon cancer:
A.
B.
C.
D.

P53
K-ras
DCC
APC

Answer 72
B. K-ras
Loss of function of p53, DCC, and APC is
implicated in colorectal cancer development
Gain of function of pro-oncogene k-ras is
involved in the development of colon cancer

Question 73
A 25 yo male is s/w GSW to the chest and
abdomen. He is in the intensive care unit and
has required multiple units of blood products.
You would expect increased levels of all of the
following hormones EXCEPT in this patient?
A. ADH
B. TSH
C. Growth hormone
D. ACTH

Answer 73
B. TSH. Tissue injury is associated with
stimulation of ADH, growth hormone, and
ACTH production

Question 74
The most common organism associated with
ventilator associated pneumonia is:
A.
B.
C.
D.

Pseudomonas
S. aureus
E. coli
Strep pneumonia

Answer 74
B. Staph aureus is the most common organism
a/w VAP

Question 75
A 67 yo patient with hyperkalemia and new
onset of arrhythmia should first receive:
A.
B.
C.
D.

Kaexylate
Calcium gluconate
Insulin
Insulin and Glucose

Answer 75
B. Calcium gluconate is the first choice in a
patient with hyperkalemia and arrhytmias to
stabilize cell membranes

Question 76
The cricothyroid muscle is innervated by:
A.
B.
C.
D.

Superior laryngeal nerve


Recurrent laryngeal nerve
Glossopharyngeal nerve
Vagus nerve

Answer 76
A. The cricothyroid is innervated by the SLN

Question 77
Contraindications to lumpectomy and radiation
are all of the following EXCEPT:
A.
B.
C.
D.

Previous radiotherapy
A large tumor
Multiple lesions
Pregnancy

Answer 77
B. A large tumor.
Contraindications to lumpectomy and XRTpregnancy, multiple lesions, and previous
radiation

Question 78
A 66 yo male with lung cancer presents with
swollen face and arms. These symptoms are
most likely due to:
A.
B.
C.
D.

SVC invasion
Invasion of the brachial plexus
Invasion of the sympathetic nerves
Paraneoplastic syndrome

Answer 78
A. This is most likely due to tumor invasion
into the SVC
Treatment of SVC invasion due to cancer is
radiation

Question 1
Which of the following statements about
primary sclerosis cholangitis is correct?
A. Is frequently associated with high fever and chills
B. Course is usually rapidly progressive to liver
failure
C. Can usually be managed successfully by antiinflammatories and corticosteroids
D. Is commonly associated with inflammatory bowel
disease

Answer 1
D. PSC is associated w/ IBD.
Review of PSC:
Most commonly affects men in 4-5th decade
A/w retroperitoneal fibrosis, Riedels thyroiditis,
pancreatitis, inflammatory bowel (more so in UC),
and DM
Sx- fatigue, fluctuating jaundice, pruritis, weight
loss, RUQ pain
ERCP shows multiple dilations and strictures
(beaded appearance)

Answer 1
Pts have antimitochondrial antibodies
Bacterial cholangitis unusual unless biliary tract has
been manipulated
Does not get better after colon resection for UC
Leads to portal hypertension and hepatic failure
(progressive fibrosis of intra and extrahepatic ducts)
Can have isolated intra or hepatic duct fibrosis
Complications- cirrhosis, cholangiocarcinoma

Answer 1
Tx- transplant needed long-term for most people
PTC drainage may be temporizing
Choledochojejunostomy may be effective in some
Balloon dilatation of strictures can provide symptomatic
relief
Cholestyramine can decrease pruritis by decreasing bile
acids
Irodeoxycholic acid can decrease symptoms by decreasing
bile acids as well as improve liver enzymes

Question 2
Which of the following regarding splenic
trauma is true?
A. Most children with blunt splenic trauma require
splenectomy
B. Patients may return to full-contact activity in 3
weeks
C. Active blushes and pseudoaneuryms are usually
managed non-surgically
D. Splenic salvage is associated with increased
transfusion.

Answer 2
D is correct. Review of splenic trauma->
Pts are considered fully healed in 6 weeks
Postsplenectomy sepsis is most common in pts 1-5 yrs and is
more likely within the first 2 years after splenectomy
Immunizations are required after trauma splenectomy
Threshold for splenectomy in children is much higher; they
rarely undergo trauma splenectomy
With conservative mgmt pt should be on bed rest for 5 days

Answer 2
Indications for operative mgmt for blunt spleen
injuries:
Pt becomes unstable despite aggressive
resuscitation, including at least 2 units of blood
Should be highly considered if the pt requires more
than 2 units of RBC to keep HCT at 25
Active blush on CT or pseudoaneurysm

Question 3
Which of the following proto-oncogenes is
associated with a G protein defect?
A.
B.
C.
D.
E.

Myc
Ras
Src
Sis
Erb B

Answer 3
B. Ras is associated with a G-protein defect
Ras

G-protien defect

Src

Tyrosine kinase defect

Sis

Platelet-derived growth factor


receptor defect

Erb B

Epidermal growth factor


receptor defect

myc

Transcription factors

Question 4
Which of the following antibodies is the most
abundant in the body?
A.
B.
C.
D.
E.

IgM
IgG
IgA
IgD
IgE

Answer 4
B. IgG is the most abundant
IgM- initial antibody made after antigen
exposure. Is the largest antibody with 5
domains and 10 binding sites
IgG- most abundant antibody. Responsible for
secondary immune response. Can cross the
placenta and provides protection in the
newborn period

Answer 4
IgA- found in secretions, in Peyers patches of
the bowel, and in breast milk. Helps prevent
microbial adherence and invasion in the gut
IgD- membrane-bound receptor on B cellsserves as an antigen receptor
IgE- Allergic reactions, parasite infections

Question 5
Regarding microsomal drug metabolism, which
of the following is true?
A. Phase II reactions describe demethylation,
oxidation, reduction, and hyrolysis reactions
B. Cimetidine is a P-450 inducer
C. Warfarin is a P-450 inhibitor
D. The P-450 system transforms aromatic
hydrocarbons into carcinogens

Answer 5
D. The P-450 system does transform aromatic
hydrocarbons into carcinogens.
To review;
Microsomal drug reaction takes place in the hepatic cell
endoplasmic reticulum via the P-450 system
Phase I reactions describe demethylation, oxidation,
reduction, and hyrolysis reactions
Phase II reactions describe those where glucoronic acid
(#1) and sulfides become attached to form a water-soluble
metabolite that is often inactive & ready for excretion.

Answer 5
Inhibitors of P-450
Allopurinol, Amiodarone

Cimetidine, Cipro
Disulfram, Erythromycin, Flagyl
Isoniazid, ketaconzaole, MAOIs, verapamil

Inducers of P-450
Cruciform vegetables, cigarette smoke, insecticides, ETOH
Dilantin, phenobarbial
Theophylline, warfarin

Question 6
A schistosomiasis abscess is best treated by:
A.
B.
C.
D.

Percutaneous drainage and antibiotics


Flagyl
Levofloxacin
Praziquantel

Answer 6
D. The treatment of a schistosomiasis liver
abscess is praziquantel. These cysts only need
percutaneous drainage for superinfection.
Surgery is indicated only for complications such
as bleeding.
These patients may also get esophageal varices

Question 7
An amoebic abscess is best treated by:
A.
B.
C.
D.

Percutaneous drainage and antibiotics


Flagyl
Levofloxacin
Praziquantel

Answer 7
B. An amoebic abscess is best treated with
flagyl. Percutaneous drainage is needed only
for super-infection. Surgery is only for
complications such as bleeding.

Question 8
A pyogenic abscess is best treated by:
A.
B.
C.
D.

Percutaneous drainage and antibiotics


Flagyl
Levofloxacin
Praziquantel

Answer 8
A. A pyogenic abscess is best treated by broad
spectrum antibiotics and percutaneous
drainage. GNRs are the most common
organisms in these cysts. You should also cover
for anaerobes
Liver abscesses can arise weeks after an
episode of diverticulitis, appendicitis, or other
abdominal infectious processes.

Question 9
All of the following are true about bladder cancer
except;
A. The most common type is transitional cell carcinoma
B. Schistosomiasis infection is a/w squamous cell cancer
of the bladder
C. Transitional cell cancer limited to the submucosa can
be treated with intra-vesicle BCG or trans-urethral
resection
D. Transitional cell cancer with muscle invasion (T2) can
be treated with wedge resection

Answer 9
D. Transitional cell cancer with muscle invasion
requires total cystectomy and ileal conduit
formation

Question 10
The most common type of ovarian tumor is:
A.
B.
C.
D.

Epithelial
Germ cell
Sex-cord
Lymphoma

Answer 10
A. The most common type of ovarian cancer is
epithelial

Question 11
The following structures are found in the
anterior compartment of the leg:
A. Posterior tibial artery, peroneal artery, tibial
nerve
B. Anterior tibial artery and superficial peroneal
nerve
C. Sural nerve, gastrocnemius, soleus, and plantaris
muscles
D. Anterior tibial artery, deep peroneal nerve

Answer 11
D. The anterior compartment of the leg
contains:
Anterior tibial artery and deep peroneal nerve
Muscles- anterior tibialis, extensor hallicus longus,
extensor digitorum longus, and communis.

The superficial posterior compartment


includes:
Sural nerve
Muscles- gastrocnemius, soleus, plantaris

Answer 11
The lateral compartment includes:
Superficial peroneal nerve
Muscles- peroneal muscles

The deep posterior compartment includes:


Posterior tibial artery, peroneal artery, and tibial
nerve
Muscles- flexor hallicus longus, flexor digitorum
longus, posterior tibialis

Question 12
You are asked to see a 1-day old infant with Downs
syndrome due to feeding intolerance. In addition to the
diagnosis of Downs the records indicate that
polyhydramnois was noted during the pregnancy. The
mother describes the babys emesis as green. Which of
the following condition does this child most likely have?

A.
B.
C.
D.

Tracheoesophageal fistula
Intussesception
Duodenal atresia
Pyloric stenosis

Answer 12
C. Duodenal atresia.

#1 cause of duodenal obstruction in newborns


Usually presents within 1st week
Usually distal to ampulla of Vater
Bilious emesis, feeding intolerance
Associated with cardiac, renal, GI anomalies as well as
polyhydramnois in the mother
20% have Downs syndrome
Double-bubble sign on plain film
Treatment- resuscitation, duodenoduodenostomy or
duodenojejunostomy

Question 13
A mother brings her 1-month old to you because of a
sudden onset of bilious emesis. You examine the
patient then order an upper GI which reveals that the
duodenum does not cross the midline. Given the
most likely diagnosis, what will ultimately be required
to correct this problem?

A. Pyloromyotomy
B. Air-contrast enema
C. Duodenojejunostomy
D. Resect Ladds band, counterclockwise rotation of bowel,
place cecum in LLQ (cecopexy), and perform an
appendectomy

Answer 13
D. This patient most likely has malrotation so the
appropriate procedure is resection of Ladds bands,
cecopexy, appendectomy, and counterclockwise
rotation of bowel.
Characteristics of malrotation
Sudden onset of bilious vomiting
Ladds bands cause duodenal obstruction, coming out of
the retroperitoneum near the Ligament of Treitz
Failure of normal counterclockwise 270 degree rotation

Answer 13
Volvulus is associated with compromise of the
SMA which can lead to intestinal infarction
90% present by 1 yr, 75% present within the 1st
month of age
Any child with bilious vomiting needs an upper GI
to rule out malrotation
On upper GI the duodenum does not cross the
midline

Question 14
A 48 yo male that drinks a case of beer daily had nausea
and vomiting at home about 5 hours before he presents
to the ER due to chest pain. Vitals- BP 100/50, HR 110;
you subsequently bolus crystalloid. A chest xray shows
a left effusion and pneumothorax. What is the most
sensitive test to diagnose this pts condition?
A. Abdominal CT
B. Chest CT
C. Angiogram

D. Gastrograffin swallow followed by thin barium swallow

Answer 14
D. Although a CT of the chest & abdomen with
oral and IC contrast is reasonable, a swallow is
the most sensitive test to diagnose
Boerhaaves syndrome

Question 15
The most common location for Boerhaaves to
occur is:
A.
B.
C.
D.

The left posterior lower esophagus


The left posterior middle esophagus
The upper stomach
The left posterior upper esophagus

Answer 15
A. The most common location for such a
rupture is the left posterior lower esophagus.
They usually will perforate into the left chest.

Question 16
You decide to explore this pt. What is the best
operative approach?
A.
B.
C.
D.

Right thoracotomy
Left thoracotomy
Thoracoabdominal incision
Laparotomy

Answer 16
B. The best approach to reach a perforation in
the left lower esophagus is a left thoracotomy.
If the lesion were in the middle or upper 1/3
you would go through the right chest

Question 17
You open the pt and find gastric contents
coming out through a hole in his esophagus.
You wash it out and then FIRST:
A. Place a chest tube and close
B. Perform a myotomy
C. Full thickness bites where you see fluid coming
out
D. Esophagectomy

Answer 17
B. Perform a myotomy. You want to see the extent
of the injured mucosa which often extends farther
than the muscle injury.
Esophagectomy isnt yet indicated in this pt
because it occurred 5 hours ago and gross
mediastinitis is unlikely
Esophagectomy is indicated:
When the esophagus is too badly damaged
When there is severe mediastinitis and the esophagus
cant be salvaged
More likely to be required in delayed presentations

Question 18
All of the following are true about nitrous
oxide (NO) except:
A.
B.
C.
D.

Increases cAMP
Precursor is arginine
Released primarily from vascular endothelium
Receptor is guanyl cyclase

Answer 18
A. Arginine is the precursor to nitrous oxide.
NO acts on guanyl cyclase to increase cGMP
and cause vasodilation

Question 19
All of the following are true of the hepatic
acute phase response except:
A.
B.
C.
D.

IL-6 is the most potent stimulus


Albumin is decreased
C-reactive protein is increased
Transferrin in increased

Answer 19
D. is false; transferrin and albumin are
decreased in the acute phase response.
Proteins that are increased include C-reactive
protein, amyloid A & P, fibrinogen,
haptoglobin, ceruloplasmin, alpha-1
antitrypsin, C3 (complement), and alpha-1
antichymotrypsin

Question 20
Which correctly describes the zones of the
retroperitoneum?
A. Zone 1 includes the pelvis and is associated with pelvic
fractures
B. Zone 2 includes the central retroperitoneum
C. If you find a hematoma in zone 2 when exploring the
patient for blunt trauma, you do not have to explore it if it
remains stable throughout the operation
D. Zone 3 injuries, whether they are blunt or penetrating,
always need to be explored

Answer 20
C. If exploring a patient for blunt injury and you
encounter a stable hematoma in zone 2, you do
not have to explore it.

Zone

Location

Associated injuries

Central
retroperitoneum

Pancreaticoduoden
al injuries, injuries
to major abdominal
vasculature

Flank, perinephric
area

Injuries to GU tract
or colon

Pelvis

Pelvic fractures

Answer 20
In general, penetrating injuries to any of the 3
zones need to be explored
Blunt zone 2 injuries that are stable can be
worked up radiographically to determine the
location of the injury. If they are found
intraoperatively and noted to be expanding,
they need to be explored.

Answer 20
Blunt zone 3 usually can be managed by pelvic
fixation.
Seventy percent of pelvic bleeding is from pelvic veins
Restoring the pelvis to its normal configuration cause the
hematoma to compress the veins and stop venous
bleeding
If the patient continues to be hemodynamically unstable,
an arteriogram with selective embolization of the bleeding
vessel coming from the internal iliac artery should be
performed

Question 1
Which of the following is true regarding the efficacy of
prophylactic antibiotics following a GSW through the colon?
A. Giving a single dose of abx 2 hrs preoperatively increases SSI in
these patients
B. Infection rates are significantly lower when antibiotics are given for
a complete 5 day course
C. There is no statistical difference when antibiotics are given within 2
hours of initial skin incision and when given within the first 3 hours
when the skin incision is made
D. There is no evidence supporting continuing abx beyond 24 hours
even when the bowel is perforated
E. The bullet is sterilized coming out of the handgun and the intraabdominal organs themselves are the major source of wound
infections in these patients

Answer 1
D. The bullet is not sterilized coming out of the barrel
The clothing draws the skin flora into the wound and is the
major source of wound infection
There have been studies comparing length of abx therapy
and no proven benefit has been demonstrated by
extending therapy beyond 24 hours if no abscess is present
and the source of bowel contamination is removed
There is a difference in SSI when an appropriate fose of abx
is given within 2 hours before the skin incision

Question 2
Which of the following is the most important
prognostic factor in patients with malignant
melanoma?
A.
B.
C.
D.
E.

Surface size
Tumor grade
Depth
SLN biopsy status
Margins of resection

Answer 2
D. The development of SLN Bx began an era where
micromets could be identified and staged more accurately,
allowing appropriate patients to receive total LN dissection
and treated with adjuvant therapy
For patients with stage 1 and 2, SNLB status has become
the most important prognostic factor
There is a survival benefit when patients with nodal
involvement receive high-dose interferon alfa 2-b adjuvant
Thickness does parallel the percent positivity of SLN (3.6%
for lesions less than 0.75 mm to 44% in tumors greater
than 4 mm)

Question 3
Which of the following is NOT a component of
informed consent?
A.
B.
C.
D.
E.

Disclosure
Comprehension
Competency
Voluntariness
Description of basic procedural standards of care

Answer 3
E is not a part of informed consent
Disclosure by the physician, patient
comprehension, competency, and
voluntariness all are.

Question 4
Which of the following is true about the
secretory diarrhea seen in gastrinoma
patients?
A. It partially is due to enterocyte damage from the
excessive acid
B. It can be improved by NG suctioning
C. It may be the only complaint in around 20% of
patients presenting with gastrinoma
D. A and C only
E. All of the above are true

Answer 4
E. All of the above are true about gastrinoma

Question 5
Which of the following forms does fat take to
enter the lymphatic system and eventually
reach the liver?
A.
B.
C.
D.

Free fatty acids


Chylomicrons
Triglycerides
Monoglycerides

Answer 5
B. Chylomicrons

Etiology of Steatorrhea in Gastrinoma


Fat digestion begins in the small bowel and begins with
the emulsification of fat by bile and lecithin, which are
partly soluble in water and partly in lipid
Larger fat globules get broken down into smaller
globules, increasing the total surface area of fat
This allows better access for pancreatic lipase, which digests TGs into Fatty acids
and 2-MGs

Bile salts separate fat lobules until they create micelles


These carry the FAs into the intestinal brush border for absorption

Etiology of Steatorrhea in Gastrinoma


Once inside cell, the FAs and MGs reform TGs
which combine with cholesterol,
phospholipids, and apoprotiens to form
chylomicrons
Chylomicrons then enter the lymphatic system
and eventually get transferred to the liver

Etiology of Steatorrhea in Gastrinoma


Gastrinoma is the second most common islet cell
tumor of the pancreas
Is the most common symptomatic endocrine tumor
of pancreas
Hallmark of Zollinger-Ellison Syndrome
25% of gastrinoma patients have MEN-1 syndrome
Main sx caused by increased gastric acid secretionabdominal pain most common (75%)
Diarrhea in 2/3 and may be only sx in 20%

Etiology of Steatorrhea in Gastrinoma


Increasing amount of gastric acid secretion
decreased the pH in duodenum after stomach
emptying
Decreased duodenal pH leads to breakdown of
pancreatic lipases- this leads to malabsorption and
steatorrhea
This also damages enterocytes and insolubilizes bile
salts
Unlike other forms of secretory diarrhea, secretory
diarrhea from gastrinoma can be treated by NG
suctioning

Question 6
A 63 yo female with a longstanding history of Type 2
diabetes and noncompliance with medication presents
with some bony deformity of her right foot and a clean
~4 cm plantar ulcer. The pulses are good and there is
no evidence of cellulitis. Which of the following is the
most appropriate initial treatment for this patient?
A. Ulcer debridement and skin grafting
B. Bony reconstruction and ulcer debridement
C. Modified fitted boots

D. Below knee amputation with early prosthetic fitting

Answer 6
C. A removable boot or appropriate shoe with
monitoring of the wound is most appropriate

Initial Treatment of Plantar Foot Ulcer in


Diabetic
Diabetes is the most common cause of neuropathy
leading to loss of protective sensation on the plantar
aspect of the foot
Bony deformity can also occur as a result of neuropathy
Combination of lack of protective sensation and
deformity leads to pressure ulceration and infection
Osteomyelitis can develop requiring bone resection or
amputation

Initial Treatment of Plantar Foot Ulcer in


Diabetic
Gold standard to treat diabetic foot ulcers- provide
appropriate pressure relief
Can be via a total contact cast or remoavable cast
boot
Further treatment o more advanced wounds
includes debridement with removal of all necrotic
tissue and to create a plantar grade foot
Surgery may be necessary in those whose deformity
doesnt respond to shoe modification
Surgical goals- resect bony prominences and infected
bone, restore bone architecture

Question 7
A 38 yo female underwent a laparoscopic Nissen
fundoplication and hiatal hernia repair 2 days ago that
was technically difficult. She now is mildly tachycardic
and complains of shortness of breath. Chest x-ray
shows an effusion on the left and a small amount of
free air under the diaphragm. Her SBP is 140/90 mmHg
and UOP is good. Which study is best to determine the
site of leak?

A.
B.
C.
D.

MRI of chest and abdomen


Water-soluble contrast study
EGD
EUS

Answer 7
B. Imaging with a water soluble contrast agent
is most appropriate

Dx Test for Pleural Effusion after Nissen


Feared complication after laparoscopic Nissen- esophageal or
gastric perforation
Causes:
Traction applied to esophagus or stomach
Inadvertent cautery burns during dissection

Leak will manifest during the first 48 hours


Peritoneal signs will be noted if spillage limited to abdomen
SOB and pleural effusion if spillage extends to chest
Site of leak should be confirmed by a water-soluble contrast
study
Perforation best handled by laparotomy though may be fixed
laparoscopically if discovered during initial operation

Question 8
Which of the following patients with an AAA
should be offered elective repair?
A. A 45 yo male with a 4 cm aneurysm that was 3.8
cm 6 months ago
B. A 59 yo female with a 5.1 cm asymptomatic
aneurysm
C. A 61 yo male with a 4.8 cm aneurysm that was
4.0 cm 6 months ago
D. All of the above should be offered elective repair

Answer 8
C is most appropriate

Indication for Surgical Treatment of


Asymptomatic Aneurysm

Ruptured AAA is the 10th leading cause of death in men in the USA
Risk of rupture increases with increasing diameter
Annual risk of rupture of 4-5 cm lesion is 1%
5-6 cm- 2-5%
3-10 cm- 3-10%; 5 year rupture is nearly 50%
Most are asymptomatic and are found incidentally
Elective repair should be offered when risk of mortality is less than
5%
Size less than 5.5cm- follow with biannual CT or U/S
Pts with aneurysms larger than 5.5 cm or which have growth over
0.5 cm in 6 months should be offered elective repair

Question 1
Of the following, which is the most critical
component of neovascularization in tumor
metastases?
A.
B.
C.
D.

HER receptor
VEGF receptor
Neu receptor
FGF receptor

Answer 1
B. One of the most critical elements in the
neovascularization of metastases is the VEGF
(vascular endothelial growth factor) receptor
Many new chemo regimens are targeting this
receptor

Question 2
Which of the following lab values is most
consisted with primary hyperparathyroidism?
A.
B.
C.
D.

Cl 105, phos, 3, elevated renal cAMP


Cl 105, phos 4, elevated renal cAMP
Cl 107, phos 3, and decreased renal cAMP
Cl 106, phos 4, decreased renal cAMP

Answer 2
A. Primary hyperparathyroidism is associated
with: an
Elevated PTH
Elevated Ca
Elevated renal cAMP (effect of PTH on the PTH
receptor in the kidney)
A Chloride / Phos ratio greater than 33

The half-life of PTH is 18 minutes

Question 3
Massive bleeding 7 days after a tracheostomy
is most likely due to:
A.
B.
C.
D.

Tracheo-carotid fistula
Tracheo-inominate fistula
Tracheo-aortic fistula
Tracheo-jugular fistula

Answer 3
B. The most common cause of massive
bleeding following a tracheostomy is a tracheoinnominate fistula
Place finger through the tracheostomy site to
try to press the innominate artery against the
sternum
If diagnosed, go to OR for median sternotomy

Answer 3
Ligate and divide the innominate artery (some
say place a graft but this is a high infection risk
for this complication)
Ligation of the innominate artery proximal to
the takeoff of the right subclavian usually
doesnt result in any neurologic dysfunction
due to collateral flow

Question 4
You start coumadin on a patient who just got
diagnosed with a PE. Three days later he starts to
slough skin from his arms and legs. All of the
following are true of this pts condition except:
A. This likely could have been prevented if you started
heparin before coumadin
B. Pts with protein kinase C deficiency are more susceptible
to this problem
C. The skin sloughing is caused by skin necrosis
D. This is most likely due to undiagnosed hemophilia A

Answer 4
D. Warfarin-induced skin necrosis occurs in patients
started on coumadin without first being given heparin. It
results from a relatively hypercoagulable state that occurs
in some people due to the relatively short half-life of
protein c & S.
Protein C & S decrease after coumadin administration
before the other factors start to decrease, leading to a
relative hypercoagulable state
Pts with protein C deficiency are at increased risk for
having this problem, which is prevented by starting heparin
before coumadin

Question 5
The most common benign tumor of the parotid
is:
The most common tumor to involve bilateral
parotids glands at the time of presentation is:
A. Mucoepidermoid ca
B. Adenoid cystic carcinoma
C. Pleomorphic adenoma
D. Warthins tumor

Answer 5
C. The most common benign tumor is a
pleomorphic adenoma
D. The tumor most likely to involve bilateral
parotids at the same time a Warthins tumor

Question 6
All of the following are true about pts that
have undescended testicles except:
A. You can wait until they are 2 yrs old to treat
B. There is a higher risk of testicular cancer in these
pts
C. The cancer risk is lower if both testicles are
brought into the scrotum early
D. The tumor they are most likely to get is a
seminoma

Answer 6
C. The risk of developing cancer is the same even if
the testicles are returned to the scrotum early.
However, returning them to the scrotum facilitates
examination.
The treatment when diagnosed is orchiopexy
through an inguinal incision.
If you cant bring the testicles down close and wait 6
months before trying again.
If you still cant get them down, divide spermatic vessels

Answer 6
The other points are true
You can wait until they are 2 yrs old to treat
There is a higher risk of testicular cancer in these pts
The tumor they are most likely to get is a seminoma

If undescended bilaterally get chromosomal studies


Prune belly syndrome- hypoplasia of the abdominal
wall, urinary tract abnormalities with a dilated
urinary system, and bilateral cryptorchidism

Question 7
Antibiotics can be subdivided into
bacteriostatic and bacteriocidal. Each of the
following is bacteriostatic except:
A.
B.
C.
D.

Bactrim
Tetracycline
Erythromycin
Gentamicin

Answer 7
D. Bacteriostatic agents include
chloramphenicol, tetracycline, clindamycin,
erythromycin, and bactrim

Question 8
Which of the following is not true about the
treatment of anal margin squamous cell cancers?
A. The anal margin is defined as those below the dentate
line
B. They tend to have a better prognosis than anal canal
lesions
C. Wide local excision is appropriate for lesions less than 3
cm
D. Metastases first tend to go to para-aortic nodes

Answer 8
D. Mets tend to go to the inguinal nodes first.
Squamous cell cancers of the anus are often
ulcerating
You can perform WLE with a 0.5 cm margin for
lesions less than 3 cm
The margin describes lesions below the dentate
line and do have a better prognosis than canal
lesions

Answer 8
An APR may be required for large lesions if the
sphincter is involved.
If the nodes are clinically positive, an inguinal
dissection should be performed.
Men have a better prognosis

Question 9
Which of the following are true about LeFort
fractures?
A. All 3 types can be managed with intramaxillary
fixation only
B. Type 3 involves the lateral orbital walls
C. Type 2 involves the lateral orbital walls
D. They generally are managed nonoperatively

Answer 9
B. Type 3 involves the lateral orbital walls.
Please refer to the following chart and
pictures.

Answer 9
Type

Description

Treatment

Maxillary fracture straight Reduction, stabilization,


intramaxillary fixation, +/across
circumzygomatic and
orbital rim suspension
wires

Lateral to nasal bone,


under eyes, down toward
maxilla

Same as type 1

Lateral orbital walls

Suspension wiring to
stable frontal bone, may
need external fixation

LeFort 1

LeFort 2

LeFort 3

Question 10
Sludging in the gallbladder and cholestatic
jaundice is most characteristic of which
antibiotic?
A.
B.
C.
D.

Quiniolones
Bactrim
Erythromycin
Ceftriaxone

Answer 10
D. Sludging in the gallbladder and cholestatic
jaundice are complications of ceftriaxone

Question 11
The most common infection in pts with large
(>35%) severe burns is:
A.
B.
C.
D.

Pneumonia
UTI
Wound sepsis
Liver abscess

Answer 11
A. Although UTI is the most common infection in
surgery pts, pneumonia is the most common
infection in people with severe burns.
Inhalation injury, decreased immunity, fluid
resuscitation leading to pulmonary edema, and
requirement for mechanical ventilation all lead to
increased risk for pneumonia
Up to 60-70% of pts with large burns get
pneumonia

Question 12
Post-op day 1 after a right upper lobectomy
your pt develops a 102 F fever. The most likely
source of the fever is:
A.
B.
C.
D.

PMNs
Platelets
Macrophages
Lymphocytes

Answer 12
C. Alveolar macrophages are activated with
atelectasis and cause fever

Question 13
The most important prognostic factor in pts
with sarcomas without metastases is:
The most important prognostic factor in pts
with breast cancer without metastases is:
A.
B.
C.
D.

Node status
Size of tumor
Tumor grade
Gender of pt

Answer 13
C and A.
Sarcomas the most important factor is tumor
grade. These tumors rarely go to lymph nodes
Nodal status is most important in breast cancer
(whether there are + nodes, how many, and
location- axillary, IMA)

Question 14
Of the anesthetics listed below, which of the
following is most likely to cause an allergic
reaction?
A.
B.
C.
D.

Lidocaine
Bupivicaine
Mepivicaine
Procaine

Answer 14
D. Amide type anesthetics (all have an I in the
1st part of their name) such as lidocaine,
bupivicaine, and mepicivaine rarely cause
allergic reactions
Ester type anesthetics such as procaine,
cocaine, and tetracaine are most likely to cause
allergic reactions because of their PABA
analogue

Question 15
The most common organism involved in
cholangitis is:
A.
B.
C.
D.

S. aureus
Streptococcus
Bacteroides fragilis
E. coli

Answer 15
D. E. coli is the most common organism
involved in cholangitis

Question 16
The most common cause of cholangitis is:
A.
B.
B.
C.

Gallstones
Malignancy
Benign biliary strictures
Iatrogenic injury

Answer 16
A. The most common cause of cholangitis in
gallstones

Question 1
All of the following are true of atrial natriuretic
peptide EXCEPT:
A. Released from atria when stretched
B. Causes natiuresis and diuresis
C. Is an inherent method of removing excess
volume in patients with CHF
D. Is a steroid hormone

Answer 1
D. ANP is released when the atrium are
stretched. The peptide inhibits Na absorption
and water absorption at the collecting ducts.

Question 2
The ventilation strategy that has been most
effective in pts with ARDS in terms of
morbidity and mortality is:
A.
B.
C.
D.

Volume control
Pressure control
Permissive hypoxia
Strict control of CO2 into normal ranges

Answer 2
B. Pressure control ventilation to avoid
barotrauma and permissive hypercapnia
(allowing an elevated CO2) while correcting
the pH and HCO3- has been the most effective
mode of ventilating pts with ARDS.
Other maneuvers, such as prone and supine
positioning, have been used.

Question 3
A 4 yr old girl is brought to the ER after being bitten
by a classmate. The bite is on her forearm and
there is not sign of devitalized tissue. The would is
irrigated and closed. Cultures of the wound will
most likely grow:

A.
B.
C.
D.
E.

S. aureus
Eikenella corrodens
Fusobacterium species
Prevotella species
Pasteurella multocida

Answer 3
A. S. aureus. The most common bacteria found in human
bites are S. aureus, S. epidermidis, alpha and beta strep,
Corynebacterium sp, and Eikenella corrodens
Occlusional human bites- teeth puncture
Infection risk similar to dog and cat bites
Hand wounds- higher infx risk

Clenched fist- Fist hits teeth and causes injury


More dangerous; can lead to septic arthritis and osteomyelitis

Answer 3
E. corrodens infects up to 25% of closed fist injuries
and can lead to chronic, indolent infections
Human bites worse than dog or cat
Tx human bites- wash w/ iodine, elevate if possible,
give broad spectrum abx if on the hand or those at
higher risk; tetanus
Tx animal bites- most are dog; tx with washout,
broad spec abx. Primary closure OK if not on hand,
less than 12 hrs old, and not a cat bite
Most common bacteria- Pasturella multocida

Question 4
4 yrs after surgery for a GSW to the right thigh, a 26
yo man is referred to you for a thrill and bruit over
the area discovered on a routine exam. Which of
the following might you also find in this pt?

A.
B.
C.
D.
E.

Cardiac output of 3.2 L/ min


HR of 55 bpm
MVO2 of 55%
BP of 160/90 mmHg
Left ventricular hypertrophy

Answer 4
E. AV fistulas can occur following penetrating trauma to the
abdomen or the extremities.
They are usually asymptomatic, especially in young pts, but
can lead to high output cardiac failure due to shunting or
distal ischemia due to steal.
Left-to-right shunting will caused elevated mixed venous O2
and lead to high output failure (LVH, tachycardia, low BP)
Branham-Nicoldoni sign- compress the artery proximal to
the AVF- causes reflex bradycardia and increased blood
pressure
Prompt surgical tx (open or endovascular) is necessary

Question 5
Which of the following is characteristic of
Merkel Cell cancer?
A. Slow growing, well-defined cutaneous lesion
B. Early distant mets
C. Locally aggressive with low chance of distal
spread
D. Frequently cured with WLE alone
E. Similar to squamous cell ca histologically

Answer 5
B. Early distant mets. Merkel cell tumors are most
similar to basal cell lesions. Both are aggressive
and may have distant mets to the bone, nodes, and
viscera
Difficult to treat though need to be removed with
2-3 cm margins, elective regional
lymphadenectomy, SNL bx, radiation
Radiation can be used as adjuvant therapy for both
local and regional control and when surgery is not
an option
Chemotherapy is under investigation

Question 6
The mechanism of 5-FU is to:
A.
B.
C.
D.

Inhibit thymidalate synthesis


DNA alkylation
Inhibit microtubule formation
Stabilize microtubules

Answer 6
A. 5-FU inhibits thymidalate synthesis.
Leucovorin increases the toxicity of 5-FU

Question 7
Histamine release is characteristic of:
A.
B.
C.
D.

Demerol
Fentanyl
Sufentanil
Morphine

Answer 7
D. Morphine has a characteristic histamine
release which can cause hypotension

Question 8
4 days after an Ivor-Lewis esophagectomy you
start enteral feeds through a j-tube. The long
chain fatty acids contained in tube feeds:
A.
B.
C.
D.

Enter the circulation via the portal vein


Enter the circulation via lymphatics
Are only synthesized in the body
Are not found in chylomicrons

Answer 8
B. Long chain fatty acids enter the body
through terminal lacteals (absorption through
the lymphatic system) either as free long chain
fatty acids or as chylomicrons

Question 9
An overdose of fentanyl is treated with:
An overdose of ativan is treated with:
A.
B.
C.
D.

Flumazenil
Narcan
Neostigmine
Edrophonium

Answer 9
B and A. All narcotic agents can be treated
with Narcan when overdoses.
Overdoses of benzodiazapenes can be treated
with flumazenil

Question 10
During the resection of a pelvic tumor, the left
ureter is inadvertently transected below the
pelvic brim. Which of the following do you
choose for immediate repair of this problem?
A.
B.
C.
D.
E.

Primary repair
Primary repair with stent
Diversion with ureteroenterostomy
Ureterocystostomy
Delayed repair and perc drain the urinoma

Answer 10
D. Ureterocystostomy
When the ureter is accidentally ligated and the injury
missed, the pt develops hydronephrosis and urosepsis. This
can lead to pressure buildup and urine leak
For minor injuries where there is a partial laceration, the
ureter can be repaired over a stent
For complete transections, primary closure should be
attempted
Below the pelvic brim, ureteroureterostomy can be difficult
so a ureterocystostomy is the procedure of choice

Question 11
Which of the following is a characteristic of a
cutaneous lymphatic malformation?
A.
B.
C.
D.
E.

Bluish mass with overlying telangiectasias


Pulsatile ballottable mass
Cystic mass with overlying vesicles
Firm, nodular mass
Irregular mass fixed to the underlying tissues

Answer 11
C. Cystic mass with overlying vesicles
Cutaneous lymphatic malformations often
occur as cystic masses with overlying vesicles.
A lymphatic mass wound not be associated
with telangiectasias.
They are also not nodular or pulsatile and they
are not be fixed to underlying tissue

Question 12
A 22 yo man suffers a severe pelvic fracture
and has hematuria. You get a retrograde
cysturethrogram and see an extraperitoneal
bladder rupture. The most appropriate
therapy is:
A.
B.
C.
D.

Foley x 7 days
Exploratory laparotomy
Nothing
Cystectomy

Answer 12
A. The most appropriate therapy for an
extraperitoneal bladder rupture is urinary
catheter drainage for several days
The most appropriate therapy for an
intraperitoneal bladder rupture is ex lap and
primary repair.

Question 13
A 27 you man is in the ICU and is s/p splenectomy
for MVA 6 hrs ago. The patient had a prolonged
transport and received 20 units of blood prior to
arrival. Currently his peak airway pressures are 65,
his abdomen is distended, he is not making urine,
and his bladder pressure is 40. The most
appropriate action is:

A.
B.
C.
D.

Increase PEEP
Volume resuscitation
Decompressive laparotomy
CT scan

Answer 13
C. This is a classic description of compartment
syndrome.
A bladder pressure above 25, decreased UOP
(compression of the IVC causes decreased
cardiac output) and elevated ventilation
pressures are part of the syndrome
Treat with decompressive laparotomy

Question 14
When performing an abdominal perineal
resection, the vessels located in the lateral
stalks are:
A.
B.
C.
D.

Middle rectal arteries


Superior rectal arteries
Superior sigmoidal arteries
Inferior rectal arteries

Answer 14
A. The middle rectal arteries are located in the
lateral stalks.

Question 15
A newborn fails to pass meconium in the 1st 24
hours and develops abdominal distention. Plain
films show a distended colon. The childs anus is in
the correct location but on rectal exam there is an
explosive release of watery stool. The next most
appropriate step is:

A.
B.
C.
D.

Upper GI
Barium enema
Enteroclysis
Rectal biopsy

Answer 15
D. The scenario is most consistent with
Hirschsprungs disease. The diagnosis is made
with rectal biopsy which shows an absence of
ganglion cells in the myenteric plexus

Question 16
A 65 yo woman has stage 2 squamous cell
cancer of the vulva. What is the most
appropriate therapy?
A. Radiation
B. Chemotherapy
C. Resection of the labia
D. Bilateral labial resection

Answer 16
D. Stage 1 vulvar cancer is limited to 1 labia and is
less than 2 cm
Stage 2 is > 2 cm
Stage 3 involves nodes or invasive disease beyond
the labia
Treatment of stage 1- remove labia
Stage 2- bilateral labial resin
Stage 3- Wide en bloc resection and nodal
dissection

Question 17
The most common cause of persistent
hyperparathyroidism after surgery is:
A.
B.
C.
D.

Parathyroid cancer
Parathyroid hyperplasia
Missed adenoma during original operation
New adenoma

Answer 17
C. The most common cause of persistent
hyperparathyroidism is a missed adenoma.
Approximately 5% of patients having surgery
for an adenoma have multiple adenomas.

Question 18
A 20 yo man has a BP of 240/120 mmHg. He
occasionally gets headaches when he works out.
Given the most likely do, which of the following
tests is the best for confirming it?
A.
B.
C.
D.

24 hr urine VMA and metanephrines


Urine cortisol
Urine aldosterone
Serum epinephrine and norepinephrine levels

Answer 18
A. The best test for pheochromocytoma is a 24
hr urine VMA and metanephrine collection

Question 19
The test you order for the last pt is suggestive
of pheo. You order a CT but cant find the
mass. The most sensitive test for finding the
pheochromocytoma is:
A.
B.
C.
D.

MIBG
MRI
PET
Angiogram

Answer 19
A. MIBG scan (I-131-metaiodobenzylguanide),
a norepi analogue, is the best test

Question 20
MIBG scan in the last pt shows a tumor in the
right adrenal. You want to get the patient
ready for surgery so you prescribe:
A.
B.
C.
D.

Propanolol
Phenoxybenzamine
Dilaudid for headaches
ASA

Answer 20
B. The 1st drug you should prescribe is an alpha
blocker. Then you want to hydrate them as
much as possible. You may have to gradually
increase the alpha blocker until they have slight
orthostatic hypotension. This should be done 4
weeks before surgery.
Do not prescribe beta blockers as the first drug
as it can lead to hypertensive crisis from
unopposed alpha stimulation

Question 1
You are evaluating abdominal imaging on a 32 yo
female patient that is s/p high speed MVA. A 3-cm
lesion is noted in the periphery of the right lobe of
the liver that is homogenous with a central stellate
scar. Which of the following liver lesions is this
most likely to be?

A.
B.
C.
D.

Hepatocellular carcinoma
Hepatic adenoma
Focal nodular hyperplasia
Amoebic abscess

Question 2
Which of the following should you recommend
for this patient?
A.
B.
C.
D.
E.

Wedge resection
Right lobectomy
Radiofrequency ablation
Observation if asymptomatic
None of the above

Answer 1
C. Focal nodular hyperplasia- central stellate
scar in a homogenous lesion

Answer 2
D. Observation is appropriate management in
asymptomatic FNH

Focal Nodular Hyperplasia


FNH is a benign liver tumor that can be managed w/
observation if asymptomatic
Resection indicated if pt has RUQ pain/ fullness, if
there is concern about the diagnosis after
observation, or if lesion has bled
Second most common benign liver tumor
Women; 20-40s
A/w OCP use

Focal Nodular Hyperplasia


Usually found incidentally
Unlikely to cause symptoms
Conservative mgmt appropriate unless patient
is symptomatic- unlikely to rupture
Radiographic features:
Has Kupffer cells so will take up sulfur colloid
MRI/ CT scan will show a hypervascular tumor

Question 3
A 35 yo woman presents with right upper quadrant
pain and undergoes a CT A/P. Her PMH is negative
though she does take birth control pills. She has a 3
cm in the right lobe of the liver that appears to be
hypervascular but does not have uptake on
subsequent sulfur colloid scan. Which of the
following tumors is most likely?

A.
B.
C.
D.

Hepatic cyst
Hepatic adenoma
Hepatocellular carcinoma
Metastatic disease

Answer 3
B. This hypervascular tumor in the right lobe
with no uptake of sulfur colloid is most likely
an adenoma.

Question 4
Given the above scenario, which of the
following should you recommend for this
patient?
A.
B.
C.
D.

Observation
Radiofrequency ablation
Resection
Multiagent chemotherapy

Answer 4
C. Given that you suspect a symptomatic
adenoma, resection is indicated

Hepatic Adenoma
Higher risk- women, steroid or OCP use, type 1
collagen storage disease
80% are symptomatic; 10-20% with risk of
bleed from rupture
Can become malignant
Most common in right lobe
Sx- pain, increased LFT, decreased BP (rupture),
palpable mass

Hepatic Adenoma
Dx- no Kupffer cells so no uptake on sulfur
colloid scan
MRI- hypervascular tumor with peripheral
blood supply
Treatment Asymptomatic- stop OCP- no further tx if regresses.
If no regression, pt needs resection
Symptomatic- resection for bleeding & malignancy
risk; if multiple and not resectable-> embolize

Question 5
A 35 yo female is found to have a 3 cm lesion in the right liver
found on CT s/p workup from an MVE. She is found to have a
3 cm tumor in the right lobe of her liver that appears
hypervascular with significant peripheral to central
enhancement. She otherwise reports feeling fine and does
not have any history of abdominal pain. Which of the
following is most likely to be true?

A.
B.
C.
D.
E.

This is the most common benign hepatic tumor


Rarely it may be associated with a consumptive coagulopathy
You should not biopsy this tumor as there is a risk for rupture
A and B
All of the above are true

Answer 5
E. This most likely is a hepatic hemangioma so
all of the above are true

Hepatic Hemagiomas

Most common benign hepatic tumor


Rare risk of rupture, most asymptomatic
More common in women
Avoid biopsy b/c of hemorrhage risk
Dx- MRI and CT- peripheral to central enhancement that
appears hypervascular
Treatment Conservative if asymptomatic
Symptomatic- resection +/- embolization
XRT and steroids if unresectable

Rare complications- consumptive coagulopathy (KasabachMerritt syndrome) and CHF- more common in children

Question 6
A 25 yo female is found to have a 3-cm lesion in her
liver found incidentally on CT consistent with a cyst.
Which of the following is true about this lesion?
A. This is an acquired lesion that is most likely secondary
to a history of abdominal trauma
B. They should be resected because there is a 10%
incidence of malignant degeneration
C. They occasionally can become infected but often can
be treated by percutaneous drainage
D. All of the above

Answer 6
C. A hepatic cyst is usually benign and does
not require treatment.

Solitary Cysts
Congenital; more common in women and in
right lobe
Resection needed if infected and cant be
treated percutaneously
Complications rare; most can be left alone
Walls have characteristic blue hue

Question 7
A 50 yo male that underwent a Billroth II gastric resection for
gastric outlet obstruction 1 month ago presents for follow-up.
He states that he experiences nausea, occasional nonbilious
emesis, and dizziness about 2 hours after he eats. Which of
the following is most likely?
A. If you were to check his glucose when the symptoms occur, it
would most likely be high
B. This usually does not improve with conservative treatment so you
should anticipate converting him to a Roux-en-Y
C. Octreotide may be helpful
D. This is most likely due to retained antrum so you will need to
consider repeat resection

Answer 7
C. This describes late dumping syndrome and
may respond to octreotide

Dumping Syndrome
Gastrointestinal and vasomotor symptoms
Thought to be due to unmetered entry of ingested food
into SB
May be seen after vagotomy & resection or division of
pyloric sphincter
Early sx- immediately after meal from hyperosmotic
load into the small bowel
Nausea, epigastric discomfort, borborygmi, palpitations, dizziness

Late- 1-3 hrs after meal


Reactive hypoglycemia in addition to sx of early dumping

Dumping Syndrome
Usually responsive to dietary modifications (small, lowfat diet, limit liquids with meals)
Octreotide- can improve symptoms
50-100 mg before meal
Inhibit release of vasoactive peptides from gut
Decrease peak insulin levels and slow intestinal transit
Prevents changes in pulse, systolic BP, and RBC volume during early dumping and
blood glc levels during late dumping

Question 8
A 50 yo male that underwent a subtotal gastrectomy with BII
reconstruction for a T2 adenocarcinoma 1 month ago presents for
follow-up. He experiences frequent epigastric pain after meals with
subsequent bilious vomiting that does not relieve the pain. He has
lost about 15 pounds though otherwise has been stable. Which of
the following is false?
A. He will likely respond well with dietary modification and bile chelators
B. This usually does not improve with conservative treatment so you should
anticipate converting him to a Roux-en-Y
C. This is most likely due to retained antrum so you will need to consider
repeat resection
D. All of the above
E. A and C only

Answer 8
B. This is consistent with alkaline reflux which
is less likely to respond to conservative
treatment and will ultimately need conversion
to a Roux-en-Y

Alkaline Reflux Gastritis


Postprandial epigastric pain, nausea, vomiting of bile,
evidence of bile reflux into stomach with gastritis
Persist in 1-2% of patients who have had vagotomy/
drainage or resection
Differential dx- recurrent ulceration, biliary or pancreatic
disease, afferent loop obstruction, esophagitis
Gastric acid analysis- basal hypochlorhydria with little
increase w/ pentagastrin administration
Obtain serum gastrum measurements to r/o ZE syndrome
and retained antrum

Alkaline Reflux Gastritis


Endoscopy- red, friable mucosa, edema, glandular
atrophy, intestinal metaplasia
Antacids, H2 lockers, bile acid chelators, dietary
manipulation not definitively beneficial
Reoperation to divert contents from gastric mucosa
Convert to Roux-en-Y GJ with 50-60 cm intestinal limb
Very effective in eliminating bilious vomiting
20-30% still have pain or persistent gastric emptying
dysfunction

Question 9
You perform a subtotal gastrectomy with B2 reconstruction on
a 62 yo man with an antral GIST tumor. On postoperative day
1 he is experiencing severe abdominal pain and nonbilious
emesis. You obtain imaging which is concerning for a
markedly dilated bowel limb proximal to the anastomosis.
Which of the following is true?
A. This is most likely secondary to anastomtic edema so you should
place an NGT and start octreotride
B. You would be less likely to have this complication if you did an
antecolic gastrojejunostomy
C. This requires urgent operative intervention
D. In the chronic form (from partial obstruction) patients may develop
microcytic anemia
E. B, C, and D

Answer 9
C. This picture is concerning for afferent limb
obstruction- the patient needs intervention as
he is high risk for duodenal stump blowout.

Afferent Limb Syndrome


Partial or complete obstruction of the afferent limb
proximal to the GJ anastomosis
Afferent limb- duodenal remnant and segment of
jejunum proximal to gastrojejunostomy
Jejunal limb is subject to adhesions, herniaton,
obstruction, volvulus
Rare (~1%) after Billroth II
Can occur immediately or remotely
Acute- operative emergency; secretions remained trapped in limb so vomiting is
nonbilious
Chronic- from partial obstruction- relief of postprandial discomfort with
projectile bilious vomiting

Afferent Limb Syndrome


One of main causes of duodenal stump blowout
Prolonged stasis in chronic disease can lead to bacterial
deconjugation of bile salts-> steatorrhea, malnutrition, Vit
B12 deficiency-> megaloblastic anemia
Pts with increased risk of developing syndrome
Jejunal portion of afferent limb longer than 30 cm
Antecolic GJ
Improper closure of mesocolic defects

Surgical intervention usually necessary


Some attempting endoscopic therapy, though not routine

Question 10
You are performing a diagnostic laparoscopy in a 28
yo female that you suspect has appendicitis. In
addition to an inflamed appendix, you find a 4 cm
right ovarian cystic mass. Which of the following
describes the most appropriate management?
A. Aspirate the cyst and send the fluid for cytology
B. Remove her appendix and perform a right
salpingoophorectomy
C. Remove her appendix and recommend to her postoperatively that she gets follow-up for an ovarian cyst
D. Perform a TAH/BSO

Answer 10
C. In a cyst on a woman of childbearing years
and unknown desire to have children, the best
step would be to perform an appendectomy
and monitor the cyst postoperatively

Question 11
A 55 yo female is undergoing diagnostic
laparoscopy for suspected appendicitis. You find an
inflamed appendix but also note a 6 cm cystic
appearing right ovarian mass. The next best step
should be:
A. Inform her next of kin and obtain consent for a BSO/
hysterectomy
B. Inform her next of kin, perform an appendectomy, and
aspirate the lesion to send for cytology
C. Inform her next of kin, perform an appendectomy and
unilateral salpingoophorectomy, await pathology
D. None of the above

Answer 11
C. Given this patient has a relatively large
cystic lesion and is most likely done with
childbearing, removing the affected tube and
ovary is an appropriate first step. She may
need further surgical treatment based on the
pathology.

Ovarian Mass- Benign


May be solid or cystic
Functional process or neoplasia
Size, age of patient, tumor characteristics factor in on necessity of
surgical removal
90% tumors in women younger than 30 benign
80% benign in 30-50 yrs
50% malignant in pts over 50
Follicular cysts- failure of a developing follicle to develop or regress.
Usually less than 8 cm
Corpus luteal cysts- occur from hemorrhage into CL; blood-filled
yellow/ granular. Usually regress in 4-8 weeks
Thecal luteal cyst- may be present in pregnancy with high circulating
gonadotrophins
Chocolate cysts- pelvic endometriosis

Treatment Ovarian Mass- Incidental


In reproductive age patient:
Solid, unilateral- ovarian resection and frozen
section
Usually will be benign stromal neoplasm (fibroma)
If lesion comes back as a germ-cell malignancy TAH/ BSO, omentectomy, staging bx/ LND
Can leave uterus and contralateral adenxa behind & perform a
retroperitoneal LND, omentectomy, staging biopsy

If bilateral dysgerminoma found (rare) Resect both if pt has no interest in fertility


Resect only larger and treat smaller with chemo if fertility
interest + or unknown

Treatment Ovarian Mass- Incidental


Cyst Usually benign and asymptomatic
Most are serous cystadenomas
Tx- premenopausal, less than 5 cm- observe. Cystectomy
or oophorectomy for cysts > 5cm, those with solid
elements, or postmenopausal women

Endometriosis- common nonmalignant cause of


pelvic pain
Treat conservatively
If diagnosis questionable, can biopsy- try to preserve
ovarian tissue if possible

Question 12
You are performing a laparoscopic ventral hernia repair on a
patient when the overhead light falls down, landing on and
breaking the patients left arm. Which of the following
describe an appropriate JCAHO response to this event?
A. Conducting a root cause analysis
B. Ensure that the chief of staff and OR supervisor are immediately
involved
C. This is not considered a sentinel event because it is not due to your
medical error
D. Development of an improvement plan and implement the
improvements
E. A and D only
F. A, B, and D

Answer 12
E. Conducting a root cause analysis and
developing an improvement plan to reduce
risks are correct

JCAHO Requirement of a Sentinel Event


Sentinel event- an unexpected occurrence that results in an
unexpected death or serious injury
Called sentinel b/c when they occur they signal the need for
immediate investigation and response
Not necessarily synonymous with medical errors- not all are due to
medical errors and not all med errors are sentinel events
Accredited organizations are expected to identify and respond to all
sentinel events. An appropriate response required by JCAHO
includes:
Conducting a root cause analysis
Developing an action plan designed to implement improvements to reduce
risk
Implementing the improvements
Monitoring the effectiveness of those improvements

Question 13
You are attempting to perform a parathyroidectomy for
hyperparathyroidism due to suspected adenoma and are using
intraoperative PTH levels. You find a small, normal-appearing
right upper, lower, and left upper gland. You cannot identify
the left lower gland. Which of the following choices is most
likely correct?
A. The patient probably only has 3 glands so the hyperparathyroidism
was probably misdiagnosed as an adenoma and is most likely
secondary to hyperplasia
B. Upper glands are more likely to be in unusual position than lower
glands
C. This gland may be found in the thymus
D. You should close at this point and repeat sestamibi postoperatively

Answer 13
C. Around 15% of inferior parathyroids are
found in the thymus

Treatment for Missing Parathyroids


Explore the sites where parathyroids are usually located
near the posterolateral surface of the thyroid gland
Most inferior glands are within 1 cm from where the
inferior thyroid artery crosses the RLN
When a lower gland is missing, it may be found in the thymus (15% of inferior
parathyroid)

Normal superior glands are more consistent in position80% are near posterior aspect of upper & middle lobes
at level of cricoid cartilage
1% of normal upper glands may be found in paraesophageal or retroesophageal
space

Question 14
A 60 yo man presents with dysphagia and pain after
swallowing. A barium swallow shows a smooth,
solitary mass in the lower 1/3 of the esophagus.
Which of the following is not true about this
condition?

A.
B.
C.
D.
E.

This tumor has a high malignant potential


It should be removed by esophageal resection
It may be adequately treated by enucleation
A and B are not true
All of the above are not true

Answer 14
D. A and B are not true. This tumor is most
likely a benign esophageal leiomyoma and can
be adequately treated with enucleation.

Benign Tumors of Esophagus


Benign tumors and cysts of the esophagus are uncommon
Can be intramural or intraluminal
Leiomyomas- 50% of benign tumors

Usually solitary in lower 1/3


Dysphagia and pain most common sx
Dx- barium swallow
Biopsying may scar tissue planes, complicating removal
Can be removed by simple enucleation

Esophageal cysts
Enteric or bronchogenic cysts
Usually intramural in the middle or lower 1/3
Treat by enucleation

Question 1
A 62 yo man presents with ischemic lesions in his right foot.
You work him up and discover a popliteal aneurysm. Which
of the following is not true about this condition?
A. There is a significant chance he has a contralateral aneurysm
B. It is an uncommon peripheral aneurysm
C. Asymptomatic lesions must be repaired
D. Around 1/3 or them also have AAA

Answer 1
B. It is an uncommon peripheral aneurysm is
false- it is the most common

Popliteal Artery Aneurysm

Most common peripheral aneurysm ( 70%)


50% are bilateral
30% also have AAA
20 30% of limb loss with distal emboli
Elective repair of all, regardless of size
Management options:
Medial exploration-> proximal/distal ligation & bypass
Acute thrombosis- preoperative thrombolytics
Endovascular repair (not yet accepted)

Question 2
A 65 yo man presents with progressive dysphagia.
You perform EGD which reveals a GE junction mass.
Which of the following would be an indication to
perform neoadjuvant treatment?

A. T1 adenocarcinoma with perineural invasion


B. T2 lesion with well-differentiated pathology
C. 2 cm LN near tumor on EUS
D. All of the above are indications for neoadjuvant
treatment with GE jxn tumors

Answer 2
C. 2 cm LN near tumor on EUS

Indication Neoadj. Tx in GE Jxn Cancer

Preoperative chemo/XRT should be done for


any:
T3/4 tumor
N + disease

T1 or T2 generally surgery first.


Staging for these tumors should also include
PET/CT.

Question 3
You perform an EGD on a patient well-known to you
with Barretts for screening. He has been wellcontrolled symptomatically with omeprazole. You
biopsy the GE jxn and final pathology returns as
Barretts with high grade dysplasia. Which of the
following should you recommend?

A.
B.
C.
D.

Nissen Fundoplication
Chemoradiation
Esophagectomy
None of the above

Answer 3
C. Esophagectomy should be recommended

Barretts & High Grade Dysplasia

Esophagectomy is considered the standard


Have 10% incidence of associated malignancy
with high-grade dysplasia
For non-high grade Barretts, endoscopic
surveillance and treatment of reflux (surgical
or medical) is appropriate

Question 4
Which of the following is a contraindication for
sentinel lymph node biopsy in breast cancer?
A. History of mastectomy
B. Invasive ductal cancer with poor histological
features
C. A patient with high grade DCIS
D. A T4 tumor with extension into the axilla

Answer 4
D. A very large tumor extending into the axilla
would not undergo SLN biopsy

Indications for SLN Bx in Breast Cancer


For our purposes, SLN indicated for any invasive
cancer except T4
For test purposes, any patient with clinically
proven nodes is not a SLN candidate (need an
ALND)
Mastectomy is not a contraindication for SLN
On the test, DCIS is not an indication unless it is
high grade disease or the patient is otherwise
undergoing a mastectomy

Question 5
A 52 yo female comes into the office noting easily
expressible bloody nipple discharge on the left side.
Exam demonstrates blood and mammogram is
otherwise negative. Which of the following is most
likely to be true?
A. This is likely an intraductal papilloma and there is a
50% chance of this being malignant
B. It should be treated with mastectomy
C. Localization with major duct excision Is usually curative
D. Patients with this condition have an underlying DCIS in
2/3 of cases

Answer 5
C. Localization with major duct excision Is usually
curative

Intraductal Papilloma of Breast


Most commonly present with bleeding/bloody
nipple discharge.
Generally resect via major duct excision or
needle localization if seen on imaging
This is a benign condition- very rare to be
malignant or premalignant

Question 6
You are examining a 67 yo male in whom you
performed a cholecystectomy on 2 weeks ago. His
final pathology reveled a focus of adenocarcinoma in
the lamina propria of the gallbladder but no invasion
into the muscularis. Which of the following should
you recommend?
A. Resection of segment 4-5 of the liver
B. Resection of segment 4-5 of the liver, CBD resection
with hepaticojejunostomy and regional LND
C. Observation only
D. Chemoradiation

Answer 6
C. In this lesion that does not invade the
muscularis, cholecystectomy is adequate

Gallbladder Cancer
If T1a only (does not invade muscle layer,
confined only to lamina propria), then
cholecystectomy alone appropriate
Anything more advanced->
Resection of gallbladder & liver fossa (wedge
resection segment 4/5 liver)
Regional node dissection

Consider resection of CBD if cystic duct


involved

Question 7
You are performing a cholecystectomy on a 35 yo female with
acute cholecystitis. It is a difficult dissection so you decide to
perform an intraoperative cholangiogram. It appears that you
have inserted the catheter through the common bile duct
itself. Which of the following is the most appropriate thing to
do next?
A. Remove the catheter, continue with cholecystectomy, ask GI to
perform ERCP and stenting postoperatively
B. Perform a choledochoduodeonostomy if the lesion is large
C. Repair the lesion over a T-tube if the injury is small
D. Perform a hepaticojejunostomy regardless of the size of the injury

Answer 7
C. Repair the lesion over a T-tube if the injury is small

Management of CBD Injury


Appeared on 2006 & 2007 tests with high % of
incorrect answers
Minor injury recognized intra-operatively- repair
primarily, usually over a T-tube or stent
Major injury recognized intra-operatively or injury
recognized post-op usually will require roux-en-y
hepaticojejunostomy
Possible incorrect option may be
hepatico/choledochoduodenostomy
Would not be a good choice in a nondilated duct

Risk For OPSI


Remember most children with sickle cell
anemia auto-splenectomize by early age.
Children younger than <4yrs are at highest risk
post-splenectomy
Splenectomy done for thalassemia carries an
increased risk

Question 8
Which of the following is the fuel source for
coloncytes and has been used for enemas in
those with ulcerative proctitis?
A.
B.
C.
D.

Lysine
Nitric oxide
Butyrate
Prostaglandin E

Answer 8
C. Butyrate

Fuel Source Coloncyte


Butyrate is the preferential source of fuel for
colonocytes
It is usually derived from bacterial
fermentation with the colon

Question 9
Which of the following is the most common
cause of delayed infection (several months out)
of a AAA graft?
A.
B.
C.
D.

Salmonella
Strep pneumoniae
Staph aureus
Staph epidermidis

Answer 9
D. Staph epidermidis

AAA Infections
40% of aneurysmal infections caused by
salmonella
Staph aureus and gram negative bacteria are
most common organisms in early graft
infection
Staph epidermidis is more chronic
Usually presents > 4 months; 2 years is average time
of presentation

Question 10
A 35 yo male that had short bowel syndrome
secondary to multiple resections from Crohns has
been on TPN for the last 3 years. You note that he
has wounds that are very slow to heal, a periorbital
rash, and some darkening of his skin creases.
Which of the following vitamin deficiencies is this
most likely from?

A.
B.
C.
D.

Vitamin K
Vitamin B 12
Copper
Zinc

Answer 10
D. Zinc

Zinc Deficiency in TPN


If they ask a question relating to TPN and
vitamin deficiency, zinc is almost always the
answer (usually appears every other year)
Zinc deficiency is associated with periorbital
rash, darkened skin creases, neuritis and
chronic, non-healing wounds

Question 11
A 35 yo male that you are evaluating for inguinal
hernia repair informs you that he has a family
history of getting dangerously overheated when
undergoing anesthesia. Which of the following is
not true about this condition?

A.
B.
C.
D.

Hyperthermia is usually the first symptom


It can be triggered by halogenated inhalational agents
Dantrolene should be used to treat
Rise in CO2 is usually the first symptom

Answer 11
Hyperthermia is usually the first symptom

Malignant Hyperthermia
Can be genetically transferred
Triggered by halogenated inhalational agents
(1 in 250,000)
Can also be triggered by succinylcholine (1 in
60,000)
Earliest sign is rise in CO2
Hyperthermia is a relatively late finding
Treat with dantrolene

Question 12
Which of the following are not diagnostic criteria
for HNPCC?
A. 3 or more family members with a HNPCC associated
cancer, one of whom is 1st degree relative of others
B. 2 generations affected
C. At least 1 diagnosed prior to age 50
D. Multiple cancers in at least one affected person

Answer 12
D. Multiple cancers in at least one affected
person

Characteristic Lynch Syndrome


HNPCC is autosomal dominant with primary feature of
right-sided colon cancers without polyposis
Altered genes are MLH1, MSH2, MSH6 and PMS2 (DNA
mismatch repair)
In women, other primary risk is endometrial cancer
Other at risk organs include stomach, small bowel, urinary
tract, ovary, pancreas and brain
Amsterdam Criteria of diagnosis of HNPCC in a family:
3 or more family members with HNPCC associated cancer, one of
whom is 1st degree relative of others
2 generations
At least 1 diagnosed prior to age 50

Question 13
You are taking care of a patient that sustained 25%
TBSA 1 week ago. His wounds have mostly been
treated with debridement and topicals and he has
been otherwise stable. He has recently developed
a metabolic acidosis after you added a new topical
to his wound care regimen. Which medication is
the most likely causative agent?

A.
B.
C.
D.

Acticoat
Silver nitrate
Mafenide
Polymixin B

Answer 13
C. Mafenide

Burn Topicals
Silver nitrate:
Broad spectrum, painless, cheap
Poor eschar penetration, may cause electrolyte imbalance

Silver sulfadiazine:
Painless, no electrolyte abnormalities, no occlusive dressing required
Little eschar penetration; misses Pseudomonas, idiosyncratic neutropenia

Mafenide:

Penetrates eschars, broad spectrum (but misses staph)


Pain and burning on application
7% have allergic reaction
May cause metabolic acidosis
Agent of choice in contaminated burns; watersoluble

Question 14
Which of the following is NOT a negative
prognostic factor in thyroid cancer?
A.
B.
C.
D.

Age over 50
Poorly differentiated histology
Female gender
Large lesion

Answer 14
C. Female gender is not a poorer prognostic
factor

Prognostic Factor Thyroid Cancer

Remember the acronym AGES:


Age (women < 50, men < 40)
Grade (poorly differentiated = bad)
Extent of disease ( extracapsular extension or
regional LN disease = bad)
Size ( > 4cm = bad)
Sex (women better than men)

Question 15
A 52 yo female underwent a total thyroidectomy for a
MNG and is found to have a 3 cm nodule with papillary
cancer on final pathology. She currently is on Cytomel
(T3) and reports feeling fairly well. You recommend
that she undergoes I-131 treatment. Which of the
following is true about management of this condition?

A.
B.
C.
D.

Patients can receive synthroid (T4) up to 1 week prior to I-131 treatment


Most T3 supplements need to be held for at least 3 weeks prior to I-131
Both of the above are true
Neither of the above are true

Answer 15
B. Most T3 supplements need to be held for at least 3
weeks prior to I-131

Preparation for I-131 scan


Patient has to be hypothyroid for the scan
Need to stop thyroid hormone before scan
Synthroid/levothyroxine (T4) stop 6 weeks prior
Cytomel/liothyronine (T3) stop 3 weeks prior

Question 16
A 31 yo female who had a successful cadaveric
renal transplant 6 months ago presents with rising
creatnine. Biopsies are performed which confirm
rejection and BK virus infection is suspected to be a
factor. Which of the following is true about this
condition?

A.
B.
C.
D.

Immunosuppression should be increased


It is an uncommon virus in the general population
It will respond well to Valtrex administration
You should decrease her MMF dosage

Answer 16
D. You should decrease her MMF dosage

BK Virus and Nephropathy


BK Virus is an important factor associated with
graft nephropathy
Prevalent in 90% of population and results in
nephropathy in 1 8% of transplant recipients
No adequate antiviral treatment
Treatment- Must immunosuppression, in
particular MMF

Question 17
A 46 yo male s/p OLTX 9 years ago presents with
fever, night sweats, and diffuse lymphadenopathy.
Biopsy of an axillary node confirms lymphoma.
Which of the following would you expect to be
true?

A. It will most likely be a Hodgkins-type disease


B. He will likely respond well to chemotherapy
C. EBV is associated with this condition
D. It is more common in renal recipients than in
heart/lung recipients

Answer 17
C. EBV is associated with this condition

Post-Transplant Lymphoma
Lymphoma is 10 100 x more common in transplant
patients than general population
Ranges from 1% incidence in kidney; 4 5% in heart/lung
patients
Especially seen in CNS
Usually NHL Bcell lymphoma related to malignant
transformation of EBV
Treatment- Reduce or withdraw immunosuppression
High dose acyclovir may be effective
Conventional chemotherapy generally not effective

Question 18
Which of the following factors are not included
in the calculation of MELD score?
A.
B.
C.
D.

Bilirubin
INR
Serum creatnine
Albumin

Answer 18
D. Albumin

MELD Score
Model for Endstage Liver Disease criteria for liver
failure
Score = 3.8 * [bilirubin (mg/dL)] + 9.6 * [Cr (mg/dL)]
+ 11.2 * [INR]
Formula predicts the risk of death in 3 months
Average score for most patients being transplanted
currently is 15
Additional points given for tumors suspected or
confirmed to be HCC

Question 19
Which of the following types of collagen is
most abundant in normal skin?
A.
B.
C.
D.

Type 1
Type 2
Type 3
Type 4

Answer 19
A. Type 1

Question 20
Which of the following types of collagen is
likely low in Ehler-Danlos syndrome?
A.
B.
C.
D.

Type 1
Type 2
Type 3
Type 4

Answer 20
C. Type 3

Question 21
About what concentration of microogranisms
does it take to impede wound healing?
A.
B.
C.
D.

1,000 organisms/ cm2


10,000
100,000 (10^5)
1,000,000

Answer 21
C. 100,000 (10^5)

Wound Strength/ Collagen


Early tensile strength due to fibrin; late due to collagen
crosslinking
Tensile strength is never equal to prewound
100,000 organisms/cm2 is enough to retard wound healing
Ultimately the ratio of type I:III collagen is 8:1 (i.e. that of
normal skin)
I- Most abundant, found in scar
II- In cartilage
III- In wound healing (low in Ehler-Danlos)
IV- In basement membrane
V - Found in cornea

Question 22
Which of the following is not true about
propothyouracil (PTU)?
A. It can be used in pregnancy
B. It prevents production of thyroid hormone but
not peripheral conversion of T3 to T4
C. It can cause agranulocytosis
D. It prevents production of thyroid hormone as
well as peripheral conversion

Answer 22
B is false. It prevents production of thyroid
hormone as well as peripheral conversion of T3
to T4

Question 23
Which of the following patients with
hyperthyroidism do not have a significant
indication to perform thyroid resection?
A. A 30 yo woman that is 20 weeks pregnant and
not fully responsive to PTU
B. A 65 yo male with Plummers disease
C. A 47 yo female with a large goiter that is
beginning to cause compressive symptoms
D. A 61 yo male that has been treated with
methimazole for 1 week with minimal result

Answer 23
D. A 61 yo male that has been treated with
methimazole for 1 week with minimal result

Indications for Thyroidectomy in


Hyperthyroidsim

Most patients with hyperthyroidism are managed with anti-thyroid meds:


PTU prevents production of thyroid hormone by gland and prevents peripheral
conversion of T4 to T3; most serious side effect is agranulocytosis
Methimazole only prevents production of thyroid hormone (which has a high relapse
rate). Should be continued for at least 6 months before trying to stop
Radioactive iodine

Surgery usually not the tx of choice, but is always an option.


Indications for surgery over other options:
1. Pregnant patient- Beta-block and then operate
2. Failed medical treatment
3. Hyperthyroidism secondary to an autonomously functioning thyroid nodule
(Plummers syndrome)- best tx is surgery and not RAI.
4. Large goiter with compressive symptoms

Question 24
A 35 yo male with a 10 year history of Crohns
presents with nausea and vomiting after meals with
increasing frequency. Upper GI and EGD reveal a
stricture in the 3rd part of his duodenum that
appears to be at least 4 cm in length. Which of the
following is the best option?

A.
B.
C.
D.

Whipple procedure
Strictureoplasty
Balloon dilation with stents if not effective
Gastrojejunostomy

Answer 24
D. Gastrojejunostomy

Duodenal Obstruction in Crohns


Assuming failed medical mgmt, gastrojejunostomy is
frequently indicated
Unlike rest of small bowel, strictureoplasty is likely
difficult in this location unless very short segment
Resection usually not possible short of Whipple
If stricture in a distal location, duodenojejunostomy
(side-to-side) might be best option

Question 25
A 42 yo man that has lived overseas, including in
different parts of Asia and Australia presents with
fever and RUQ pain. CT scan reveals a lesion
consistent with an echinococcal abscess in the right
lobe of the liver. Which of the following should be
recommended to treat this patient?

A. Right hepatectomy
B. RFA
C. 6 weeks of IV Flagyl and Cefepime
D. Removal of the cyst and administration of
mebendazole

Answer 25
D. Removal of the cyst and administration of
mebendazole

Treatment Echinococcal Abscesses


Diagnosis by indirect hemagglutination and
ELISA
Treatment is surgical with complete
cystectomy and avoiding spillage
Antiparisitic rx is with mebendazole

Question 26
A 72 yo man is following up 1 month s/p
parotidectomy for malignant disease. He notes
sweating in association with eating and the thought
of food. Which of the following is true about this
syndrome?

A.
B.
C.
D.

It is likely due to dysfunction of the facial nerve


It is usually a sign of recurrent malignancy
Surgical treatment is usually necessary
Antiperspirants should be recommended

Answer 26
D. Antiperspirants should be recommended

Treatment Freys Syndrome


Post-gustatory sweating (Freys syndrome) is
associated with parotidectomy
Patients get perspiration/flushing overlying site
of parotid gland
Auriculotemporal nerve is the culprit.
Symptomatic treatment only- usually managed
with a roll-on anti-perspirant.

Question 27
A 18 yo football player is brought in after a tackle
where he describes his leg being crushed. He is
tender over the lateral knee and leg and you note
dorsiflexion of the foot is decreased. X-ray shows
fracture of the fibular head. Which nerve is most
likely injured?

A.
B.
C.
D.

Femoral
Posterior tibial
Common peroneal
Sciatic

Answer 27
C. Common peroneal

Paralysis Common Peroneal Nerve


Nerve lies laterally below the knee and wraps
around the head of the fibula
Can be injured with trauma to fibular head or
from compression of the lateral aspect of the
knee joint
Result is foot drop with diminished dorsiflexion
of the ankle.

Question 28
Which of the following is not a strong
preoperative predictor of cardiac
complications?
A.
B.
C.
D.

JVD and gallop in a patient with CHF


MI within the last 6 months
Frequent PVCs
Need for multiple agents to control hypertension

Answer 28
D. Need for multiple agents to control hypertension

Preop Predictors Cardiac Complications


On test almost every year and widely missed
The Goldman Index is the best recognized attempt at
correlating cardiac sxs with periop complications
It assigns a numeric grade to multiple risk factors.
They range from 3-11 points, depending on the symptom.

The worst prognostic sxs are:


11- uncompensated CHF (evidenced by elevated CVP/JVD/S3 gallop)
10- recent MI (within 6 months)
7- > 5 PVC/min on EKG
7- non-sinus rhythm or PACs on EKG

Question 29
A 31 yo woman underwent a CT scan s/p MVA and
incidentally is found to have a 4 cm lesion in segment 7
of her liver. The area appears to have a central stellate
scar and the rest of the hepatic parenchyma appears
normal. Which of the following is true about this
condition?

A.
B.
C.
D.

It has a 20% chance of developing malignancy so should be resected


It has a fairly high incidence of bleeding and rupture
If a sulfur colloid scan is performed, it would appear cold
It may be observed unless it is symptomatic

Answer 29
D. It may be observed unless it is symptomatic

Focal Nodular Hyperplasia

Typically < 5cm


Usually incidental and asymptomatic
Hallmark characteristic is central stellate scar
seen on imaging
Have intact Kupffer cells so will be hot on
sulfur colloid scan
No rx needed unless symptomatic; resection
then is indicated

Question 30
A 40 yo female undergoes a sterotactic breast
biopsy and you are reviewing her pathology results.
Which of the following is associated with an
increased risk of breast cancer in either breast but
is not cancerous itself?
A.
B.
C.
D.

Atypical ductal hyperplasia (ADH)


Atypical lobular hyperplasia (ALH)
LCIS
All of the above

Answer 30
D. All of the above

Question 31
The biopsy result in this patient is consistent
with atypical ductal hyperplasia. There are no
other lesions or concerning areas in either
breast. Which of the following should be
recommended to this patient?
A.
B.
C.
D.

Wire guided lumpectomy


Wire guided lumpectomy and SLN biopsy
Chemoradiation
Observation with possible tamoxifen treatment

Answer 31
D. Observation with possible tamoxifen treatment

Benign Proliferative Diseases with Breast Cancer


Risk
3 benign proliferative lesions that increase risk of
developing breast cancer but arent themselves premalignant:
Atypical ductal hyperplasia (ADH)
Atypical lobular hyperplasia (ALH)
LCIS

Any of these found on needle biopsy warrant excision


because of risk of assoc cancer
Risk is bilateral, not just in ipsilateral breast
However, purpose of excision is to have adequate sample
and once excised, dont need negative margins
Consider chemoprevention with Tamoxifen

Question 32
A 32 yo female is found to have a right hepatic 4 cm
hemangioma found incidentally when undergoing
CT scan for trauma. Which of the following is not
true regarding hemagioma in adults?
A. May benefit from daily aspirin therapy- has been
shown to lead to reduction in size
B. This patient may be observed
C. This lesion is almost always benign
D. If symptomatic, either enucleation or segmental
resection is appropriate

Answer 32
A. May benefit from daily aspirin therapy- has been
shown to lead to reduction in size

Question 33
A 4 year old patient is found to have
thrombocytopenia and shortness of breath. During
workup, a giant hepatic hemangioma is found.
Which of the following is true about this condition?

A.
B.
C.
D.

Large hemangiomas may be observed if asymptomatic


This lesion can lead to AV shunting and heart failure
Thrombocytosis is the most common finding
This lesion may involute with prostaglandin therapy

Answer 33
B. This lesion can lead to AV shunting and heart
failure

Treatment of Liver Hemangioma

On test 3 years in a row


Common and usually incidental
Giant hemangioma = > 10cm
Diag test of choice =MRI
Therefore if CT suggestive, use MRI to confirm.

Treat only for symptoms


If incidental, leave alone
Surgical mgmt can be either enucleation or anatomic resection.
If both are choice, go with enucleation.

No need/benefit for non-surgical therapy.


Giant hemangioma in pediatric patient may present with Kasabach-Merit
syndrome:
Hepatic sequestration and thrombocytopenia
Can also lead to AV shunting and heart failure
Large hemangiomas in kids should be resected when found.

Question 34
Which of the following correctly describes a T3
colon tumor?
A. Invasion into adjacent organs/ structures
B. Lesion involving mucosa/submucosa but not
through
C. Lesion invading into but not through muscularis
propria
D. Lesion invading through muscularis propria

Answer 34
D describes T3- invasion through muscularis
propria

T3 Characteristics for Colon Cancer

Memorize T staging:
T1- mucosa/submucosa
T2- into but not through muscularis propria
T3- through muscularis propria
T4- invades other organs/structures

Question 35
A 65 yo male presents with nonbilious emesis. He
has a of H. pylori for which he declined treatment
for 6 years ago. His films are suggestive of gastric
outlet obstruction. You suspect MALT lymphoma
based on EGD. Which of the following should you
recommend?

A.
B.
C.
D.

Treatment of H. pylori
Radiation only
Gastrectomy
Chemoradiation

Answer 35
C. Gastrectomy is appropriate with
obstruction. Otherwise gastric MALT should be
treated by eradicating H. pylori

Treatment MALT of Stomach


These are treated initially with eradication of
H. pylori
If that fails-> chemo and/or XRT
If that fails, then resect.
Exception-> those presenting with symptoms
of gastric outlet obstruction, then surgery best
initial treatment

Question 36
Which of the following myeloproliferative
disorders may benefit from splenectomy?
A.
B.
C.
D.

AML
CLL
Myelofibrosis
T-cell lymphoma of HIV

Answer 36
C. Myelofibrosis

Myeloproliferative Disease Benefitting from


Splenectomy
Myelofibrosis may benefit- patients can get
extramedullary hematopoeisis in spleen
This condition is also associated with increased
incidence of splenic/portal vein thrombosis
postoperatively

Question 37
A 57 yo male presents with chest pain 2 hours after
vomiting up a large meal where he feels he drank
too much wine. His workup in the ER is significant
for a left pleural effusion. Which of the following
should you do next to confirm the diagnosis?

A.
B.
C.
D.

MRI
EGD
Esophagogram
Left thoracotomy

Answer 37
C. It is appropriate to confirm perforation with
an esophogram

Question 39
You confirm perforation and proceed to the
operating room. Which side would you
perform the thoracotomy?
A. Left
B. Right

Answer 39
A. Left

Question 40
It is now ~ 3.5 hours after he developed symptoms. You
identify a lower esophageal perforation and that there
is fairly little contamination. The tissue appears
otherwise healthy and the patient is stable. Which of
the following is the appropriate next step?
A. Esophagectomy and anastomosis
B. Creation of a spit fistula, flap coverage of perforation
C. Primary repair with placement of a G-tube and drains
D. Mediastinal washout with planned return to OR in 24 hours

Answer 41
C. Primary repair with placement of a G-tube and
drains

Boerhaaves Perforation
Classic history- chest pain, fever, respiratory distress after
emesis resulting from large meal
Diagnosis usually suspected by left pleural effusion on CXR
Confirmatory test- esophagram with H2O soluble contrast.
If that isnt a choice, then choose orally contrasted CT
Treatment If early, primary repair(left thoractomy) +/- g-tube and j-tube.
If late, then consider spit fistula and g-tube/j-tube and delayed
restoration of GI continuity
If severely ill, mediastinal washout/drainage alone can be done
initially with more definitive surgery after recovery.

Question 42
A 21 yo female presents with abdominal pain and
subsequent workup reveals numerous
hamartomatous polyps throughout her bowel as
well as mucocutaneous hyperpigmentaiton. Which
of the following is true about this condition?
A. Prophylactic colectomy should be recommended
B. The polyps are usually adenomatous though have a low
malignant potential
C. The oral lesions have a high malignant potential
D. This is associated with sex cord tumors

Answer 42
D. This is associated with sex cord tumors

Peutz-Jeghers Syndrome
Autosomal dominant disease associated with colonic
polyps
Thought to primarily represent hamartomas and
mucocutaneous hyperpigmentation (most often oral)
No significant increased risk of cancer in these
polyps, so prophylactic colectomy not warranted,
just endoscopic removal.
Syndrome is associated with unusual sex cord tumors
in women and Sertoli cell tumors in men

Question 43
Which of the following deficiencies is
associated with glucose intolerance?
A.
B.
C.
D.
E.

Zinc
Phosporous
Biotin
Chromium
Vitamin A

Answer 43
D. Chromium

Question 44
Which of the following deficiencies is
associated with cardiomyopathy?
A.
B.
C.
D.
E.

Zinc
Selenium
Biotin
Chromium
Vitamin A

Answer 44
B. Selenium

Vitamin Deficiencies
Phosphorus: weakness, paresthesias
Zinc: perioral rash, alopecia, poor wound healing, impaired
immunity, change in taste
Copper: anemia, neutropenia, pancytopenia
Iron: anemia
Linoleic acid: dermatitis, alopecia, blurred vision, paresthesias
Selenium: cardiomyopathy, weakness, alopecia
Vitamin A: night blindness, skin keratosis
Chromium: glucose intolerance (relative diabetes), peripheral
neuropathy
Biotin: alopecia, neuritis

Question 1
A 20 yo man who recently underwent an ex lap for
a bowel obstruction has had NGT output of over
1000 ml per day for 3 days. Which metabolic
derangement is most likely to be true?

A.
B.
C.
D.

Hypochloremic, hypokalemic metabolic alkalosis


Hyperchloremic, hypokalemic metabolic alkalosis
Hypochloremic, hyperkalemic metabolic alkalosis
Hyperchloremic, hyperkalemic metabolic alkalosis

Answer 1
A. This person is losing water and HCl via the NG tube.
To compensate for the water loss, the kidney reabsorbs Na in
exchange for potassium, leading to hypokalemia
The kidney also recognizes that is losing K so it exchanges K for
H, which results in a paradoxical aciduria
The end result is hypochloremic, hypokalemic metabolic
alkalosis with paradoxical aciduria.
Tx- volume resuscitation with K+ containing solutions. This can
be D10 NS with 10 meq/L KCL in children or D5 NS with 20
meq/L KCL in adults

Question 2 (4 part question)


Sensitivity of a test reflects:
A.
B.
C.
D.

Ability to detect disease


Ability to say no disease is present
Rejecting the null hypothesis incorrectly
Accepting the null hypothesis when it is false

Question 3
Specificity of a test reflects:
A.
B.
C.
D.

Ability to detect disease


Ability to say no disease is present
Rejecting the null hypothesis incorrectly
Accepting the null hypothesis when it is false

Question 4
Type 1 error reflects:
A.
B.
C.
D.

Ability to detect disease


Ability to say no disease is present
Rejecting the null hypothesis incorrectly
Accepting the null hypothesis when it is false

Question 5
Type II error reflects:
A.
B.
C.
D.

Ability to detect disease


Ability to say no disease is present
Rejecting the null hypothesis incorrectly
Accepting the null hypothesis when it is false

Answers 2-5
2- a; 3-b; 4-c; 5-d
Sensitivity reflects ability to detect disease.
Is equal to true positives / (true positives + false negatives)
With high sensitivity a negative result means the pt is very unlikely to have the
disease

Specificity reflects ability to state that no disease is


present
Is equal to true negatives / (true negative + false positive)
With high specificity a positive result means that the patient is very likely to
have the disease

Answers 2-5
Type 1 errors reject the null hypothesis
incorrectly (thinking there is a correlation
when there really isnt one)
Type II errors accept the null hypothesis
incorrectly (you didnt find a correlation but
one actually exists
The most common reason for a type II error is
low sample size

Question 6
Spontaneous closure is least likely in fistulae
originating from:
A.
B.
C.
D.
E.

Colon
Esophagus
Pancreas
Stomach
Small intestine

Answer 6
D. Fistulae involving the stomach are the least
likely to close

Question 7
Biliary-enteric fistula most commonly connect
the:
A.
B.
C.
D.
E.

Gallbladder and ileum


Gallbladder and duodenum
Common bile duct and jejunum
Gallbladder and jejunum
Common bile duct and ileum

Answer 7
B. Biliary-enteric fistula most commonly
connect the gallbladder and duodenum. This is
usually caused by severe cholecystitis with
abscess and/or perforation and subsequent
erosion into the duodenal wall
A large stone may erode into the duodenum
and subsequently cause gallstone ileus

Question 8
Meckels diverticulum:
A. Is a false diverticulum
B. Is asymptomatic in most cases
C. Commonly presents as gastrointestinal bleeding
in adults
D. Commonly presents with intestinal obstruction in
children
E. Is found in approximately 5-10% of the
population

Answer 8

B. Meckels diverticulum are asymptomatic in most cases.


They are true congenital diverticula
Vestigal remnant of omphalomesenteric duct and most frequent
malformation of the GI tract
2% of population have them; males more likely to be symptomatic
Rule of 2's:

2% (of the population)


2 feet (from the ileocecal valve)
2 inches (in length)
2% are symptomatic
2 types of common ectopic tissue (gastric and pancreatic)
Most common age at clinical presentation is 2
Males are 2 times as likely to be affected.

It can also be present as an indirect hernia, typically on the right side,


where it is known as a Hernia of Littre

Question 9
The most common site of gastrointestinal
lymphoma is :
A.
B.
C.
D.
E.

Small intestine
Stomach
Colon
Duodenum
Appendix

Answer 9
B. The stomach is the most common site of
gastrointestinal lymphomas

Question 10
Rightward shift of oxyhemoglobin dissociation
curve occurs with:
A. Hypothermia
B. Acidosis
C. Decrease in 2,3-diphosphoglycerate (DPG)
D. Hypocapnia
E. Methemoglobinemia

Answer 10
B. Acidosis causes a right shift of the
oxyhemoglobin dissociation curve. All of the
other listed factors shift it to the left.

Question 11
Phosphorous:
A.
B.
C.
D.

Is a major extracellular anion


Is passively absorbed from the GI tract
Deficiency may result in insulin resistance
Deficiency is rare in hospitalized patients

Answer 11
C. A patient that is suffering from
hypophosphatemia may demonstrate insulin
resistance.

Question 12
Hemangioma of the liver:
A. Is the most common benign hepatic tumor
B. Is diagnosed with percutaneous needle biopsy
C. Is associated with alpha fetoprotein level
D. Should be resected as soon as diagnosed

Answer 12
A. Hemangiomas are the most common benign tumor of the
liver.
They are more common in women
Rupture is rare. They are usually asymptomatic
Avoid biopsy-> this risks hemorrhage
Peripheral to central enhancement on CT
Can operate or embolize if symptomatic
Steroids can be used to treat masses that are unresectable
Kasabach-Merritt syndrome- rare complication of
hemangioma.
Mass causes consumptive coagulopathy and CHF
This is more common in children

Question 13
Leiomyoma of the esophagus:
A. Commonly presents with dysphagia
B. Is more common in females
C. Is usually multiple
D. Is usually diagnosed with endoscopic biopsy
E. Is usually located in the lower 1/3 of the
esophagus

Answer 13
E. Leiomyomas are usually located in the distal 1/3
of the esophagus
They are the most common benign tumor of the
esophagus
Dx- endoscopy, esophogram to r/o cancer
Symptoms- dysphagia, pain
Do not biopsy- scarring can make subsequent
resection more difficult
Operate when they are >5cm or symptomatic.
Enucleation via thoracotomy is appropriate

Question 14
The best operative approach to a choledochal
cyst is:
A.
B.
C.
D.

Cystoduodenostomy
Cystojejunostomy
Roux-en-Y cystojejunostomy
Cyst excision and hepaticojejunostomy

Answer 14
D. Cyst excision with reconstruction via a
hepaticojejunostomy is the preferred treatment.
They need to be resected because of malignant
potential (15%).
They are common in females and Asians
90% are extrahepatic
Most are type 1- fusiform dilation of the CBD
Possibly caused by angle of insertion of the duct
which leads to reflux of pancreatic enzymes during
development

Answer 14

Question 15
Li-Fraumeni syndrome shows increased
incidence of:
A.
B.
C.
D.
E.

Colon cancer
Ovarian cancer
Lung cancer
Breast cancer
Pancreatic cancer

Answer 15
D. Breast cancer
Li-Fraumeni syndrome is due to a defect in the
p53 gene
Patients are at risk for:
Childhood sarcomas
Breast cancer
Brain tumors
Leukemia
Adrenal cancer

Question 16
During cell cycle, DNA replication occurs in:
A.
B.
C.
D.

G1 phase
G2 phase
S phase
M phase

Answer 16
C. DNA replication occurs during the S
(synthetic) phase

Question 17
Small bowel obstruction in an elderly female
without external hernia or previous surgery is
most likely caused by:
A.
B.
C.
D.
E.

Small bowel neoplasm


Volvulus
Gallstone ileus
Abdominal abscess
Obturator hernia

Answer 17
C. Gallstone ileus would be the most likely
cause of SBO in an elderly patient without a
clear hernia

Question 18
Malignant small bowel neoplasms most
commonly present with:
A. Weight loss
B. Abdominal pain
C. GI bleeding
D. Jaundice
E. Bowel perforation

Answer 18
A. Of the choices, weight loss is the most
common presenting symptom of small bowel
malignancies.
Small bowel adenocarcinomas are rare
High proportion are in the duodenum
When diagnosed, they need to be resected
with lymphadenectomy

Question 19
A 50 yo man undergoes LAR followed by
chemo-XRT. 6 weeks after this is complete he
has severe proctitis and bleeding per rectum.
He has required several transfusions for this
problem. The best therapy for this pt is:
A.
B.
C.
D.

Angio-embolization of the rectal arteries


Abdominoperineal resection
Formalin fixation of the rectum
Antibiotics

Answer 19
C. Radiation proctitis leading to severe
bleeding is best treated by formalin fixation of
the rectum

Question 20
You perform laparoscopy for presumed
appendicitis on a 25 yo man. He has terminal
ileitis but the cecum looks normal. You should:
A.
B.
C.
D.

Perform an appendectomy
Close
Place a drain
Perform ileal resection

Answer 20
A. Pts with presumed appendicitis but instead
are found to have terminal ileitis should
undergo appendectomy so that confusion of
ileitis and appendicitis will not occur in the
future.
If the cecum is involved with the inflammation,
then leave the appendix- if you perform an
appendectomy in this situation you have a high
chance for a leak

Question 1
The microtubule organizing center of the cell is
called a:
A.
B.
C.
D.
E.

Centrosome
Lysosome
Codon
Kinetochore
Leucine zipper

Answer 1
A. The centrosome is the microtubule
organizing center.
Other cell organelles and structures:
Rough endoplasmic reticulum- synthesizes
proteins that are exported (increased in
pancreatic acinar cells)
Smooth endoplasmic reticulum- lipid/ steroid
synthesis, detoxifies drugs (increased in liver
and adrenal cortex)

Answer 1
Golgi apparatus- modifies proteins with
carbohydrates; proteins are then transported
to the cellular membrane, are secreted, or are
targeted to lysosomes
Lysosomes- have digestive enzymes that
degrade engulfed particles and worn out
organelles
Phagosome- Engulfed large particle; these fuse
with lysosomes

Answer 1
Endosome- engulfed small particles; these fuse
with lysosome
Protein kinase C- activated by calcium and
diacylglycerol (DAG)
Phosphorylates other enzymes and proteins

Protein kinase A- activated by cAMP


Phosphorylates other enzymes and proteins

Myosin- thick filaments, use ATP to slide along


actin to cause muscle contraction

Answer 1
Actin- thin filaments that interact with myosin above
Intermediate filaments- keratin (hair/ nails), desmin
(muscle), vimentin (fibroblasts)
Microtubules form specialized cellular structures such
as cilia, neuronal axons, and mitotic spindles.
Also involved in the transport of organelles in the cell (form a latticework inside
the cell)
Centriole- a specialized microtubule involved in cell division (form spindle fibers
which pull chromosome apart)

Question 2
Treatment for intractable abdominal pain from
chronic pancreatitis with a normal pancreatic
duct is usually:
A.
B.
C.
D.

Percutaneous drainage
Resection
Lateral pancreatico-jejunostomy
Nonoperative management

Answer 2
B. Resection may be indicated in pts with
chronic intractable abdominal pain due to
pancreatitis

Question 3
Treatment for intractable abdominal pain in a
patient with chronic pancreatitis and a dilated
pancreatic duct is usually:
A.
B.
C.
D.

Percutaneous drainage
Resection
Lateral pancreatico-jejunostomy
Nonoperative management

Answer 3
C. Patients with intractable pain due to chronic
pancreatitis and a dilated duct may benefit
from lateral pancreaticojejunostomy (Puestow
procedure ) if the duct is greater than 8 mm

Question 4
You operate on a 25 yo man with presumed
appendicitis, send a frozen section because
there is a mass at the tip of the appendix, &
path comes back as a 2.5 cm carcinoid. The
most appropriate step in his management is:
A.
B.
C.
D.

Right hemicolectomy
Close
Post-op XRT
Chemotherapy post-op

Answer 4
A. Appendectomy is appropriate management
for carcinoid localized to the appendix, as long
as they are less than 2 cm, not at the base, and
there is no evidence for metastatic disease.
If the above criteria are not met, perform a
right hemicolectomy

Question 5
A newborn is found to be in severe respiratory
distress immediately following birth. CXR shows
loops of bowel filling the left chest. All of the
following are true except:
A. Pts with this problem have about a 50% overall survival
B. The incidence of this problem is higher on the left than
the right
C. Both lungs are often dysfunctional
D. Pts with this problem require immediate repair after
birth

Answer 5
D. The above scenario is consistent with a
congenital diaphragmatic hernia. This is
associated with a 50% survival overall.
It is more common on the left (80%) and both
lungs are usually dysfunctional- one from
compression by the bowel and the dysfunction
in the contralateral lung is not completely
understood

Answer 5
Pulmonary hypertenstion is frequent in this
population
Current treatment of choice in these children is
stabilization with high frequency ventilation,
ECMO, and/or inhaled NO before repair.
Delayed repair after the patient is better able to
tolerate surgery is thought to improve survival

Question 6
57 yo man has a 3 cm mass in the RLL on chest CT.
Biopsy shows the mass is an adenocarcinoma. You
perform mediastinoscopy and a right paratracheal
lymph node is positive for cancer. The next best
step is:
A. Right lower lobe resection only
B. Right lower love resection and mediastinal lymph node
dissection
C. Pneumonectomy and mediastinal lymph node
dissection
D. Chemo & radiation

Answer 6
D. Positive paratracheal nodes identified on
mediastinoscopy are considered N2 disease
and the pt is unresectable. Therefore, chemo/
XRT is the best answer.
Some protocols would have the pt undergo
chemo/ XRT and if the pt had a great response,
resection of the primary along with
mediastinal lymph node dissection would be
offered.

Question 7
A 50 yo woman comes in with a chief
complaint of a nodule in her neck. You get a
TSH and a T3, both of which are normal. The
most appropriate management is:
A.
B.
C.
D.

Thyroid lobectomy
Total thyroidectomy
Ultrasound and FNA
Neck MRI

Answer 7
C. After getting baseline TFTs you should get
an ultrasound and an FNA

Question 8
Ultrasound of the nodule reveals a 1.2 cm
mass. The pathology comes back as follicular
cells. The next most appropriate step is:
A.
B.
C.
D.

Thyroid lobectomy
Nothing
Neck MRI
Neck CT

Answer 8
A. Follicular cells on FNA end up being a
follicular cell cancer in 10% of cases. Therefore
you need to do a lobectomy to get a definitive
diagnosis

Question 9
You take the above patient to the OR for a
lobectomy. Pathology shows a 1.9 cm follicular
carcinoma. The next best step is:
A.
B.
C.
D.

Completion total thyroidecotmy


Close
Post-op chemotherapy
Post-op radiation

Answer 9
A. Thyroid cancer with a size > 1cm requires
total thyroidecotmy
Indications for total thyroidectomy include:
Tumor > 1 cm
Extra-thyroidal disease (capsular invasion, clinical or
+ nodal disease, or metastases)
Multicentric disease
History of XRT

Question 10
A 10 yo boy has a cyst and a cyst tract near the
angle of his mandible. The cyst has had
recurrent infections. This cyst most likely
connects to the:
A.
B.
C.
D.

External auditory canal


Tonsillar pillar
Nasal septum
Thoracic duct

Answer 10
A. Type 1 branchial cleft cysts extend from the
angle of the mandible to the external auditory
canal.

Question 11
A 10 yo boy presents with a cyst in his lateral neck
medial to the anterior border of the
sternocleidomastiod. This cyst most likely connects
to the:
A.
B.
C.
D.

External auditory canal


Tonsillar pillar
Nasal septum
Thoracic duct

Answer 11
B. Type II branchial cleft cysts extend from the
anterior border of the SCM though the carotid
bifurcation to the tonsillar pillar.

Question 12
The monoclonal antibody that specifically
blocks CD3 molecule on T cells is:
A.
B.
C.
D.

OKT 3
ATGAM
Thymoglobulin
Zenepax

Answer 12
A. OKT3 binds the CD3 molecule on
lymphocytes. This inhibits formation of the Tcell receptor complex and causes opsonization
of the T cell. This drug can be used to treat
rejection that is refractory to pulse steroids.

Question 13
The monoclonal antibody that specifically
blocks IL-2 receptors is:
A.
B.
C.
D.

OKT 3
ATGAM
Thymoglobulin
Zenepax

Answer 13
D. Zenepax is an antibody against human IL-2
receptors. It is often used with induction
therapy immediately after transplantation

Question 14
All of the following are true of Pagets disease
of the breast except:
A. It describes scaly lesions on the nipple that when
biopsied demonstrate Pagets cells
B. Pts with this disease have underlying DCIS or
ductal carcinoma
C. Modified radical mastectomy is appropriate for
pts found to have carcinoma
D. Modified radical mastectomy must be done in
these patients who are found to have DCIS

Answer 14
D. All of the options are true of Pagets disease
of the breast. If only DCIS is found it is
appropriate to offer simple (not modified
radical) mastectomy

Question 15
All of the following are true of familial hypocalcemic
hypocalciuria except:
A. Patients usually have a calcium in the 9-11 range and
decreased levels of urine calcium
B. Treatment requires parathyroidectomy
C. The disease is caused by a defect in the PTH receptor in
the distal convoluted tubule of the kidney that causes
increased resorption of calcium
D. Parathyroid hormone levels in these patients is normal

Answer 15
B is incorrect. The calcium levels in these
patients is usually not that high; they do not
require parathyroidectomy

Question 16
A 4 yo female is brought to see you because of a
painful limp. She has no other diagnosed past
medical history though there is a strong family
history for forming blood clots. X-ray shows
flattening of the femoral head. Which of the
following is most likely the cause of her limp?

A.
B.
C.
D.

Osgood-Schlatter disease
Legg-Calve-Perthes disease
Slipped capital femoral epiphysis
Congenital dislocation of the hip

Answer 16
C. This most likely is Legg-Calve Perthes disease.
LCP is characterized by:

AVN of the femoral head


Usually appears in children older than 2 yrs
Can result from a hypercoagulable state
10% are bilateral
Painful gait limp
X-ray- flattening of the femoral head
Tx- maintain ROM with limited exercise
Surgery if femoral head not covered by acetabulum
Femoral head will remodel without sequlae

Question 17
A 12 yo male that is fairly overweight presents
with painful gait. X-ray shows widening and
irregularity of the epiphyseal plate. The most
likely diagnosis is:
A.
B.
C.
D.

Osgood-Schlatter disease
Legg-Calve-Perthes disease
Slipped capital femoral epiphysis
Congenital dislocation of the hip

Answer 17
C. This is most likely a slipped capital femoral
epiphysis. In addition to painful gait in a 10-13
yo male and the above stated x-ray findings,,
this disease is also characterized by:
Increased AVN risk of femoral head
Is treated by surgical pinning

Answer 17
Regarding the other 2 listed diseases some key
points include,
Congenital hip dislocation:

1.5 per 1,000 births, more common in Caucasians


8 times more common females
The left hip is twice as often involved
25% have bilateral involvement
Limited abduction of flexed hip, shortening of leg
Tx- Pavlik harness- keeps legs abducted and femoral head
in acetabulum

Answer 17
Osgood-Schlatter disease
Tibial tubercle apophysitis
Traction injury from the quadriceps
Most common in adolescents aged 13-15
Pain in front of the knee
Irregular shape or fragmenting of tibial tubercle seen on
x-ray
Tx either with rest for mild symptoms or cast x 6 weeks
for more severe symptoms

Question 18
You are performing a Whipple on a patient with a
pancreatic head mass. You attempt to pass your
finger behind the pancreas from below and get a large
amount of blood return when you remove it. You
place pressure on the neck of pancreas to tamponade
the bleeding which seems to control it. You most
likely have injured:
A. Aorta
B. IVC
C. Celiac artery
D. SMV

Answer 18
D. The SMV lies directly behind the neck of the
pancreas and is the most likely vessel injured
when attempting to get behind the neck of the
pancreas

Question 19
The following are indications to resect MCNs
A. Asymptomatic masses > 4 cm in diameter
B. Symptomatic masses of any size
C. Small asymptomatic masses with internal
excresences or solid components
D. All of the above

Answer 19
D. All of the above are indications to resect
MCNs

Question 20
All of the following are true of mucinous cystic
neoplasms except:
A.High CEA in cyst fluid
B.Most commonly seen in middle aged women
C. CA 19-9 often elevated
D. Communicate with pancreatic duct

Answer 20
D. Mucinous cystic neoplasms do not
communicate with the pancreatic duct

Question 1
A 15 yo boy is struck in the abdomen when he goes
over the handlebars on his bike. You get a CT abdomen
with contrast but cannot identify any abnormality. He
vomits twice the next day after clears are attempted.
He also is more distended. The most appropriate
management is:
A. Repeat the CT
B. Zofran
C. Exploratory laparotomy

D. DPL
(Answers after ? #2)

Question 2
You repeat the childs CT and identify a
hematoma in the 3rd portion of the duodenum.
There is no extravasation of contrast noted.
The next best step is:
A.
B.
C.
D.

Exploratory laparotomy, evacuate the clot


Percutaneous evacuation of the clot
NGT and TPN
Whipple

Answers 1&2
A & C. If a trauma patient has a chance in clinical status or fails
to progress, you should consider repeating imaging studies.
Pancreatic, small bowel, duodenal, an diaphragmatic injuries
can present late.
Duodenal hematomas can occur as a delayed presentation of
trauma. The 1st step is to make the diagnosis. An UGI probably
is the best test though these injuries can also be seen on CT
with IV & PO contrast.

Next you need to assess whether there is a contrast leak. If so,


the patient needs an operation. If not, manage w/ TPN/ NPO
for 3 weeks. Most resolve with conservative mgmt.

Question 3
Peripheral nerves regenerate at:
A.
B.
C.
D.

0.01 mm/day
0.1 mm/day
1 mm/day
5 mm/day

Answer 3
C. Nerves regenerate at 1 mm/day

Question 4
The most important stimulator of TNF alpha
release is:
A.
B.
C.
D.

IL-1
IL-6
Lipopolysaccharide
Shock

Answer 4
C. The most potent stimulator for TNF-alpha is
lipopolysaccharide

Question 5
In patients that have undergone total
proctocolectomy for FAP the most common
cause of death is:
A.
B.
C.
D.

Liver cancer
Adrenal cancer
Duodenal cancer
Small bowel cancer

Answer 5
C. Patients with FAP can also develop
duodenal polyps which should be followed in
removed as they have malignant potential.

Question 6
A 55 yo woman in the ICU has a PMH of a fib & PVD. She
recently developed pneumonia and is intubated. Her
anticoagulation has been held for a week. She now is on
levophed and vasopressin from presumed sepsis. You get
called because her abdomen is now distended and there
are heme + stools. She is tender but does not have gross
peritoneal signs. Lactic acid is 6, WBC = 20, BP 100/60 &
HR= 90. The most appropriate mgmt for this patient now
is:

A.
B.
C.
D.

Laparotomy
Peritoneal tap
CT scan
Angiogram

Answer 6
D. This pt may have mesenteric ischemia. Going to CT or
the OR is not altogether unreasonable, but since she is
stable, angiogram is the most appropriate step.
If the angio shows only constricted vessels (NOMI or nonocclusive
mesenteric ischemia) then intra-arterial infusion of papaverine
can be started. This can improve blood flow to the bowel.
If an embolus is found, she needs to go for an embolectomy
If this is a thrombosis, thrombolysis with TPA can he attempted.
In any of the above cases, if the exam becomes worrisome, go
directly to the OR

Question 7
The angiogram ordered in the previous pt shows a
meniscus sign 5 cm down from the SMA takeoff.
The jejunal branches proximal to this are normal.
This patient most likely has
A.
B.
C.
D.

NOMI
SMA thrombus
SMA embolus
Venous thrombosis

Answer 7
C. A meniscus sign and sparing of the proximal
jejunal branches is most consistent with SMA
embolism

Question 8
Management of the above patient requires:
A. Continued ICU care
B. Direct papaverine injection
C. SMA embolectomy, resection of necrotic bowel,
2nd look operation
D. SMA thrombectomy, SMA bypass, resect necrotic
bowel, 2nd look tomorrow

Answer 8
C. This pt should go to the OR for
embolectomy, resection of necrotic bowel, and
a 2nd look procedure the next day.

Question 9
All of the following are true about ovarian cancer
except;
A. Ovarian cancer with peritoneal mets is usually treated
primarily with total abdominal hysterectomy, bilateral
oophorectomy, resection of peritoneal mets, and
omentectomy
B. Debulking therapy can be effective in these patients
C. Risk factors include nulliparity, early menarche, and late
menopause
D. Risk factors include oral contraceptive pills

Answer 9
D. The use of OCPs lowers the risk of ovarian
cancer The rest of the answers are true.
Ovarian cancers is one malignancy where
debulking can be effective

Question 10
Which of the following statements about von
Willebrand factor is correct?
A. Primarily made by the liver
B. Stored and released by lymphocytes
C. Critical for platelets to adhere to normal
endothelium
D. Binds to platelet-specific receptors
E. Catalyzes the activation of factor V.

Answer 10
D. vWF binds to platelet specific receptors
VW disease is the most common congenital bleeding
disorder.
Type I and II are autosomal dominant; type III is
autosomal recessive
vWF links GpIb receptor on platelets to collagen
Diseased pts have a normal PT, PTT may be normal
or prolonged
Bleeding time is prolonged

Answer 10
Risocetin test is used for diagnosis
Type 1 is most common (70%); symptoms are
usually mild
Type III causes the most severe bleeding
In Type I and III there is a reduced amount of
circulating vWF.
Tx- cryoprecipitate, DDAVP, conjugated estrogens

Type II there is a defect in the vWF molecule itselfpt has it but it doesnt work well
Tx; cryoprecipitate

Answer 10
Remember, cryoprecipate has the highest
concentration of vWF, VIII, and is used to treat
both VWB disease and hemophilia A (factor VIII
deficiency); it also contains fibrinogen
FFP- has high levels of all factors including V and
VIII, protein C & S, and AT-III
DDAVP and conjugated estrogens cause the
release of VIII and vWF from endothelium

Question 11
Delayed type hypersensitivity as observed in the
cutaneous response to tuberculin is mediated by
which of the following?

A. IgD to tuberculoprotein
B. IgM antibody to cell wall antigen
C. NK lymphocytes activated with interleukin-2 (IL-2)
CD4+ T-lymphocytes sensitized to mycobacterial antigens
Macrophages expressing the T-cell receptor alpha beta

Answer 11
D. CD4+ t-lymphocytes sensitized to
mycobacterial antigens. Please review the
chart on the next slide of hypersensitivity
reactions.

Answer 11
Type

Description

Examples

Immediate hypersensitivity (allergic)


IgE mediated; mast & basophils release
histamine, serotonin, &bradykinin in response
to release of protein from eosinophils, which
have IgE receptors for the antigen

Bee stings, peanuts, hay


fever

IgG or IgM reacts w/ cell bound antigen

ABO & Rh incompatibility,


Graves Disease,
Myasthenia gravis, ITP

Immune complex deposition

Serum sickness,
Rheumatoid arthritis, SLE

Delayed type hypersensitivity

TB skin test, contact

Question 12
All of the following are nutritionally essential
except:
A.
B.
C.
D.
E.

Proline
Leucine
Tryptophan
Threonine
Linolenic and alpha linoleic acid

Answer 12
A. Proline
Protein digestion begins in the stomach with
pepsin, then trypsin, chymotrypsin, and
carboxypeptidase
Trypsinogen released by the pancreas and is
activated by enterokinase in the duodenum
Other pancreatic protein enzymes are then activated by
trypsin
Trypsin can also autoactivate other trypsinogen molecules

Answer 12
Protein is broken down to amino acids, dipeptides,
and tripeptides by proteases
These are then absorbed by secondary active
transport and released as free amino acids into the
portal vein
Try to limit protein intake in pts with liver and/ or
renal failure to avoid ammonia buildup and possibly
worsen encephalopathy
Nonessential amino acids are the ones that begin
with A or G as well as proline and serine

Answer 12
Branched chain amino acids- leucine,
isoleucine, valine (LIV)
Metabolized in muscle
Possibly important in pts with liver failure
Are all essential amino acids

Question 13
A 55 yo man is undergoing a right hemicolectomy
for colon cancer. When you enter the abdomen
you notice that the tumor is directly invading into
the liver about 0.5 cm based on intraop ultrasound.
The best management would be:
A. Right hemicolectomy only and leave the liver
component
B. En bloc resection of the right colon and the wedge of
liver with the cancer
C. Wedge the liver lesion out only
D. Close

Answer 13
B. Direct invasion of another structure by
colon cancer requires en bloc resection if
possible.

Question 14
A 35 yo woman presents with tinnitus and
hearing loss. A MRI of the head shows a tumor
at the cerebello-pontine angle. The most likely
diagnosis is:
A.
B.
C.
D.

Glioma
Glioma multiforme
Neuroma
Medulloblastoma

Answer 14
C. Unsteadiness, tinnitus, and heraring loss are
classic sx of an acoustic neuroma. A tumor at
the cerebello-pontine angle almost ensures the
diagnosis

Question 15
The most important step in the treatment of a
pt with Zenkers diverticulum is:
A. Resection of the diverticulum
B. Division of the superior laryngeal constrictor
muscles
C. Esophagectomy
D. Division of the cricopharyngeus muscle

Answer 15
D. The most important step in treating a Zenkers
diverticulum is performing a cricopharyngomyotomy.
The anatomic problem is the failure of the upper
esophageal sphincter to relax during swallowing
The diverticulum is usually resected but in some
situations when the diverticulum is too hard to
removed it may be suspended upward such that it
drains into the esophagus

Question 16
A 24 yo college pitcher comes to the emergency
room with acute pain and swelling in his right arm.
An ultrasound shows a clot in the subclavian vein.
The next most appropriate step is;

A. Thrombolytic therapy
B. Catheter embolectomy
C. Resection of the 1st rib
D. Resection of the subclavian vein and reconstruction
with a 10 mm Gore graft

Answer 16
A. Paget von Schrotters disease is acute thrombosis
of the subclavian vein. Classically, it presents in
pitchers or patients who have a lot of strenuous
repetitive motion in that arm.
The initial treatment is thrombolytic therapy to open
up the vein
It is a little controversial what to do next. Usually the
1st rib is resected during the same hospital stay in
order to not have a repeat episode

Question 17
A 75 yo woman falls while getting up from her
wheelchair and experiences numbness in her index
finger and thumb as well as difficulty moving her
thumb.. She has a history of rheumatoid arthritis. She
has tenderness on the dorsal aspect of her wrist and at
the base of her thumb. Her findings are suggestive of
which diagnosis?

A.
B.
C.
D.
E.

Scaphoid fracture
Lunate dislocation
Bennet fracture
Colles fracture
Smith fracture

Answer 17
D. Colles fracture
A Colles fx is characterized by posterior angulation
and displacemennt of the distal radius, with or
without avulsion of the ulnar styloid. There can
also be a proximal shift of the distal radial fragment
Post menopausal women with osteoporosis are at
higher risk
RA and osteoarthritis increase the risk of this fx and
make management more difficult

Answer 17
Dinner fork appearance when viewed in
pronation
Reduce fracture manually and place in a dorsal
plaster splint
Follow with serial x-rays to assess reduction
If reduction is lost after swelling subsides, may
need repair closed reduction or ORIF.

Answer 17
The scaphoid is most commonly fractured bone in
the wrist
Occurs when the palmar surface of the hand is abducted
and stops the fall
Symptoms- tenderness in snuffbox, tubercle of scaphoid
Fractures at the narrowest point; if untreated, proximal
part develops avascular necrosis
Tx- thumb spica case or screw fixation

Answer 17
Lunate/ perilunate dislocations are rare
Usually from high-force injury
Significant ligamentous injury
Dislocation compresses median nerve in carpal
tunnel
Tx- can be open or closed reduction

Answer 17
Smith fracture Distal radius fracture with fragment angulated
anteriorly
Unstable; requires fixation

Bennet fracture
Carpo-metacarpal fracture/ dislocation of the
thumb.
Tx- reduction and internal fixation + cast for 4-6 wks

Question 18
Which of the following about the scalenus
muscle is true?
A. The anterior body inserts into the first rib
posterior to the subclavian vein
B. The middle body inserts into the first rib anterior
to the brachial plexus
C. The phrenic nerve crosses the anterior body from
the medial to lateral aspect
D. The muscle originates from the transverse
processes of C1 to C6

Answer 18
A. The anterior body inserts on the first rib posterior
to the subclavian vein
The scalene muscles include the anterior, middle,
and posterior body
They originate from the transverse processes of C2C6
Anterior body inserts onto the first rib between the
subclavian vein and artery + brachial plexus
The middle body inserts on the first rib posterior to
the subclavian artery & brachial plexus

Answer 18
The posterior body inserts onto the second rib
posterior to the subclavian artery and brachial
plexus.
The phrenic nerve crosses the anterior body from
lateral to medial
Thoracic duct inserts into the junction of the jugular
and subclavian veins anterior to the anterior body
of the muscle on the left side
Anterior body may need to be resected in the
treatment of thoracic outlet syndrome as it can
compress on local neurovascular structures

Question 19
Which of the following is characteristic of MEN
2a?
A. Parathyroid hyperplasia, pancreatic tumor,
pituitary tumor
B. Parathyroid hyperplasia, medullary ca of thyroid,
pheochromocytoma
C. Medullary ca of thyroid, pheochromocytoma,
marfanoid habitus, mucosal neuroma
D. Von Recklinghausen's Disease

Answer 19
B is characteristic of MEN 2a
MEN I
Parathyroid hyperplasia, pancreatic tumor (gastrinoma
most common) , pituitary tumor (prolactinoma most
common)
Insulinoma most common overall endocrine tumor (not in
MEN pts, however)
Parathyroid hyperplasia is first part to become
asymptomatic
Pancreatic tumors lead to morbidity and mortality
Need to correct parathyroid disease first

Answer 19
MEN 2a
Parathyroid hyperplasia, medullary ca of thyroid,
pheochromocytoma
Medullary ca is first part to be symptomatic due to
diarrhea from calcitonin
Major morbidity of syndrome is medullary ca.
Pheo needs to be treated first, though they are
usually benign

Answer 19
MEN 2b
Medullary ca of thyroid, pheochromocytoma,
marfanoid habitus, mucosal neuromas
First part to become symptomatic- medullary ca
Major morbidity is from medullary ca
Pheo needs to be treated first
Medullary ca from MEN 2b is more aggressive than
2a

Question 20
You are working a patient up for a newly diagnosed lung
mass who is otherwise asymptomatic. On CT of the
chest, you note a descending aortic dissection. Which
of the following is not true?
A. Dissections occur in the medial layer of the vessel wall
B. Symptomatic patients may demonstrate searing chest pain and unequal
pulses or BP of upper extremities
C. All ascending dissections need operative management
D. All descending dissections need operative management

Answer 20
Dissections can be classified via the Stanford or
Debakey classification
Stanford A- any ascending involvement; B- descending
involvement only
Debakey Type 1- Ascending and descending
Type 2- Ascending only
Type 3- Descending only

Most dissections start in the ascending aorta

Answer 20
Can mimic MI
Sx- searing chest pain, may have unequal pulses in
upper extremities
95% have severe hypertension
Other risk factors- Marfans, previous coarcation
repair, atherosclerosis, infection, syphilis
CXR may show wide mediastinum
Dx with CT chest + contrast
Dissection occurs in media layer of vessel

Answer 20
70% have aortic insufficiency (aortic valve cusp
sheared off)
Coronary arteries can become occluded
Cardiac failure from aortic insufficiency or
tamponade usually causes death
If possible, tx medically with Nipride and beta
blockers
Postop complications- MI, renal failure, paraplegia
Paraplegia is caused by ischemia due to intercostal artery
occlusion and artery of Adamkiewicz

Answer 20
Surgery aims to obliterate false lumen and
place graft
Operate on all ascending dissections
Operate on descending dissections with visceral,
renal, or leg ishcemia, persistent pain, or large size
(vessel dilatation)
Need to follow these pts lifetime with serial CTs30% eventually develop aneurysm that will require
surgery

Question 1
Which of the following about coagulation is
true?
A:
B.
C.
D.

PT is not affected by liver synthetic function.


PTT would be abnormal in factor VII deficiency
PTT would be unaffected by factor VII deficiency
PTT does not measure fibrinogen

Answer 1
C. PTT would be unaffected by factor VII deficiency.
PT measures II, V, VII, X, and fibrinogen. It is the
best test for liver synthetic function
PTT measures most factors including fibrinogen
except VII and XIII so it does not pick up factor VII
deficiency
PTT of 60-90 is desirable for anticoagulation
ACT= activated clotting time; need value of 150-200
for routine anticoagulation

Question 2
A patient that you are treating for a lower GI bleed develops fever
soon after blood transfusion is initiated. He is otherwise stable. All
of the following are true of this pts most likely condition except:
A. It is usually caused by a recipient antibody to the donors WBC
B. You should treat this by discontinuing the transfusion and giving
the patient blood that is filtered for WBCs in the future
C. It is usually due to IgG against IgA in an IgA deficient recipient
D. It is the most common transfusion reaction

Answer 2
C is false
This most likely is a febrile nonhemolytic
transfusion reaction, which is the most
common reaction
Usually due to recipient Ab to donor WBC
Appropriate treatment includes discontinuing the
transfusion and giving the pt blood filtered for WBC
in the future

Answer 2
Review of other transfusion reactions:
TRALI (transfusion related lung injury)
Rare; caused by antibodies to the recipients WBC
Causes clots in pulmonary capillaries
Anaphylaxis- Bronchospasm, urticaria, hypotension
Caused by IgG against IgA in IgA-deficient recipient
Tx- fluids, lasix, benadryl, pressors, epinephrine, steroids
Urticaria- Usually nonhemolytic
Reaction against plasma proteins or IgA in transfused blood
Tx- benadryl, supportive

Question 3
You perform EGD on a 50 yo and find a small mass in the
stomach. It comes back as a GIST so you decide to operate on
him. In the OR you find a small mass in the liver that comes
back as metastatic disease. The next best step is:
A.
B.
C.
D.

Radiation
5 FU and cisplatin
Gleevec
Surveillance

Answer 3
C. Gleevec, a tyrosine kinase inhibitor, has
been found to be extremely effective for
metastatic GIST

Question 4
A 65 yo man undergoes an Ivor-Lewis
esophagectomy. On POD 6 he develops a 103
fever and a WBC of 18. You start IVF and send
off cultures. The next best step is:
A.
B.
C.
D.

Chest CT
CT abdomen
Gastrograffin followed by thin barium swallow
Chest MRI

Answer 4
C. One of the most feared complications of
esophagectomy is the development of a leak.
This is usually manifested by fevers and an
elevated WBC. Severe leaks can result in
tachycardia and hypotension
The diagnostic study of choice is a gastrograffin
followed by a thin barium swallow.

Question 5
The barium swallow reveals a non-contained
leak at the level of the anastomosis. The next
best step is:
A.
B.
C.
D.

Chest tube
Re-exploration
Esophageal stent
Upper endoscopy

Answer 5
B. The patient needs exploration to find the
source of the leak

Question 6
You re-explore the above pt and find a pinhole leak
at the top of your suture line. The tissue otherwise
looks viable. The next best step in mgmt is:
A. Take the anastomosis down and perform
esophagostomy
B. Repair the defect and place an intercostal muscle flap
C. Lay a drain over the hole only
D. Place an esophageal stent

Answer 6
B. Finding just a tiny hole in the esophagus can be
repaired though you should re-enforce the repair
with a pleural, pericardial fat, or muscle flap to cover
the area. You should also place drains.
Keep the pt NPO x 1 week & repeat the swallow. Use
J-tube feeds until then
Esophagostomy for diversion can be considered
though this is pretty aggressive for this problem

Question 7
Instead of finding a tiny hole, you find that the
anastomosed stomach is necrotic. You then
should:
A. Take the conduit down and perform
esophagostomy
B. Repair the defect and cover with an intercostal
flap
C. Lay a drain over the hole
D. Place an esophageal stent

Answer 7
A. In this scenario, you need to take down the
conduit, resect the necrotic portion of
stomach, place drains, and perform an
esophagostomy
In 3 months you may consider reconnecting
the pt, likely with a colonic interposition

Question 8
The most common syndrome following
vagotomy and pyloroplasty is:
A.
B.
C.
D.

Diarrhea
Dumping syndrome
Gallstones
Alkaline reflux gastritis

Answer 8
B. Diarrhea is the most common problem
following vagotomy. Also true of post
vagotomy diarrhea:
Occurs because of sustained postprandial organized
MMCs
Results in nonconjugated bile salts in colon
Tx- cholestyramine, octreotide
Surgery usually not necessary though reversed
jejunal interposition graft can be considered

Question 9
A 60 yo patient that underwent antrectomy with
Billroth 2 reconstruction presents with epigastric
pain, nausea, and vomiting that often occurs after
meals. The discomfort is not relieved by vomiting.
The most likely diagnosis is:
A.
B.
C.
D.

Symptomatic cholelithiasis
Gastroparesis
Dumping syndrome
Alkaline reflux gastritis

Answer 9
D. This most likely represents alkaline reflux
gastritis. This is characterized by:
Postprandial epigastric pain, n/v
Pain not relieved by vomiting
Diagnose with EGD- will see bile in the stomach
Tx- PPI, cholestyramine, reglan
If medical mgmt not effective, consider conversion
to Roux-en-Y with afferent limb 60 cm distal to the
original gastrojejunostomy

Question 10
3 months after Billroth 1 for ulcer disease your patient notes
feeling dizzy and faint after eating as well as frequent diarrhea.
All of the following are true of the most likely condition except:
A. Surgery is rarely necessary though if needed, options
include conversion to Roux-en-Y or a procedure that will
increase the gastric reservoir or emptying time
B. You should first try small meals high in protein but low in
fat and carbs
C. Octreotride is not usually effective
D. Most cases improve medically

Answer 10
C is not true. All of the other statements are
applicable to dumping syndrome.
Surgery is rarely necessary though if needed, options
include conversion to Roux-en-Y or a procedure that will
increase the gastric reservoir (jejunal pouch) or emptying
time (reversed jejunal loop)
You should first try small meals high in protein but low in fat
and carbs
Octreotride is usually effective (take before meals)
Most cases improve medically

Answer 10
Dumping is due to the rapid entry of carbs into the
small bowel
There are 2 parts
Hyperosmotic load in bowel causes fluid shift ->
hypotension, diarrhea, dizziness
Insulin release (2nd phase) can cause hypoglycemia (rarely
occurs)

In addition to the other recs, pts should:


Avoid lying down after a meal
Avoid liquids with meals

Question 11
The most commonly injured nerve during CEA
is:
A.
B.
C.
D.

Vagus
Hypoglossal
Lingual
Glossopharyngeal

Answer 11
A. The most commonly injured nerve is the
vagus.
Because the recurrent laryngeal nerve has not
yet branched from the vagus at the level where
you are operating while doing a CEA, injury to
the vagus nerve in the neck places the pt at
risk for hoarseness.

Question 12
Which organ does not contain lymphatics?
A.
B.
C.
D.

Lung
Liver
Spleen
Muscle

Answer 12
D. Organs that do not contain lymphatics
include brain, tendon, muscle, bone, cartilage,
and cornea

Question 13
Myasthenia gravis affects:
A.
B.
C.
D.

Beta adrenergic receptors


Alpha adrenergic receptors
Acetylcholine receptors
Dopamine receptors

Answer 13
C. Myasthenia gravis involves formation of
antibodies to acetylcholine receptors which
impairs binding of acetylcholine

Question 14
A 56 yo woman presents with a weepy, scaly
lesion on her right nipple. You perform an H &
P and next:
A.
B.
C.
D.

Bilateral mammogram
Mammogram of the affected side
Core needle biopsy
FNA

Answer 14
A. The 1st step in any pt over the age of 30
with a suspected breast problem is to get a
bilateral mammogram

Question 15
The above pts MMG is normal. You should
next perform:
A.
B.
C.
D.

Modified radical mastectomy


Simple mastectomy
Wedge biopsy
Radiation

Answer 15
C. This could be Pagets or inflammatory
breast cancer. You need to get a skin biopsy
with some breast tissue first

Question 16
The biopsy shows Pagets and DCIS. You should
now offer her:
A. MRM
B. Simple mastectomy, including the nipple-areolar
complex
C. Lumpectomy and axillary LND
D. Radiation

Answer 16
B. The appropriate treatment of Pagets with
underlying DCIS is simple mastectomy
including the nipple-areolar complex
Note that simple mastectomy usually means
you leave the nipple but in Pagets you need to
take it

Question 17
You perform a simple mastectomy and
immediately send the entire specimen to
pathology. The pathologist calls and says there
is a 1 cm ductal carcinoma with at least a 1cm
clear margin. You next should:
A.
B.
C.
D.

Close
Sentinel node biopsy
Axillary node dissection
Chest wall resection

Answer 17
C. You need to perform a formal axillary
dissection. Since the breast tissue is already
gone sentinel node bx is no longer an option

Question 18
A 70 yo man presents for elective umbilical
hernia repair. He has a history of ascites
secondary to cirrhosis. Should he be offered
repair?
A. Yes, if it is significantly affecting his lifestyle
B. Yes, if he is leaking ascites from his hernia
C. Yes, if he is on the liver transplant list
D. No, he should not be offered repair

Answer 18
B: Yes, if he is leaking ascites.
The repair of umbilical hernias in cirrhotics is
associated with a high rate of recurrence
secondary to the ascites and nutritional
deficiencies, resulting in muscular wasting and
fascial thinning.
~20% of cirrhotics develop an abdominal wall
hernia.

Answer 18
Surgery on cirrhotics is complicated by risk of
hemorrhage from variceal disruption, peritonitis,
ascitic leak, and hepatic decompensation.
Minimize ascites pre-op via salt restriction,
diuretics, and possible paracentesis. TIPS also can
be considered.
Without adequate control of the ascites,
recurrence can be expected to be up to 73%

Question 19
At reoperation for a recurrence of an inguinal
hernia following a laparoscopic repair, the
most common location for the recurrent hernia
is:
A. Lateral
B. Medial
C. Anterior
D. Posterior

Answer 19
B. The most common location for breakdown
of a laparoscopic hernia repair is the medial
portion of the mesh.
This is usually because the mesh is too small or
that it was not appropriately attached
medially.

Question 20
Each drug is matched with a characteristic. Which of the following is
incorrect?
A. Etomidate- causes fewer hemodynamic changes than most other anesthetics
and is fast-acting
B. Ketamine- often used in children, can cause hallucinations, and does not
cause respiratory depression
C. Propofol- can cause respiratory depression but provides both analgesia and
amnesia
D. Sodium thiopental (barbiturate) is fast acting but can decrease cerebral blood
flow and blood pressure

Answer 20
C. Propofol is not an analgesic.
It does, however, have a rapid distribution
It can cause hypotension and respiratory depression
Is broken down in the liver.
It should not be used in pts with egg allergies

Etomidate
Causes fewer hemodynamic changes than most other anesthetics and is fastacting
Continuous infusions can lead to adrenocortical suppression

1. The lesion pictured in the xray may be associated with


dysphagia, regurgitation of food, and bad breath. All of
the following techniques may be used for treatment
EXCEPT

a.

Diverticulectomy and myotomy

b. Myotomy alone

c. Diverticulectomy alone
d. Internal pharyngoesophageal myotomy
e. Diverticulopexy with or without myotomy

Answer 1
C- Zenkers pharyngoesophageal diverticulum
These are a pulsion/ false diverticulum
Treatment is indicated in symptomatic patientsperform diverticulectomy and cricopharyngeal
myotomy to prevent recurrence
Diverticulopexy is acceptable in elderly/high-risk
patients

L cervical incision
Retract SCM/carotid sheath laterally,
thyroid/trachea medially
Divide inferior thyroid artery
40-Fr bougie in esophagus, dissect pouch at
base

extramucosal esophagomyotomy in both


directions (7-10 cm from base of pouch)
excise diverticulum with stapler if > 2 cm

Answer 1
Other esophageal diverticula:
Midesophageal
Traction
True diverticulum
caused by external inflammation (e.g. TB) that pulls out esophagus
new evidence suggest also may be from pulsion
more common on right
Epiphrenic
distal esophagus
Pulsion
False
more common on right
Tx if symptomatic or > 3 cm
repair via L thoracotomy, diverticulectomy, esophagomyotomy opposite side

Question 2
A 70-year-old man with a history of chronic obstructive pulmonary
disease present to the ED with diffuse abdominal pain and
distention. Labs show a WBC of 14,000 with a left shift of 90%. CT
confirms a small amount of pneumoperitoneum and mild small
bowel dilation. At laparotomy, diffuse small bowel diverticulosis of
the proximal jejunum is noted. The surgical procedure of choice
would be:
a.
b.
c.
d.
e.

irrigation of peritoneal cavity and drainage


jejunal resection and anastomosis of the perforated segment
jejunal resection and diverting ostomy
jejunal resection of the entire segment of diverticulosis
closure of perforated diverticulum

Answer 2
B. Resection and anastomosis of the
perforated segment.
SB diverticula are usually asymptomatic
Tx for perforation: resect perforated segment
and anastomosis
If perforation cannot be found, no resection
Resection of all diverticula not indicated

Question 3
A 30 yo woman presents 7 days s/p lap chole
with nausea, RUQ pain, and malaise. The most
appropriate next step is:
A.
B.
C.
D.

RUQ ultrasound
Change pain meds
ERCP
Antibiotics

Answer 3
A. Pain and nausea after a lap chole needs to
be worked up. Start by sending labs, including
LFTs, then obtain a scan.
The author of this question recommended u/s
though CT is acceptable & what we usually do

Question 4
An 8 x 8 cm fluid collection is found in the
gallbladder fossa of this patient. The next step
should be:
A. Antibiotics
B. Re-exploration
C. ERCP
D. Percutaneous drainage

Answer 4
D. Place a drain and see what you get

Question 5
The drainage is bilious fluid. You should next
order:
A. ERCP
B. Exploratory laparotomy
Abdominal CT
D. Broad spectrum antibiotics

Answer 5
A. Bile should make you think of a cystic duct
stump leak, biliary injury, or leak from Duct of
Luscka. This can often be treated with ERCP.

Question 6
ERCP shows free extravasation of contrast
coming from the cystic duct stump remnant.
The next most appropriate step is;
A.
B.
C.
D.

Exploratory laparotomy
Broad spectrum abx
PTC tube
ERCP, sphincterotomy, stent

Answer 6
D. Most likely, a clip fell off the cystic duct. 95% of
the time this can be effectively treated with
sphincterotomy and a temporary stent
The cystic duct remnant eventually scars down
This also effectively treats a duct of luschka leak
More serious injuries may require reoperation
though this should be done 8 weeks
postoperatively

Question 7
A 50 yo post-menopausal woman is diagnosed with
ovarian cancer. You find that she had disease in
both ovaries though all of the peritoneal &
diaphragmatic biopsies as well as peritoneal
washings are negative. What stage is this patients
cancer?

A.
B.
C.
D.

I
II
III
IV

Answer 7
A. Ovarian cancer limited to both ovaries is
stage I

Question 8
The above patient requires:
A. Oophrectomy and partial oophrectomy
B. Bilateral oophrectomy
C. Bilateral oophrectomy and salpingectomy
D. Total abdominal hysterectomy and bilateral
oophorectomy

Answer 8
D. Even though the disease is stage 1, total
abdominal hysterectomy and bilateral
oophrectomy is indicated

Question 9
A 35 yo man is POD#6 from a cadaveric kidney
transplant develops a rise in his creatnine. The
most appropriate step is:
A.
B.
C.
D.

Emergent re-operation
Angiography
OKT 3
Ultrasound

Answer 9
D. The 1st step in the work-up of a kidney
transplant pt with an elevated creatnine or
decreased UOP should be an ultrasound.
Ultrasound may assess vascular supply, looks for
ureteral compression, and can identify fluid
collections consistent with either urine leaks,
lymphoceles, hematomas, or seromas.
A biopsy can be done at the same time
Some people empirically treat with steroids while
waiting for the results of the biopsy

Question 10
In the previous pt, the ultrasound shows flow
acceleration and narrowing at the level of the
arterial anastomosis. The next step should be:
A.
B.
C.
D.

Emergent reoperation
Angiography
OKT 3
Additional biopsy

Answer 10
B. Angiogram with angioplasty and stent
placement is the treatment of choice for a tight
arterial anastomosis following kidney
transplantation

Question 11
Instead of the above, ultrasound is normal.
The next most appropriate step is;
A.
B.
C.
D.

Emergent re-operation
Angiography
OKT 3
Biopsy

Answer 11
D. If there is no obvious mechanical problem
with the graft you should obtain a biopsy

Question 12
Biopsy shows acute tubulitis. This is consistent
with:
A.
B.
C.
D.

Acute rejection
UTI
Chronic rejection
Renal vein thrombosis

Answer 12
A. Acute tubulitis is consistent with rejection.
A more severe reaction would involve
vasculitis.
This pt should be started on pulse dose
steroids.
Creatnine should be followed and the kidney
should be rebiopsied in 5-7 days

Question 13
After being treated with pulse-dose steroids for a
week, you re-biopsy the kidney and there is no
longer any signs of acute rejection although there is
a large fluid collection anterior to the kidney. You
send the fluid off and the creatnine is 20 (serum is
0/8.) The next step should be:

A.
B.
C.
D.

Explant the kidney


Try to repair the cysto-ureteral anastomosis
Place a stent and percutaneous drainage
Nothing

Answer 14
C. The most appropriate treatment for a urine
leak in most cases is stent placement across the
ureteral anastomosis and percutaneous drainage
of the fluid collection.
Trying to redo the anastomosis is usually
unsuccessful though it may be appropriate if the
anastomosis fell apart very early, like POD 1

Question 15
Which of the following can cause prolonged
ventilatory requirements?
A.
B.
C.
D.

Hypokalemia
Hypomagnesemia
Hyponatremia
Hypophosphatemia

Answer 15
D. Hypophosphatemia can result in prolonged
ventilation due to relative ATP insufficiency (need
phosphate to convert ADP to ATP).
K and Mg are important ions involved in gut motility
(these should be replaced in pts with a prolonged
ileus.)

Deficiencies in K and Mg can also contribute to heart


arrhythmias such as atrial fibrillation

Question 16
Which of the following may be associated with
increased gastric acid and increased gastrin
levels?
A. Gastric cancer
B. People on long-term PPI
C. Patients with pernicious anemia
D. A patient with retained antrum after partial
gastrectomy for ulcer disease

Answer 16
D. Patients with retained antrum may have high gastrin
and high acid levels
Other diseases that may lead to high gastrin and high acidZES, antral cell hyperplasia, renal failure, gastric outlet
obstruction, short bowel syndrome
Disease that may cause high gastrin and normal to low
acid- Pernicious anemia, cancer, chronic gastritis,
postvagotomy pts, people on medical acid suppression

Question 17
Which of the following is not true about
lobular carcinoma in situ (LCIS)?
A. 40% get cancer in either breast
B. It is considered a marker for breast cancer- in
itself it is not premalignant
C. It has no calcifications on MMG and is not
palpable
D. If you find this when you are resecting a lesion
for other lesions, you need to go back and get
negative margins

Answer 17
D. If LCIS is found you do not need to go back and get
negative margins. A, B, and C are true.
Other facts about LCIS:

Primarily found in premenopausal women; usually an incidental finding


Pts with it that ultimately develop breast cancer get ductal cancer (70%)
Multifocal disease is common
5% risk of having a synchronous cancer when this is diagnosed
Treatment options include monitoring, tamoxifen, and bilateral subcutaneous
mastectomy without axillary dissection

Question 18
Which of the following statements is not true?
A. Macula densa senses low Na/Cl and produces renin
B. Renin converts angiotensin 1 to angiotensin II
C. Angiotensin I is converted to angiotensin II in the
lung by ACE
D. Angiotensin II is a vasoconstrictor that causes an
increased aldosterone level -> sodium is therefore
retained and K/H is lost in the urine

Answer 18
B is incorrect.
Macula densa senses low Na/CL -> renin is then
produced
Renin converts angiotensiongen to angiotensin I
Angiotensin I gets converted to AT II in the lung by
angiotensin converting enzyme (ACE)
AT II causes vasoconstriction and aldosterone
release
Aldosterone leads to sodium uptake as well as loss
of H & K

Question 19
What should the rate of maintenance fluid be
on a 50 kg boy?
A.
B.
C.
D.

120 cc/hr
90 cc/hr
60 cc/hr
30 cc/hr

Answer 19
B. For the 1st 10 kg, the rate is 4 cc/kg/hr
For the next 10 kg, the rate is 2 cc/kg/hr
For anything after that, the rate is 1 cc/kg/hr

Question 20
Erythema multiforme is most likely with which
of the following antibiotics?
A.
B.
C.
D.

Quinolones
Bactrim
Erythromycin
Ceftriaxone

Answer 20
B. Bactrim has many side effects associated
with it. These include allergic reactions, renal
damage, erythema multiforme, and hemolysis
in G6PD deficient pts

Question 21
Of the following, which would be the best initial treatment
of most patients with bleeding esophageal varices?
a. Intravenous pitressin and nitroglycerin
b. Placement of a Sengstaken-Blakemore tube
c. Emergency esophagoscopy with sclerotherapy or variceal
ligation
d. Transjugular intrahepatic portosystemic shunt (TIPS)
e. Intravenous octreotide

Answer E

- Pitressin induces cardiac ischemia, which can be offset by nitro


- Octreotide is safer, acts by decreasing splanchnic blood flow
50 ug bolus, then 50 ug/h for 48-72 hours

The modified Sengstaken-Blakemore tube.


Note the accessory nasogastric (N-G) tube
for suctioning of secretions above the
esophageal balloon and the two clamps, one
secured with tape, to prevent inadvertent
decompression of the gastric balloon.

Question 1
Regarding the management of Crohns disease, which of
the following are not true?
A. If you operate on a patient that you suspect has appendicitis and you find
inflammation consistent with Crohns that does not affect the cecum, you
should still do an appendectomy as the fistula rate will not increase
B. Perianal abscesses in Crohns should be treated the same as other perianal
abscesses
C. Aphthous ulcers is more common in ulcerative colitis
D. A patient that is found to have granulomas, fissures, and submucosal fibrosis
is more likely to have Crohns

Answer 1
C is not true. All of the rest are accurate
statements regarding Crohns disease

Question 2
Which of the following regarding lung physiology is
not true?
A. The minimal pre-thoracotomy PFTs need to be an FEV1
of at least > 2L / 1L / 0.6L for
pneumonectomy/lobectomy/wedge resection
consecutively
B. Small cell lung cancer is the most common as well as
the most likely to demonstrate neoplastic syndrome
C. Type 2 alveoli make surfactant
D. Pancoast tumors involve sympathetic chain and/or
ulnar nerve

Answer 2
B is not true. Although small cell neoplasms
are the most likely to cause paraneoplastic
syndromes, they are not the most common
lung cancer
Adenocarcinoma is the most common lung
cancer

Question 3
You perform a core needle biopsy in a patient
with an axillary mass and it comes back as a
melanoma. You cannot find any skin lesions
over the patients body. Which of the following
is the most appropriate next step?
A. Formal axillary lymph node dissection
B. Surveillance
C. Mastectomy
D Chemotherapy and radiation

Answer 3
A. ALND is the most appropriate in this case.
There are a number of possibilities in a scenario like
this
1. The patient has a primary somewhere that you
cant find as some melanomas are not pigmented.
2. The primary lesion could have regressed
spontaneously
3. Different from breast cancer you do a I, II, & III
axillary dissection with melanoma

Question 4
The earliest and most specific sign of malignant
hyperthermia is:
A.
B.
C.
D.
E.

High fever
Hypotension
Increase in end tidal CO2
Tachycardia
Hypoxia

Answer 4
C. Increase in end-tidal CO2
Malignant hyperthermia is characterized by:
A defect in calcium metabolism. Calcium released from
sarcoplasmic reticulum causes muscle excitationcontraction syndrome
After an increase in ETCO2, fever, tachycardia, rigidity,
acidosis, and hyperkalemia begin
10/mg dantrolene inhibits calcium release. Also use
cooling blankets, fluids, bicarb
Is associated with succinylcholine (the only depolarizing
agent)

Question 5
The most frequent manifestation of blunt
myocardial contusion is:
A.
B.
C.
D.
E.

AV block
Atrial flutter
Premature ventricular contractions
Premature atrial contractions
Atrial fibrillation

Answer 5
C. Premature ventricular contractions.
V-tach and V-fib are the most common
arrhythmias with this injury that lead to death
Risk of death highest in the 1st 24 hrs
SVT is the most common arrhythmia overall
Monitor these patients closely

Question 6
The most commonly injured nerve under
general anaesthesia is:
A.
B.
C.
D.
E.

Radial nerve
Ulnar nerve
Median nerve
Brachial plexus
Common peroneal nerve

Answer 6
B. The ulnar nerve. It is important to make
sure the arms are positioned carefully to
prevent ulnar injury

Question 7
Respiratory distress associated with goiter is
most commonly caused by:
A. Recurrent laryngeal nerve palsy
B./ Malignant tracheal invasion
C. Retrosternal extension of goiter
D. Hemorrhage in a large goiter

Answer 7

C. Retrosternal extension of goiter.


Goiter describes any abnormal thyroid enlargement
Most identifiable cause- iodine deficiency
Diffuse enlargement without functional
abnormality- non-toxic goiter
Can be treated with iodine suppression, thioamides,
subtotal thyroidectomy, or lobectomy if medical
therapy is ineffective

Answer 7
Substernal goiter
Usually secondary- vessels originate from superior
and inferior thyroid arteries
Primary substernal goiter is rare; blood supply
originates from innominate
Can cause airway compression
Tx similar to other goiters

Question 8
Malignant hyperthermia:
A. Can be induced by local anasthetics
B. Can be induced by nondepolarizing muscle
relaxants
C. Can be induced by nitrous oxide
D. Is related to disordered K+ metabolism
E. Is more common in children than adults

Answer 8
E. Malignant hyperthermia is more common in
children than in adults

Question 9
The most common cardiac anomaly found in
adults is:
A.
B.
C.
D.

Atrial septal defect


Ventricular septal defect
Transposition of great vessels
Coarctation of the aorta

Answer 9
A. Atrial septal defect

Question 10
The most common complication of epidural
analgesia is:
A.
B.
C.
D.

Hypotension
Nausea
Respiratory distress
Deep vein thrombosis

Answer 10
C. Respiratory distress is the most frequent
complication following epidural anaesthesia
Epidurals cause sympathetic denervation and
vasodilation
Morphine in epidural can cause respiratory
depression
Lidocaine in epidural can cause decreased HR and
BP
Dilute concentrations can allow sparing of motor
function

Answer 10
Tx for acute hypotension and bradycardia- turn
epidural down, give fluids, phenylephrine, atropine
T5 epidural can affect cardiac accelerator nerves
Epidural is contraindicated with:
Hypertrophic cardiomyopathy
Cyanotic heart disease-> can get inadvertent spinal
anaesthesia

Question 11
The lateral boundary of a femoral hernia is:
A.
B.
C.
D.

The femoral nerve


The femoral artery
The femoral vein
The lacunar ligament

Answer 11
C. The femoral vein
Boundaries for femoral canal: Coopers ligament, the
inguinal ligament, and femoral vein
Femoral sheath- artery and femoral branch of
genitofemoral nerve is most lateral
Vein is medial to artery
Femoral canal is medial to vein- contains preperitoneal
fat, lymph nodes (Cloquets), and connective tissue.
Femoral hernia occurs when the blind end of the
femoral canal becomes a opening through which
peritoneal sac can protrude.

Answer 11
Femoral hernia boundaries:
superior- Inguinal ligament
inferior- pectineal ligament
medial- lacunar ligament (attaches to pubis &
connects inguinal and pectineal ligaments)
lateral- femoral vein

Question 12
A characteristic of primary hyperaldosteronism
is:
A.
B.
C.
D.
E.

Hyperkalemia
Hyperreninism
Hypertension
Hyperplasia of zona reticularis
Hyperplasia of zona fasciculata

Answer 12
C. Hypertension
Hyperaldosteronism (Conns syndrome)
Characterized by hypertension secondary to sodium
retention without edema
Hypokalemia, weakness, polydipsia, polyuria
Primary disease ~80% because of adenomas;
hyperplasia ~10%, ovarian tumors and carcinoma
less common

Question 13
Regarding the adrenal gland:
A. The adrenal cortex does not have nerve supply
B. The adrenal medulla is supplied by postganglionic
adrenergic fibers
C. The right adrenal vein drains into the renal vein
D. The left adrenal vein drains into the IVC

Answer 13
A. The adrenal cortex does not have a nerve
supply
The right adrenal vein drains into the IVC
where the left drains into the renal

Question 14
During an operation for appendicitis, the
appendix is noted to be normal but the
fallopian tube is thickened and surrounded by
purulent exudate. The operative management
should be:
A.
B.
C.
D.

Appendectomy
Appendectomy and salpingectomy
Salpingectomy
No operative intervention

Answer 14
A. Perform an appendectomy and treat the
salpingitis/ PID with antibiotics. It is safe to
perform an appendectomy in these
circumstances.

Question 15
In the preoperative preparation of
pheochromocytoma, medications should be
give in which order?
A.
B.
C.
D.

Diuretics and then alpha blockers


Alpha blockers and then beta blockers
Beta blockers and then alpha blockers
Diuretics and then beta blockers

Answer 15
B. The appropriate treatment preoperatively is
to first start alpha blockers and then beta
blockers if needed.
Treating with a beta blocker first may lead to
unopposed alpha stimulation, hypertensive
crisis, and heart failure in pts with a
cardiomyopathy

Question 16
Hyperinsulinism in a newborn is most likely
caused by:
A.
B.
C.
D.

Nesidioblastosis
Glycogen storage disease
Benign insulinoma
Malignant insulinoma

Answer 16
A. Nesidioblastosis (an old-school term I want you to be familiar with
in case it comes up)
Nesidioblastosis is a term for hyperinsulinemic hypoglycemia
attributed to excessive function of pancreatic beta cells
The abnormal histological aspects of the tissue included the presence
of islet cell enlargement, islet cell dysplasia, beta cells budding from
ductal epithelium, and islets in apposition to ducts.
Most congenital hyperinsulinism is caused by different mechanisms
than excessive proliferation of beta cells in a fetal pattern
This term fell into disfavor after it was recognized in the late 1980s
that the characteristic tissue features were sometimes seen in
pancreatic tissue from normal infants and even adults, and is not
consistently associated with hyperinsulinemic hypoglycemia.

Question 17
A patient with abdominal wall desmoid tumor
should be screened for:
A.
B.
C.
D.
E.

Lung cancer
Colon polyps
Breast cancer
Medullary thyroid cancer
Pancreatic cancer

Answer 17
B. Colon polyps
Gardners syndrome- FAP + desmoid tumors
FAP
An autosomal dominant disease
All get colon cancer by age 40
APC gene on chromosome 5
20% of cases are spontaneous
Polyps are not present at birth; form in puberty

Answer 17
Perform flex sig to check for polyps in pts
suspected to have FAP
Need prophylactic colectomy at age 20
Surgical options- j-pouch; total
proctocolectomy with ileostomy
Following colectomy, need to watch for
periampullary tumors of the duodenum
Turcots syndrome- colon cancer a/w APC
gene, brain tumors

Question 18
Neutropenic enterocolitis is a complication of:
A.
B.
C.
D.

Cytarabine
Cyclophosphamide
Doxorubicin
Cisplatin

Answer 18

A. Cytarabine
Bleomycin and busulfan- pulm fibrosis
Cisplatin- nephrotoxic, ototoxic, neurotoxic
Carboplatin- myelosuppresion
Vincristine- peripheral neuropathy, neurotoxic
Vinblastine- myelosuppression
Cyclophosphamide- SIADH, gonadal dysfunction,
hemorrhagic cystitis
Methotrexate- renal toxicity and radiation recall

Question 19
The most common intra-abdominal solid
tumor in children is:
A.
B.
C.
D.

Nephroblastoma
Neuroblastoma
Rhabdomyosarcoma
Fibrosarcoma

Answer 19
B. Neuroblastomas are the most common
intra-abdominal solid tumors in children

Question 20
An indication for laparotomy in neonatal
necrotizing enterocolitis is:
A.
B.
C.
D.

Distended bowel loops


Thickened bowel wall
Abdominal wall erythema
Pneumatosis intestinalis

Answer 20
C. Abdominal wall erythema
Other indications to operate- free air,
peritonitis, clinical deterioration
Need barium contrast enema before taking
down ostomies in patients that formerly had
NEC to rule out distal stenoses

1. A 55 year-old male, who is noted to have diarrhea, flushing and


bronchoconstriction as well as right-sided cardiac valvular disease, is most
likely to have his primary tumor where?
a) appendix
b) bronchus
c) rectum
d) stomach
e) ileum

1. A 55 year-old male, who is noted to have diarrhea, flushing and


bronchoconstriction as well as right-sided cardiac valvular disease, is most
likely to have his primary tumor where?
a) appendix
b) bronchus
c) rectum
d) stomach
e) ileum

2. A patient has a serum sodium of 115 but is asymptomatic.


His sodium is corrected by the next day and is 135. However,
the patient develops seizures and cerebral edema. What is
the most likely etiology of this?
a) central pontine myelinolysis
b) hepatic encephalopathy from unrecognized liver failure and
poor sodium homeostasis
c) hypercalcemia
d) hypomagnesemia
e) Wernickes encephalopathy

2. A patient has a serum sodium of 115 but is asymptomatic.


His sodium is corrected by the next day and is 135. However,
the patient develops seizures and cerebral edema. What is
the most likely etiology of this?
a) central pontine myelinolysis
b) hepatic encephalopathy from unrecognized liver failure and
poor sodium homeostasis
c) hypercalcemia
d) hypomagnesemia
e) Wernickes encephalopathy

3. Which hormone can be used to stimulate small bowel


hypertrophy in order to lessen the symptoms of short gut
syndrome?
a) glucagon
b) enteroglucagon
c) somatostatin
d) growth hormone
e) insulin-like growth factor

3. Which hormone can be used to stimulate small bowel


hypertrophy in order to lessen the symptoms of short gut
syndrome?
a) glucagon
b) enteroglucagon
c) somatostatin
d) growth hormone
e) insulin-like growth factor

4. Which is the only pancreatic enzyme which is not


secreted as a prohormone? It is active at the time of
secretion into the pancreatic duct.
a) phospholipase A2
b) chymotrypsin
c) colipase
d) carboxypeptidase A
e) lipase

4. Which is the only pancreatic enzyme which is not


secreted as a prohormone? It is active at the time of
secretion into the pancreatic duct.
a) phospholipase A2
b) chymotrypsin
c) colipase
d) carboxypeptidase A
e) lipase

5. In which portion of the gastrointestinal tract are


proteins mostly absorbed?
a) stomach
b) duodenum
c) jejunum
d) ileum
e) colon

5. In which portion of the gastrointestinal tract are


proteins mostly absorbed?
a) stomach
b) duodenum
c) jejunum
d) ileum
e) colon

6. Which of the following is the toxic component of


lipopolysaccharide (LPS)?
a) O-specific antigen
b) lipid A moiety
c) core M protein
d) M protein coat
e) capsule wall

6. Which of the following is the toxic component of


lipopolysaccharide (LPS)?
a) O-specific antigen
b) lipid A moiety
c) core M protein
d) M protein coat
e) capsule wall

7. Mechanism of inheritance in MEN I, MEN IIa and


MEN IIb?
a) genomic, non-dominant
b) X-linked
c) autosomal dominant
d) autosomal recessive
e) sex-linked

7. Mechanism of inheritance in MEN I, MEN IIa and


MEN IIb?
a) genomic, non-dominant
b) X-linked
c) autosomal dominant
d) autosomal recessive
e) sex-linked

8. Which bacterial genus produces the most potent


exotoxins?
a) Staphylococcus
b) Pseudomonas
c) Klebsiella
d) Streptococcus
e) Serratia

8. Which bacterial genus produces the most potent


exotoxins?
a) Staphylococcus
b) Pseudomonas
c) Klebsiella
d) Streptococcus
e) Serratia

9. A 22 year-old male undergoes extensive small bowel


resection for complications related to Crohns disease.
He develops short gut syndrome. Which vitamin is he
most likely to malabsorb?
a) riboflavin
b) niacin
c) vitamin B6
d) vitamin B12
e) vitamin C

9. A 22 year-old male undergoes extensive small bowel


resection for complications related to Crohns disease.
He develops short gut syndrome. Which vitamin is he
most likely to malabsorb?
a) riboflavin
b) niacin
c) vitamin B6
d) vitamin B12
e) vitamin C

10. What nerve injury is associated with a proximal


shaft of humerus fracture?
a) radial
b) musculocutaneous
c) axillary
d) median
e) long thoracic

10. What nerve injury is associated with a proximal


shaft of humerus fracture?
a) radial
b) musculocutaneous
c) axillary
d) median
e) long thoracic

11. What is the mechanism for suppression of gastrin


release?
a) antral distention
b) elaboration of secretin which has a reciprocal
relationship with gastrin
c) decreased levels of pancreatic polypeptide
d) direct vagal inhibition via release of gastrin-inhibiting
peptide (GIP)
e) antral acidification

11. What is the mechanism for suppression of gastrin


release?
a) antral distention
b) elaboration of secretin which has a reciprocal
relationship with gastrin
c) decreased levels of pancreatic polypeptide
d) direct vagal inhibition via release of gastrin-inhibiting
peptide (GIP)
e) antral acidification

12. Cholesterol solubility depends on which three


factors?
a) cholesterol, bile salts, and phospholipids
b) cholesterol, bilirubin, and phospholipids
c) cholesterol, proteins, and bile salts
d) cholesterol, proteins, and phospholipids
e) cholesterol, bilirubin, and proteins

12. Cholesterol solubility depends on which three


factors?
a) cholesterol, bile salts, and phospholipids
b) cholesterol, bilirubin, and phospholipids
c) cholesterol, proteins, and bile salts
d) cholesterol, proteins, and phospholipids
e) cholesterol, bilirubin, and proteins

13. The surface gastric epithelial cell secretes which


substance?
a) mucus
b) HCl
c) pepsin
d) intrinsic factor
e) bicarbonate

13. The surface gastric epithelial cell secretes which


substance?
a) mucus
b) HCl
c) pepsin
d) intrinsic factor
e) bicarbonate

14. Mass movement is the characteristic motility pattern


of which portion of the gastrointestinal tract?
a) stomach
b) duodenum
c) jejunum
d) ileum

e) colon

14. Mass movement is the characteristic motility pattern


of which portion of the gastrointestinal tract?
a) stomach
b) duodenum
c) jejunum
d) ileum

e) colon

15. The characteristic pattern of motility in the fasted


state is the migrating myoelectric complex (MMC).
Which hormone is responsible for regulating the MMC?
a) pancreatic polypeptide
b) bombesin
c) somatostatin
d) motilin
e) neurotensin

15. The characteristic pattern of motility in the fasted


state is the migrating myoelectric complex (MMC).
Which hormone is responsible for regulating the MMC?
a) pancreatic polypeptide
b) bombesin
c) somatostatin
d) motilin
e) neurotensin

16. When there is an increased stimulus for pancreatic


exocrine secretions, which electrolyte will decrease the
most in the pancreatic effluent?
a) sodium
b) chloride
c) bicarbonate
d) potassium
e) calcium

16. When there is an increased stimulus for pancreatic


exocrine secretions, which electrolyte will decrease the
most in the pancreatic effluent?
a) sodium
b) chloride
c) bicarbonate
d) potassium
e) calcium

17. What is the mechanism of activation of the


pancreatic proenzymes?
a) activation by enterokinase
b) activation by pepsin
c) activation by duodenal acidification
d) activation by trypsin
e) activation by lipase

17. What is the mechanism of activation of the


pancreatic proenzymes?
a) activation by enterokinase
b) activation by pepsin
c) activation by duodenal acidification
d) activation by trypsin
e) activation by lipase

18. Which of the following is the best indicator of


sepsis?
a) tachycardia
b) decreased oxygen consumption
c) decreased cardiac output
d) peripheral vasoconstriction
e) decreased systemic vascular resistance

18. Which of the following is the best indicator of


sepsis?
a) tachycardia
b) decreased oxygen consumption
c) decreased cardiac output
d) peripheral vasoconstriction
e) decreased systemic vascular resistance

19. What is the most common mechanism for the


development of resistance by a bacterial cell?
a) spontaneous mutation
b) development of multidrug resistance phenotype
c) chromosome transfer
d) bacteriophage infection
e) plasmid acquisition

19. What is the most common mechanism for the


development of resistance by a bacterial cell?
a) spontaneous mutation
b) development of multidrug resistance phenotype
c) chromosome transfer
d) bacteriophage infection
e) plasmid acquisition

20. You can declare someone brain-dead if they


exhibit one of these?
a) hypothermia
b) gag reflex
c) corneal reflex
d) brisk deep tendon reflexes
e) pupillary light reflex

20. You can declare someone brain-dead if they


exhibit one of these?
a) hypothermia
b) gag reflex
c) corneal reflex
d) brisk deep tendon reflexes
e) pupillary light reflex

21. A cirrhotic patient with intractable ascites has a


peritoneovenous shunt placed and is noted to begin
oozing from many sites including his incisions. What
is the best explanation for this?
a)
b)
c)
d)
e)

disseminated intravascular coagulation


unrecognized preoperative coagulopathy
dilutional thrombocytopenia
heparin-induced thrombocytopenia
hypothermia-induced thrombocytopenia

21. A cirrhotic patient with intractable ascites has a


peritoneovenous shunt placed and is noted to begin
oozing from many sites including his incisions. What
is the best explanation for this?
a)
b)
c)
d)
e)

disseminated intravascular coagulation


unrecognized preoperative coagulopathy
dilutional thrombocytopenia
heparin-induced thrombocytopenia
hypothermia-induced thrombocytopenia

22. A 70 year-old male is 3 months postoperative from an


aortobifemoral revascularization for aortoiliac occlusive disease.
He returns to the office complaining of pain and swelling in his right
groin. On physical examination, the right groin appears fluctuant
and tender. It drains 10 cc of clear, non-foul smelling material.
What bacterial species is most likely to cause this infection?
a) Clostridium perfringens
b) Staphylococcus aureus
c) Staphylococcus epidermidis
d) Streptococcus pneumoniae
e) Group A Streptococci

22. A 70 year-old male is 3 months postoperative from an


aortobifemoral revascularization for aortoiliac occlusive disease.
He returns to the office complaining of pain and swelling in his right
groin. On physical examination, the right groin appears fluctuant
and tender. It drains 10 cc of clear, non-foul smelling material.
What bacterial species is most likely to cause this infection?
a) Clostridium perfringens
b) Staphylococcus aureus
c) Staphylococcus epidermidis
d) Streptococcus pneumoniae
e) Group A Streptococci

23. From a hematologic viewpoint, the loss of which


immune function performed by the spleen makes
patients susceptible to overwhelming post-splenectomy
infection (OPSI)?
a) loss of IgA
b) loss of IgM
c) loss of IgG
d) loss of IgE
e) loss of IgD

23. From a hematologic viewpoint, the loss of which


immune function performed by the spleen makes
patients susceptible to overwhelming post-splenectomy
infection (OPSI)?
a) loss of IgA
b) loss of IgM
c) loss of IgG
d) loss of IgE
e) loss of IgD

24. Which of the following statements is true concerning


intraperitoneal bacteria?
a) They are absorbed by the diaphragmatic lymphatics.
b) It is normal to have bacteria in the peritoneal cavity.
c) They are cleared by intraperitoneal lymphocytes.
d) Characteristically, secondary peritonitis is caused by a single
organism and it is usually a gram negative infection.
e) Secondary peritonitis is usually a blood borne infection from
a primary pulmonary focus.

24. Which of the following statements is true concerning


intraperitoneal bacteria?
a) They are absorbed by the diaphragmatic lymphatics.
b) It is normal to have bacteria in the peritoneal cavity.
c) They are cleared by intraperitoneal lymphocytes.
d) Characteristically, secondary peritonitis is caused by a single
organism and it is usually a gram negative infection.
e) Secondary peritonitis is usually a blood borne infection from
a primary pulmonary focus.

25. An axial groin flap is based on the blood supply of


which artery?
a)
b)
c)
d)
e)

superficial epigastric
superficial circumflex
deep circumflex
pudendal
deep inferior epigastric

25. An axial groin flap is based on the blood supply of


which artery?
a)
b)
c)
d)
e)

superficial epigastric
superficial circumflex
deep circumflex
pudendal
deep inferior epigastric

26. What is the most common type of bacteria in the


colon?
a) Escherichia coli
b) Lactobacillus spp.
c) Peptostreptococcus spp.
d) Peptococcus spp.
e) Bacteroides spp.

26. What is the most common type of bacteria in the


colon?
a) Escherichia coli
b) Lactobacillus spp.
c) Peptostreptococcus spp.
d) Peptococcus spp.
e) Bacteroides spp.

27. A patient undergoes total thyroidectomy with right neck


dissection. The surgeon injures the R hypoglossal nerve, the R
superior laryngeal nerve, and the L recurrent laryngeal nerve.
What deficits would you expect to find postoperatively?

a)
b)
c)
d)
e)

Cord
Tongue deviation
L cord tensed
L
L cord tensed
R
R cord tensed
L
R cord tensed
R
both cords tensed L

27. A patient undergoes total thyroidectomy with right neck


dissection. The surgeon injures the R hypoglossal nerve, the R
superior laryngeal nerve, and the L recurrent laryngeal nerve.
What deficits would you expect to find postoperatively?

a)
b)
c)
d)
e)

Cord
Tongue deviation
L cord tensed
L
L cord tensed
R
R cord tensed
L
R cord tensed
R
both cords tensed L

28. Which of the following sarcomas has the greatest


tendency to metastasize to regional lymph nodes?
a)
b)
c)
d)
e)

liposarcoma
malignant fibrous histiocytoma
schwannoma
epitheloid sarcoma
dermoid sarcoma

28. Which of the following sarcomas has the greatest


tendency to metastasize to regional lymph nodes?
a)
b)
c)
d)
e)

liposarcoma
malignant fibrous histiocytoma
schwannoma
epitheloid sarcoma
dermoid sarcoma

29. A 78 year-old female has a node-negative breast


cancer with positive ER/PR. Her stage is T1C N0 M0. What
is the most appropriate therapy?
a) tamoxifen for 1 year
b) tamoxifen for 5 years
c) CMF followed by tamoxifen for 1 year
d) CMF followed by tamoxifen for 5 years
e) no further therapy

29. A 78 year-old female has a node-negative breast


cancer with positive ER/PR. Her stage is T1C N0 M0. What
is the most appropriate therapy?
a) tamoxifen for 1 year
b) tamoxifen for 5 years
c) CMF followed by tamoxifen for 1 year
d) CMF followed by tamoxifen for 5 years
e) no further therapy

30. Where is the most common location to find an


accessory spleen?
a) gastrocolic ligament
b) splenocolic ligament
c) gastrosplenic ligament
d) greater omentum
e) splenic hilum

30. Where is the most common location to find an


accessory spleen?
a) gastrocolic ligament
b) splenocolic ligament
c) gastrosplenic ligament
d) greater omentum
e) splenic hilum

31. A patient undergoes laparoscopic cholecystectomy and 5


days later the pathology report reveals that there was a 0.9
mm foci of adenocarcinoma limited to the mucosa. What is
the next appropriate step in management?
a) observation
b) wedge resection of the liver of segment V with
lymphadenectomy
c) radiation
d) chemotherapy
e) hepatic infuse-aid pump

31. A patient undergoes laparoscopic cholecystectomy and 5


days later the pathology report reveals that there was a 0.9
mm foci of adenocarcinoma limited to the mucosa. What is
the next appropriate step in management?
a) observation
b) wedge resection of the liver of segment V with
lymphadenectomy
c) radiation
d) chemotherapy
e) hepatic infuse-aid pump

32. Which of the following is an absolute contraindication


to the use of an intra-aortic balloon pump (IABP)?
a) mitral stenosis
b) pulmonic stenosis
c) aortic stenosis
d) aortic insufficiency

e) mitral insufficiency

32. Which of the following is an absolute contraindication


to the use of an intra-aortic balloon pump (IABP)?
a) mitral stenosis
b) pulmonic stenosis
c) aortic stenosis
d) aortic insufficiency

e) mitral insufficiency

33. A 21 year-old jogger develops claudication after 100 yards. On


physical examination, he has normal pulses in his lower extremities
including the affected leg. What is the next most appropriate step
in management?
a) place the patient on Trental
b) obtain ABIs
c) arteriogram looking for deviation of the popliteal artery since this
most likely represents popliteal entrapment syndrome
d) graded exercise regimen
e) reassurance and observation

33. A 21 year-old jogger develops claudication after 100 yards. On


physical examination, he has normal pulses in his lower extremities
including the affected leg. What is the next most appropriate step
in management?
a) place the patient on Trental
b) obtain ABIs
c) arteriogram looking for deviation of the popliteal artery since
this most likely represents popliteal entrapment syndrome
d) graded exercise regimen
e) reassurance and observation

34. Which of the following drugs increases the cardiac


output and increases the systemic vascular resistance?
a) isoproterenol
b) milrinone
c) amrinone
d) norepinephrine

e) dobutamine

34. Which of the following drugs increases the cardiac


output and increases the systemic vascular resistance?
a) isoproterenol
b) milrinone
c) amrinone
d) norepinephrine

e) dobutamine

35. What period of time in wound healing is


synthesis of collagen at its maximum?
a)
b)
c)
d)
e)

0 1 week
3 6 weeks
3 6 months
6 9 months
9 12 months

35. What period of time in wound healing is


synthesis of collagen at its maximum?
a)
b)
c)
d)
e)

0 1 week
3 6 weeks
3 6 months
6 9 months
9 12 months

36. Which statement concerning the relationships in the


hepatoduodenal ligament is true?
a) The portal vein is posterior to the CBD and hepatic artery; CBD is
medial to the hepatic artery.
b) The portal vein is posterior to the CBD and hepatic artery; CBD is
lateral to the hepatic artery.
c) The portal vein is anterior to the CBD and hepatic artery.
d) The portal vein is lateral to the CBD and both are anterior to the
hepatic artery.
e) The portal vein is medial to the CBD and both are anterior to the
hepatic artery.

36. Which statement concerning the relationships in the


hepatoduodenal ligament is true?
a) The portal vein is posterior to the CBD and hepatic artery; CBD is
medial to the hepatic artery.
b) The portal vein is posterior to the CBD and hepatic artery; CBD is
lateral to the hepatic artery.
c) The portal vein is anterior to the CBD and hepatic artery.
d) The portal vein is lateral to the CBD and both are anterior to the
hepatic artery.
e) The portal vein is medial to the CBD and both are anterior to the
hepatic artery.

37. A patient who has suffered multi-organ trauma is taken back to


the operating room to repair intra-abdominal injuries. He receives
multiple transfusions in the OR and it is noted that there is oozing
from the retroperitoneum where the R colon was mobilized. After
packing and allowing adequate time to stop, it is still noted to be
oozing. Intra-operatively his coags and platelets are checked and
found to be within normal limits. What should be done at this
time?
a) administer platelets
b) administer FFP
c) administer cryoprecipitate
d) administer DDAVP
e) continue to cauterize the area of bleeding

37. A patient who has suffered multi-organ trauma is taken back to


the operating room to repair intra-abdominal injuries. He receives
multiple transfusions in the OR and it is noted that there is oozing
from the retroperitoneum where the R colon was mobilized. After
packing and allowing adequate time to stop, it is still noted to be
oozing. Intra-operatively his coags and platelets are checked and
found to be within normal limits. What should be done at this
time?
a) administer platelets
b) administer FFP
c) administer cryoprecipitate
d) administer DDAVP
e) continue to cauterize the area of bleeding

38. Which of the following tends to make bile


lithogenic?
a) oral chenodeoxylate
b) resection of the jejunum
c) resection of the distal stomach
d) resection of the terminal ileum
e) resection of the colon

38. Which of the following tends to make bile


lithogenic?
a) oral chenodeoxylate
b) resection of the jejunum
c) resection of the distal stomach
d) resection of the terminal ileum
e) resection of the colon

39. What is the most frequent complication after a level I


and II axillary lymph node dissection?
a) seroma
b) hematoma
c) winged scapula
d) loss of sensation on the posterior aspect of the upper
arm
e) lymphedema

39. What is the most frequent complication after a level I


and II axillary lymph node dissection?
a) seroma
b) hematoma
c) winged scapula
d) loss of sensation on the posterior aspect of the upper
arm
e) lymphedema

40. A patient sustained a blunt MVA and was noted to have a


liver injury which was managed non-operatively. Two months
later, he develops hematemesis and melena. EGD and
colonoscopy are noncontributory. What is the next most
appropriate step in management?
a) vagotomy and antrectomy
b) vagotomy and pyloroplasty
c) exploratory laparotomy and liver resection
d) angiography and embolization
e) observation

40. A patient sustained a blunt MVA and was noted to have a


liver injury which was managed non-operatively. Two months
later, he develops hematemesis and melena. EGD and
colonoscopy are noncontributory. What is the next most
appropriate step in management?
a) vagotomy and antrectomy
b) vagotomy and pyloroplasty
c) exploratory laparotomy and liver resection
d) angiography and embolization
e) observation

41. A patient with Crohns disease is operated on for


symptoms of bowel obstruction. At the time of operation, it
is noted that there are two areas of stricture in the small
intestine and severe ileocecal Crohns disease. What is the
next most appropriate step in management?
a) ileocecectomy and two enterectomies
b) ileocecectomy and two stricturoplasties
c) ileocecectomy and bypass of the strictures
d) three stricturoplasties
e) enterectomy encompassing all diseased segments

41. A patient with Crohns disease is operated on for


symptoms of bowel obstruction. At the time of operation, it
is noted that there are two areas of stricture in the small
intestine and severe ileocecal Crohns disease. What is the
next most appropriate step in management?
a) ileocecectomy and two enterectomies
b) ileocecectomy and two stricturoplasties
c) ileocecectomy and bypass of the strictures
d) three stricturoplasties
e) enterectomy encompassing all diseased segments

42. A woman presents after blunt MVA. Abdominal CT shows


a 3 cm right adrenal mass. All endocrinologic studies are
negative and she has no identifiable signs of hormone excess.
What is the next most appropriate step in management?
a) transperitoneal adrenalectomy

b) extraperitoneal adrenalectomy
c) MRI of the abdomen
d) selective venous sampling
e) observation and repeat CT scan in 6 months

42. A woman presents after blunt MVA. Abdominal CT shows


a 3 cm right adrenal mass. All endocrinologic studies are
negative and she has no identifiable signs of hormone excess.
What is the next most appropriate step in management?
a) transperitoneal adrenalectomy

b) extraperitoneal adrenalectomy
c) MRI of the abdomen
d) selective venous sampling
e) observation and repeat CT scan in 6 months

43. A 25 year-old female undergoes resection of one foot


of small intestine including her terminal ileum. Several
months later, she presents with renal stones. What kind
of renal stones did she most likely develop?
a) calcium phosphate
b) calcium oxalate
c) struvite
d) uric acid
e) magnesium ammonium phosphate

43. A 25 year-old female undergoes resection of one foot


of small intestine including her terminal ileum. Several
months later, she presents with renal stones. What kind
of renal stones did she most likely develop?
a) calcium phosphate
b) calcium oxalate
c) struvite
d) uric acid
e) magnesium ammonium phosphate

44. A patient with a lateral neck mass undergoes FNA which


reveals squamous cell carcinoma. Further work-up is
negative for the primary tumor. The patients Epstein-Barr
virus (EBV) titers are noted to be high. Where is the most
likely source of the primary?
a) tonsillar pillar
b) hypopharnyx
c) nasopharnyx
d) floor of mouth
e) pyriform sinus

44. A patient with a lateral neck mass undergoes FNA which


reveals squamous cell carcinoma. Further work-up is
negative for the primary tumor. The patients Epstein-Barr
virus (EBV) titers are noted to be high. Where is the most
likely source of the primary?
a) tonsillar pillar
b) hypopharnyx
c) nasopharnyx
d) floor of mouth
e) pyriform sinus

45. von Willebrand factor (vWF) is synthesized in


which of the following?
a) platelet
b) macrophage
c) endothelial cell
d) RBC
e) hepatocyte

45. von Willebrand factor (vWF) is synthesized in


which of the following?
a) platelet
b) macrophage
c) endothelial cell
d) RBC
e) hepatocyte

46. In performing a pericardiocentesis, the most


appropriate manner to ensure the needle is in the
pericardial sac is to:
a)
b)
c)
d)
e)

check for pulsatile flow


do it under fluoroscopy
connect it to an EKG lead
check to see if the blood clots
never do it unless there is ultrasound guidance
available

46. In performing a pericardiocentesis, the most


appropriate manner to ensure the needle is in the
pericardial sac is to:
a)
b)
c)
d)
e)

check for pulsatile flow


do it under fluoroscopy
connect it to an EKG lead
check to see if the blood clots
never do it unless there is ultrasound guidance
available

47. The initial site of blood cell production in the


embryo occurs in which of the following?
a) liver
b) yolk sac
c) bone marrow
d) spleen
e) thymus

47. The initial site of blood cell production in the


embryo occurs in which of the following?
a) liver
b) yolk sac
c) bone marrow
d) spleen
e) thymus

48. Which of the following electrolyte abnormalities


exacerbates digitalis toxicity and also maintains
metabolic alkalosis?
a) hypernatremia
b) hyponatremia
c) hyperkalemia
d) hypokalemia
e) hypercalcemia

48. Which of the following electrolyte abnormalities


exacerbates digitalis toxicity and also maintains
metabolic alkalosis?
a) hypernatremia
b) hyponatremia
c) hyperkalemia
d) hypokalemia
e) hypercalcemia

49. In a patient with renal failure, which neuromuscular


blocking agent would you give in order to maintain
constant plasma levels?
a)
b)
c)
d)
e)

succinylcholine
D-tubocurarine
pancuronium
atracurium
vecuronium

49. In a patient with renal failure, which neuromuscular


blocking agent would you give in order to maintain
constant plasma levels?
a)
b)
c)
d)
e)

succinylcholine
D-tubocurarine
pancuronium
atracurium
vecuronium

50. Which of the following amino acids is an essential


amino acid?
a) tyrosine
b) proline
c) glutamine
d) valine
e) alanine

50. Which of the following amino acids is an essential


amino acid?
a) tyrosine
b) proline
c) glutamine
d) valine
e) alanine

51. Patients with kidney transplants have an increased


incidence of developing cancer. Which of the following
cancers has the highest incidence?
a) Kaposis sarcoma
b) lymphomas
c) skin cancer
d) lip cancer
e) vulvar cancer

51. Patients with kidney transplants have an increased


incidence of developing cancer. Which of the following
cancers has the highest incidence?
a) Kaposis sarcoma
b) lymphomas
c) skin cancer
d) lip cancer
e) vulvar cancer

52. An otherwise healthy 22 year-old pregnant female


who develops C. difficile colitis after antibiotic treatment
should receive which of the following regimens?
a) PO flagyl
b) PO vancomycin
c) IV flagyl
d) IV vancomycin
e) No treatment in pregnant patients

52. An otherwise healthy 22 year-old pregnant female


who develops C. difficile colitis after antibiotic treatment
should receive which of the following regimens?
a) PO flagyl
b) PO vancomycin
c) IV flagyl
d) IV vancomycin
e) No treatment in pregnant patients

53. What is the initial management of a patient with


a 5 cm amebic liver abscess?
a) metronidazole
b) percutaneous drainage
c) open surgical drainage
d) mebendazole
e) ampicillin, gentamicin, and flagyl

53. What is the initial management of a patient with


a 5 cm amebic liver abscess?
a) metronidazole
b) percutaneous drainage
c) open surgical drainage
d) mebendazole
e) ampicillin, gentamicin, and flagyl

54. Which of the following equations is correct for


the determination of oxygen delivery?
a)
b)
c)
d)
e)

[SaO2 x Hgb x 1.34] + PaO2 (0.003)


[PaO2 x Hgb x 1.34] + SaO2 (0.003)
[PaO2/Hgb x 1.34] + SaO2 (0.003) x CO
[SaO2 x Hgb x 1.34] + PaO2 (0.003) x CO
[SaO2 x PaO2 x 0.003] + 1.34 (Hgb) x CO

54. Which of the following equations is correct for


the determination of oxygen delivery?
a)
b)
c)
d)
e)

[SaO2 x Hgb x 1.34] + PaO2 (0.003)


[PaO2 x Hgb x 1.34] + SaO2 (0.003)
[PaO2/Hgb x 1.34] + SaO2 (0.003) x CO
[SaO2 x Hgb x 1.34] + PaO2 (0.003) x CO
[SaO2 x PaO2 x 0.003] + 1.34 (Hgb) x CO

55. Which of the following is true concerning meperidine


(demerol)?
a) contraindicated in patients who are taking MAO
inhibitors
b) requires less naloxone to reverse than morphine
c) preferable to use in renal failure over morphine
d) is an agonist/antagonist drug
e) none of the statements are true

55. Which of the following is true concerning meperidine


(demerol)?
a) contraindicated in patients who are taking MAO
inhibitors
b) requires less naloxone to reverse than morphine
c) preferable to use in renal failure over morphine
d) is an agonist/antagonist drug
e) none of the statements are true

56. When looking at carotid endarterectomies


statistically, which of the following could be considered a
continuous variable?
a) smoking
b) ulcerated plaque
c) race
d) TIAs
e) blood pressure

56. When looking at carotid endarterectomies


statistically, which of the following could be considered a
continuous variable?
a) smoking
b) ulcerated plaque
c) race
d) TIAs
e) blood pressure

57. In which portion of the pancreas are insulinomas


predominantly found?
a) head
b) neck
c) body
d) tail
e) equal frequency throughout pancreas

57. In which portion of the pancreas are insulinomas


predominantly found?
a) head
b) neck
c) body
d) tail
e) equal frequency throughout pancreas

58. A 45 year-old female is noted to have inflammatory


breast cancer without ulceration. Which of the following
is the most appropriate initial treatment?
a) toilet mastectomy
b) modified radical mastectomy
c) radical mastectomy
d) neoadjuvant chemotherapy
e) neoadjuvant radiation therapy

58. A 45 year-old female is noted to have inflammatory


breast cancer without ulceration. Which of the following
is the most appropriate initial treatment?
a) toilet mastectomy
b) modified radical mastectomy
c) radical mastectomy
d) neoadjuvant chemotherapy
e) neoadjuvant radiation therapy

59. Which of the following is true concerning splenic artery


aneurysms?

a) They have no predilection for either sex.


b) When symptomatic, these aneurysms can often be
managed conservatively.
c) Women of childbearing age are at increased risk of rupture
and should have elective repair.
d) Surgical treatment should never include splenectomy.
e) Proximal aneurysms are best managed by exclusion and
vascular reconstruction in order for splenic salvage.

59. Which of the following is true concerning splenic artery


aneurysms?

a) They have no predilection for either sex.


b) When symptomatic, these aneurysms can often be
managed conservatively.
c) Women of childbearing age are at increased risk of rupture
and should have elective repair.
d) Surgical treatment should never include splenectomy.
e) Proximal aneurysms are best managed by exclusion and
vascular reconstruction in order for splenic salvage.

60. Which of the following is true concerning the thyroid gland?


a) Thyroid tissue found in the lateral neck compartments is known
as lateral aberrant thyroid tissue and is an embryologic variation.
b) Any thyroid tissue found in lymph node tissue in the neck which is
extrathyroid represents metastatic deposits from thyroid
carcinoma.
c) Lingual thyroid results from a failure of the third pharyngeal
pouch to descend in a normal fashion.
d) Lingual thyroid usually occurs in conjunction with normal
anatomic thyroid tissue.
e) Lingual thyroid is best treated by surgical excision.

60. Which of the following is true concerning the thyroid gland?


a) Thyroid tissue found in the lateral neck compartments is known
as lateral aberrant thyroid tissue and is an embryologic variation.
b) Any thyroid tissue found in lymph node tissue in the neck which
is extrathyroid represents metastatic deposits from thyroid
carcinoma.
c) Lingual thyroid results from a failure of the third pharyngeal
pouch to descend in a normal fashion.
d) Lingual thyroid usually occurs in conjunction with normal
anatomic thyroid tissue.
e) Lingual thyroid is best treated by surgical excision.

61. Which of the following is the initial site of spread


of ovarian cancer?
a) bone
b) liver
c) rectum
d) bladder
e) peritoneum

61. Which of the following is the initial site of spread


of ovarian cancer?
a) bone
b) liver
c) rectum
d) bladder
e) peritoneum

62. Resection of the terminal ileum results in which


one of the following?
a) iron-deficiency anemia
b) sideroblastic anemia
c) macrocytic anemia
d) pernicious anemia
e) calcium phosphate renal stones

62. Resection of the terminal ileum results in which


one of the following?
a) iron-deficiency anemia
b) sideroblastic anemia
c) macrocytic anemia
d) pernicious anemia
e) calcium phosphate renal stones

63. A critically ill patient has been receiving TPN via a two week-old
central line. Her TPN is turned off so that she may be given a blood
transfusion. She receives one month-old blood through her central
venous catheter. Toward the end of her four hour transfusion, she
is noted to be hypothermic, hypotensive and comatose. What is
the most likely explanation?
a) transfusion reaction
b) hypoglycemia
c) hyperkalemia
d) candidemia
e) line sepsis with bacteremia

63. A critically ill patient has been receiving TPN via a two week-old
central line. Her TPN is turned off so that she may be given a blood
transfusion. She receives one month-old blood through her central
venous catheter. Toward the end of her four hour transfusion, she
is noted to be hypothermic, hypotensive and comatose. What is
the most likely explanation?
a) transfusion reaction
b) hypoglycemia
c) hyperkalemia
d) candidemia
e) line sepsis with bacteremia

64. What is the chance that any two siblings share


one haplotype?
a) 0%
b) 25%
c) 50%
d) 100%
e) not enough information

64. What is the chance that any two siblings share


one haplotype?
a) 0%
b) 25%
c) 50%
d) 100%
e) not enough information

65. Which of the following statements is true concerning congenital


lobar emphysema?

a) It is usually limited to the upper lobes and characterized by severe


air trapping.
b) Lung destruction associated with this condition is irreversible and
often requires pneumonectomy.
c) Treatment consists of conservative measures and rarely requires
excision of the affected lobe.
d) Most are located in the anterior mediastinum.
e) Like intralobar pulmonary sequestrations, they drain into the
azygous veins.

65. Which of the following statements is true concerning congenital


lobar emphysema?

a) It is usually limited to the upper lobes and characterized by


severe air trapping.
b) Lung destruction associated with this condition is irreversible and
often requires pneumonectomy.
c) Treatment consists of conservative measures and rarely requires
excision of the affected lobe.
d) Most are located in the anterior mediastinum.
e) Like intralobar pulmonary sequestrations, they drain into the
azygous veins.

66. T3 lesion of the colon or rectum means:


a) limited to the mucosa
b) invasion of submucosa
c) invasion of muscularis propria
d) invasion of subserosa or nonperitonealized pericolic fat
e) invasion of contiguous structures

66. T3 lesion of the colon or rectum means:


a) limited to the mucosa
b) invasion of submucosa
c) invasion of muscularis propria
d) invasion of subserosa or nonperitonealized pericolic
fat
e) invasion of contiguous structures

67. During normal activity, energy expenditure in a


healthy adult male in kcal/kg/day is approximately?
a) 15 kcal
b) 20 kcal
c) 35 kcal
d) 50 kcal
e) none of the above

67. During normal activity, energy expenditure in a


healthy adult male in kcal/kg/day is approximately?
a) 15 kcal
b) 20 kcal
c) 35 kcal
d) 50 kcal
e) none of the above

68. In the performance of a femoral hernia repair,


which of the following structures must be divided?
a)
b)
c)
d)
e)

transversalis fascia
lacunar ligament
inguinal ligament
internal oblique muscle
coopers ligament

68. In the performance of a femoral hernia repair,


which of the following structures must be divided?
a)
b)
c)
d)
e)

transversalis fascia
lacunar ligament
inguinal ligament
internal oblique muscle
coopers ligament

69. This is the normal ultrasound anatomy of the rectal wall.


Which of the rings represents the muscularis propria?

a) inner white line


b) first black line
c) middle white line
d) second black line
e) outer white line

69. This is the normal ultrasound anatomy of the rectal wall.


Which of the rings represents the muscularis propria?

a) inner white line


b) first black line
c) middle white line
d) second black line
e) outer white line

70. Concerning a femoropopliteal bypass: Which


structure is anterior to the graft at midthigh?
a) gracilis
b) adductor magnus
c) rectus femoris
d) femoral vein
e) deep femoral artery

70. Concerning a femoropopliteal bypass: Which


structure is anterior to the graft at midthigh?
a) gracilis
b) adductor magnus
c) rectus femoris
d) femoral vein
e) deep femoral artery

71. Which is the most common benign tumor of


the parotid?
A) Adenoid cystic
B) Acinic cell
C) Mucoepidermoid
D) Pleiomorphic adenoma

E) Warthins tumor

71. Which is the most common benign tumor of


the parotid?
A) Adenoid cystic
B) Acinic cell
C) Mucoepidermoid
D) Pleiomorphic adenoma

E) Warthins tumor

72. During the performance of a parotidectomy, you notice


facial nerve involvement by the tumor. What is the most
appropriate step in the management of the tumor.
A) Resect the superficial parotid gland and radiate the facial
nerve and deep portion of the gland.
B) Abort the procedure since the patient is incurable.
C) Resect the nerve and perform a nerve graft.
D) Resect nothing and give chemotherapy and radiation
therapy to the entire gland.
E) Resect the nerve and plan on a staged repair of the facial
nerve.

72. During the performance of a parotidectomy, you notice


facial nerve involvement by the tumor. What is the most
appropriate step in the management of the tumor.
A) Resect the superficial parotid gland and radiate the facial
nerve and deep portion of the gland.
B) Abort the procedure since the patient is incurable.
C) Resect the nerve and perform a nerve graft.
D) Resect nothing and give chemotherapy and radiation
therapy to the entire gland.
E) Resect the nerve and plan on a staged repair of the facial
nerve.

73. After a neck dissection, you notice the patient


has lost taste over the anterior ipsilateral tongue.
What nerve has most likely been injured?
A) Buccal
B) Masseter
C) Hypoglossal
D) Lingual
E) Marginal mandibular

73. After a neck dissection, you notice the patient


has lost taste over the anterior ipsilateral tongue.
What nerve has most likely been injured?
A) Buccal
B) Masseter
C) Hypoglossal
D) Lingual
E) Marginal mandibular

74. You have performed a laryngectomy/


pharyngectomy and post-operatively you notice that there is
cellulitis and purulent drainage from your neck incision. The patient is
febrile with an elevated WBC. What is the most appropriate step?

A)

Open the wound in the OR and attempt suture repair of


the pharyngeal anastomosis.

B)

Open the wound in the OR and obtain good drainage


without attempting to repair the pharyngeal anastomosis.

C)

Start the patient on IV antibiotics only.

D)

Place PEG and start IV antibiotics.

E)

Place PEG and attempt suture repair of the pharyngeal


anastomosis.

74. You have performed a laryngectomy/


pharyngectomy and post-operatively you notice that there is
cellulitis and purulent drainage from your neck incision. The patient is
febrile with an elevated WBC. What is the most appropriate step?

A)

Open the wound in the OR and attempt suture repair of


the pharyngeal anastomosis.

B)

Open the wound in the OR and obtain good drainage


without attempting to repair the pharyngeal
anastomosis.

C)

Start the patient on IV antibiotics only.

D)

Place PEG and start IV antibiotics.

E)

Place PEG and attempt suture repair of the pharyngeal


anastomosis.

75. A patient presents with a neck mass and FNA shows


squamous cell carcinoma. The patient has extremely
high EBV titers. Further work-up does not show any
primary source of tumor. What is the most likely source
of the primary?
A) Nasopharynx
B) Tonsillar pillar
C) Hard Palate
D) Soft Palate
E) Epiglottis

75. A patient presents with a neck mass and FNA shows


squamous cell carcinoma. The patient has extremely
high EBV titers. Further work-up does not show any
primary source of tumor. What is the most likely source
of the primary?
A) Nasopharynx
B) Tonsillar pillar
C) Hard Palate
D) Soft Palate
E) Epiglottis

76. A patient presents with an inguinal lymph node which is biopsied


and is positive for melanoma. Further work-up and physical exam does
not reveal a primary source of the tumor. What is the next most
appropriate step in management?

A) Sentinel lymph node biopsy of the inguinal lymph


node basin.

B) Sentinel lymph node biopsy of the contralateral


nodal basin.
C) Ipsilateral inguinal lymphadenectomy.
D) IFN-alpha if this is Stage 3 melanoma.
E) Il-2 treatment if this is Stage 4 melanoma.

76. A patient presents with an inguinal lymph node which is biopsied


and is positive for melanoma. Further work-up and physical exam does
not reveal a primary source of the tumor. What is the next most
appropriate step in management?

A) Sentinel lymph node biopsy of the inguinal lymph


node basin.

B) Sentinel lymph node biopsy of the contralateral


nodal basin.
C) Ipsilateral inguinal lymphadenectomy.
D) IFN-alpha if this is Stage 3 melanoma.
E) Il-2 treatment if this is Stage 4 melanoma.

77. A woman on mammography is noted to have a cluster


of microcalcifications. Excisional biopsy shows
atypical hyperplasia with LCIS. The margins are
negative. What is the next most appropriate step in
management?
A) Modified radical mastectomy
B) Lumpectomy alone
C) Lumpectomy and radiation
D) Lumpectomy and Tamoxifen
E) Observe

77. A woman on mammography is noted to have a cluster


of microcalcifications. Excisional biopsy shows
atypical hyperplasia with LCIS. The margins are
negative. What is the next most appropriate step in
management?
A) Modified radical mastectomy
B) Lumpectomy alone
C) Lumpectomy and radiation
D) Lumpectomy and Tamoxifen
E) Observe

78. A woman presents with a scaly rash on her nipple which is


tender and itches. Biopsy shows Pagets disease of the
breast. What would you expect of a mass on histology if
found in the excisional biopsy specimen?
A) Invasive lobular carcinoma
B) Invasive ductal carcinoma
C) Tubular carcinoma of the breast
D) Papillary carcinoma of the breast
E) Eczema

78. A woman presents with a scaly rash on her nipple which is


tender and itches. Biopsy shows Pagets disease of the
breast. What would you expect of a mass on histology if
found in the excisional biopsy specimen?
A) Invasive lobular carcinoma
B) Invasive ductal carcinoma
C) Tubular carcinoma of the breast
D) Papillary carcinoma of the breast
E) Eczema

79. A woman has an 8 cm cystosarcoma phyllodes tumor of


the breast and is small-breasted. What is the most
appropriate management?
A) Quandrantectomy

B) Wide local excision with 2 cm margins


C) Radiation and Tamoxifen
D) Total mastectomy
E) None of the above

79. A woman has an 8 cm cystosarcoma phyllodes tumor of


the breast and is small-breasted. What is the most
appropriate management?
A) Quandrantectomy

B) Wide local excision with 2 cm margins


C) Radiation and Tamoxifen
D) Total mastectomy
E) None of the above

80. A woman presents with peau-d orange in the lower half


of her breast. Her breast is markedly abnormal on
examination. There is no palpable lymphadenopathy.
What is the next most appropriate step in management?

A) Total mastectomy
B) Toilet mastectomy
C) Modified radical mastectomy
D) Radical mastectomy
E) Chemotherapy

80. A woman presents with peau-d orange in the lower half


of her breast. Her breast is markedly abnormal on
examination. There is no palpable lymphadenopathy.
What is the next most appropriate step in management?

A) Total mastectomy
B) Toilet mastectomy
C) Modified radical mastectomy
D) Radical mastectomy
E) Chemotherapy

81. A woman has microcalcifications on mammogram


and biopsy shows DCIS. Which algorithim is
associated with the lowest rate of recurrence?
A) Lumpectomy and RT
B) Lumpectomy, RT and axillary dissection
C) Lumpectomy, RT and Tamoxifen
D) Lumpectomy and Tamoxifen
E) Total mastectomy

81. A woman has microcalcifications on mammogram


and biopsy shows DCIS. Which algorithim is
associated with the lowest rate of recurrence?
A) Lumpectomy and RT
B) Lumpectomy, RT and axillary dissection
C) Lumpectomy, RT and Tamoxifen
D) Lumpectomy and Tamoxifen
E) Total mastectomy

82. A 78 y.o. woman has a node-negative breast


cancer with positive ER/PR. Her stage is T1CN0M0.
What is the most appropriate therapy?
A) Tamoxifen for 1 year
B) Tamoxifen for 5 years
C) CMF followed by Tamoxifen for 1 year
D) CMF followed by Tamoixfen for 5 years
E) None of the above

82. A 78 y.o. woman has a node-negative breast


cancer with positive ER/PR. Her stage is T1CN0M0.
What is the most appropriate therapy?
A) Tamoxifen for 1 year
B) Tamoxifen for 5 years
C) CMF followed by Tamoxifen for 1 year
D) CMF followed by Tamoixfen for 5 years
E) None of the above

83. Which drug increases the cardiac output and


increases the systemic vascular resistance?
A) Isoproterenol
B) Milrinone
C) Amrinone

D) Norepinephrine
E) Dobutamine

83. Which drug increases the cardiac output and


increases the systemic vascular resistance?
A) Isoproterenol
B) Milrinone
C) Amrinone

D) Norepinephrine
E) Dobutamine

84. In a patient who is POD #6 from an abdominal


surgery, what is the most likely cause of renal and
liver failure?
A) Infection
B) CHF
C) Drug toxicity

D) Abdominal compartment syndrome


E) Anesthetic reaction and shockliver

84. In a patient who is POD #6 from an abdominal


surgery, what is the most likely cause of renal and
liver failure?
A) Infection
B) CHF
C) Drug toxicity

D) Abdominal compartment syndrome


E) Anesthetic reaction and shockliver

85. A patient is given Cefotan for antibiotic prophylaxis and has an


uneventful operation. On POD # 3, the patient develops
jaundice and has an elevated bilirubin and dark urine. What has
most likely occurred?

A) Liver failure secondary to anesthetics given at


surgery.

B) Hemolytic immune reaction to Cefotan.


C) Hemolytic immune reaction to anesthetics.

D) TTP secondary to Cefotan.


E) Development of idiopathic hemolytic uremia
secondary to Cefotan.

85. A patient is given Cefotan for antibiotic prophylaxis and has an


uneventful operation. On POD # 3, the patient develops
jaundice and has an elevated bilirubin and dark urine. What has
most likely occurred?

A) Liver failure secondary to anesthetics given at


surgery.

B) Hemolytic immune reaction to Cefotan.


C) Hemolytic immune reaction to anesthetics.

D) TTP secondary to Cefotan.


E) Development of idiopathic hemolytic uremia
secondary to Cefotan.

86. A 68 y.o. man who had a recent MI develops massive


UGIB. It is noted to be coming from esophageal
varices. What is the best agent to treat this patient
with?
A) Somatostatin
B) Nitrates
C) Vasopressin
D) Dopamine
E) Dobutamine

86. A 68 y.o. man who had a recent MI develops massive


UGIB. It is noted to be coming from esophageal
varices. What is the best agent to treat this patient
with?
A) Somatostatin
B) Nitrates
C) Vasopressin
D) Dopamine
E) Dobutamine

87. Which is an absolute contraindication to the use


of the intra-aortic balloon pump (IABP)?
A) Aortic insufficiency
B) Aortic stenosis
C) Mitral insufficiency
D) Mitral stenosis
E) Pulmonary stenosis

87. Which is an absolute contraindication to the use


of the intra-aortic balloon pump (IABP)?
A) Aortic insufficiency
B) Aortic stenosis
C) Mitral insufficiency
D) Mitral stenosis
E) Pulmonary stenosis

88. A patient is on TPN and has an RQ of 1.05. He is also


hypercarbic and has a high respiratory rate of 36
bpm. What is most likely the explanation for this
occurrence?
A) excessive lipid ingestion
B) deficient lipid ingestion
C) excessive carbohydrate administration
D) insufficient carbohydrate administration
E) lipogenesis

88. A patient is on TPN and has an RQ of 1.05. He is also


hypercarbic and has a high respiratory rate of 36
bpm. What is most likely the explanation for this
occurrence?
A) excessive lipid ingestion
B) deficient lipid ingestion
C) excessive carbohydrate administration
D) insufficient carbohydrate administration
E) lipogenesis

89. Vinyl chloride is associated with which


malignancy?
A) Small cell carcinoma of the lung

B) Mesothelioma
C) Struma ovarii

D) Hemangiosarcoma
E) Angiosarcoma of the liver

89. Vinyl chloride is associated with which


malignancy?
A) Small cell carcinoma of the lung

B) Mesothelioma
C) Struma ovarii

D) Hemangiosarcoma
E) Angiosarcoma of the liver

90. Which of the following is the most appropriate candidate for gastric
bypass?
A) 70 y.o. healthy woman with no medical problems and BMI = 42.

B) 35 y.o. woman with diabetes, severe OSA, HTN, high chol with
BMI = 34.
C) 42 y.o. man s/p recent MI 2 months ago with diabetes, high chol
and BMI = 45.
D) 55 y.o. woman on prednisone for severe asthma and BMI = 44.

E) 47 y.o woman with HTN, diabetes, severe OSA, arthritis and GERD
with BMI = 36.

90. Which of the following is the most appropriate candidate for gastric
bypass?
A) 70 y.o. healthy woman with no medical problems and BMI = 42.

B) 35 y.o. woman with diabetes, severe OSA, HTN, high chol with
BMI = 34.
C) 42 y.o. man s/p recent MI 2 months ago with diabetes, high chol
and BMI = 45.
D) 55 y.o. woman on prednisone for severe asthma and BMI = 44.

E) 47 y.o woman with HTN, diabetes, severe OSA, arthritis and


GERD with BMI = 36.

91. What is the next appropriate step in the management of a


21 y.o woman with a lateral anal fissure?
A) Lateral internal sphincterotomy.

B) Wide incision and drainage.


C) Wide incision and drainage with seton placement.
D) Seton placement without I & D.

E) Barium enema.

91. What is the next appropriate step in the management of a


21 y.o woman with a lateral anal fissure?
A) Lateral internal sphincterotomy.

B) Wide incision and drainage.


C) Wide incision and drainage with seton placement.
D) Seton placement without I & D.

E) Barium enema.

You might also like